CAT Questions | CAT Reading Comprehension

CAT Verbal Ability | RC for CAT

The following questions are from Reading Comprehension for Verbal Ability for CAT. Practice RC Passages for CAT. Please scroll the page to see them all. Reading Comprehension questions are an integral part of the CAT Exam. RC Passages and the accompanying questions account for around 24 questions out of 34 questions in CAT Verbal Section. If you would like to take these questions as a Quiz, head on here to take these questions in a test format, absolutely free.

Passage 1 : CAT Reading Comprehension: Power in language

The first systems of writing developed and used by the Germanic peoples were runic alphabets. The runes functioned as letters, but they were much more than just letters in the sense in which we today understand the term. Each rune was an ideographic or pictographic symbol of some cosmological principle or power, and to write a rune was to invoke and direct the force for which it stood. Indeed, in every Germanic language, the word “rune” (from Proto-Germanic *runo) means both “letter” and “secret” or “mystery,” and its original meaning, which likely predated the adoption of the runic alphabet, may have been simply “(hushed) message.”

Each rune had a name that hinted at the philosophical and magical significance of its visual form and the sound for which it stands, which was almost always the first sound of the rune’s name. For example, the T-rune, called *Tiwaz in the Proto-Germanic language, is named after the god Tiwaz (known as Tyr in the Viking Age). Tiwaz was perceived to dwell within the daytime sky, and, accordingly, the visual form of the T-rune is an arrow pointed upward (which surely also hints at the god’s martial role). The T-rune was often carved as a standalone ideograph, apart from the writing of any particular word, as part of spells cast to ensure victory in battle.

The runic alphabets are called “futharks” after the first six runes (Fehu, Uruz, Thurisaz, Ansuz, Raidho, Kaunan), in much the same way that the word “alphabet” comes from the names of the first two Hebrew letters (Aleph, Beth). There are three principal futharks: the 24-character Elder Futhark, the first fully-formed runic alphabet, whose development had begun by the first century CE and had been completed before the year 400; the 16-character Younger Futhark, which began to diverge from the Elder Futhark around the beginning of the Viking Age (c. 750 CE) and eventually replaced that older alphabet in Scandinavia; and the 33-character Anglo-Saxon Futhorc, which gradually altered and added to the Elder Futhark in England. On some inscriptions, the twenty-four runes of the Elder Futhark were divided into three ættir (Old Norse, “families”) of eight runes each, but the significance of this division is unfortunately unknown.

Runes were traditionally carved onto stone, wood, bone, metal, or some similarly hard surface rather than drawn with ink and pen on parchment. This explains their sharp, angular form, which was well-suited to the medium.

Much of our current knowledge of the meanings the ancient Germanic peoples attributed to the runes comes from the three “Rune Poems,” documents from Iceland, Norway, and England that provide a short stanza about each rune in their respective futharks (the Younger Futhark is treated in the Icelandic and Norwegian Rune Poems, while the Anglo-Saxon Futhorc is discussed in the Old English Rune Poem).

While runologists argue over many of the details of the historical origins of runic writing, there is widespread agreement on a general outline. The runes are presumed to have been derived from one of the many Old Italic alphabets in use among the Mediterranean peoples of the first century CE, who lived to the south of the Germanic tribes. Earlier Germanic sacred symbols, such as those preserved in northern European petroglyphs, were also likely influential in the development of the script.

The earliest possibly runic inscription is found on the Meldorf brooch, which was manufactured in the north of modern-day Germany around 50 CE. The inscription is highly ambiguous, however, and scholars are divided over whether its letters are runic or Roman. The earliest unambiguous runic inscriptions are found on the Vimose comb from Vimose, Denmark and the Øvre Stabu spearhead from southern Norway, both of which date to approximately 160 CE. The earliest known carving of the entire futhark, in order, is that on the Kylver stone from Gotland, Sweden, which dates to roughly 400 CE.

The transmission of writing from southern Europe to northern Europe likely took place via Germanic warbands, the dominant northern European military institution of the period, who would have encountered Italic writing firsthand during campaigns amongst their southerly neighbors. This hypothesis is supported by the association that runes have always had with the god Odin, who, in the Proto-Germanic period, under his original name *Woðanaz, was the divine model of the human warband leader and the invisible patron of the warband’s activities. The Roman historian Tacitus tells us that Odin (“Mercury” in the interpretatio romana) was already established as the dominant god in the pantheons of many of the Germanic tribes by the first century.

From the perspective of the ancient Germanic peoples themselves, however, the runes came from no source as mundane as an Old Italic alphabet. The runes were never “invented,” but are instead eternal, pre-existent forces that Odin himself discovered by undergoing a tremendous ordeal.

  1. The word “pantheon” in the passage refers to

    1. A temple of all the gods
    2. All the gods collectively of a religion
    3. A monument or building commemorating a nation's dead heroes
    4. A domed circular temple at Rome, erected a.d. 120–124 by Hadrian
    Choice B

  2. Which of the following statements is incorrect?

    1. Unlike the Latin alphabet, which is an essentially utilitarian script, the runes are symbols of some of the most powerful forces in the cosmos
    2. Runic writing was probably first used in southern Europe and was carried north by Germanic tribes.
    3. The word “rune” and its meaning was derived from the runic alphabet.
    4. The first runic alphabets date back to the 1st century CE.
    Choice C

  3. Which of the following can be inferred from the passage?
    a. Runic script was most likely derived from Old Italic script.
    b. Runes were not used so much as a simple writing system, but rather as magical signs to be used for charms.
    c. In the Proto-Germanic period, the god Tiwaz was associated with war, victory, marriage and the diurnal sky.
    d. The knowledge of the meanings attributed to the runes of the Younger Futhark is derived from the three Rune poems.

    1. All the above
    2. ii and iv
    3. i, ii and iv
    4. i and iii
    Choice D

  4. Which of the following cannot be reasonably inferred with regard to the beliefs of the Proto-Germanic people?

    1. Odin came upon the runes after going through a lot of torment.
    2. The name of a rune was almost always the first sound of a God’s name
    3. The cosmological power represented by a rune was invoked by writing it.
    4. Proto-German Gods were modeled on humans.
    Choice B

Passage 2 : CAT Reading Comprehension: Upholding the Law

Hard cases, it is said, make bad law. The adage is widely considered true for the Supreme Court of India which held in the height of the Emergency, in ADM Jabalpur v. Shivkant Shukla that detenus under the Maintenance of Internal Security Act (MISA) could not approach the judiciary if their fundamental rights were violated. Not only was the law laid down unconscionable, but it also smacked of a Court more “executive-minded than the executive”, complicit in its own independence being shattered by an all-powerful government. So deep has been the impact of this judgment that the Supreme Court’s current activist avatar is widely viewed as having its genesis in a continuing need to atone. Expressions of such atonement have created another Court made to measure — this time not to the measure of the government but rather the aggrandised self-image of some of its judges.

Let us look back to the ADM Jabalpur case. As a court of law, the Supreme Court was called upon in the case to balance the interest of public order in an Emergency with the right to life and personal liberty guaranteed to every person. Nine High Courts called upon to perform the same function had found a nuanced answer by which they had held that the right to life cannot be absolutely subservient to public order merely because the government declared so — the legality of detentions could be judicially reviewed, though the intention of the government would not be second-guessed by the Court. This was a delicate balance. The Supreme Court however reversed this view and made the right to life and personal liberty literally a bounty of the government. Given that the consequences of their error were entirely to the government’s advantage, it was widely viewed as the death of an independent judiciary. The excessively deferential, almost apologetic language used by the judges confirmed this impression.

Today, however, while public interest litigation has restored the independent image of the Supreme Court, it has achieved this at the cost of quality, discipline and the constitutional role judges are expected to perform. The Court monitors criminal trials, protects the environment, regulates political advertising, lays down norms for sexual harassment in the workplace, sets guidelines for adoption, supervises police reform among a range of other tasks of government. That all these tasks are crucial but tardily undertaken by government can scarcely be questioned. But for an unelected and largely unaccountable institution such as the Supreme Court to be at the forefront of matters relating to governance is equally dangerous — the choice of issues it takes up is arbitrary, their remit is not legal, their results often counterproductive, requiring a degree of technical competence and institutional capacity in ensuring compliance that the Court simply does not possess. This sets an unhealthy precedent for other courts and tribunals in the country, particularly the latter whose chairpersons are usually retired Supreme Court Justices. To take a particularly egregious example, the National Green Tribunal has banned diesel vehicles more than 10 years old in Delhi and if reports are to be believed, is considering imposing a congestion charge for cars as well. That neither of these are judicial functions and are being unjustly being usurped by a tribunal that has far exceeded its mandate, is evidence of the chain reaction that the Supreme Court’s activist avatar has set off across the judicial spectrum.

Finally, the Court’s activism adds to a massive backlog of regular cases that makes the Indian justice delivery mechanism, slow, unreliable and inefficient for the ordinary litigant. As on March 1, 2015, there were over 61,000 cases pending in the Supreme Court alone. It might be worthwhile for the Court to set its own house in order, concomitantly with telling other wings of government how to do so.

As we mark 40 years of the Emergency and the darkest period in the Supreme Court’s history, it might be time to not single-mindedly harp on the significance of an independent judiciary. Judicial independence, is and must remain a cherished virtue. However, it would be blinkered to not confront newer challenges that damage the credibility of our independent judiciary today — unpardonable delays and overweening judges taking on the mantle of national government by proxy. The Supreme Court 40 years on is a different institution — it must be cognizant of its history but not at the cost of being blind to its present.

  1. Which of the following is a suitable title for the passage?

    1. An Atonement Gone Too Far
    2. Sanctimony from a Ruined Pedestal
    3. The ADM Jabalpur's Case: The Supreme Court's Darkest Hour
    4. Overcompensating for Past Mistakes
    Choice A

  2. The author says that the Supreme Court was “more executive-minded than the executive” during the Emergency. Which of the following options captures the essence of what the writer means by the phrase: 'more “executive-minded than the executive”'?

    1. The Supreme Court abdicated its independence to an authoritarian government by embracing its perspective.
    2. The Supreme Court was more emphatic than the Government about exercising executive power under the MISA.
    3. The Supreme Court reflected the unconscionable actions taken by the government by upholding its laws.
    4. The Supreme Court wanted to curry favor with the government through its deferential decisions during Emergency.
    Choice B

  3. Which of the following cannot be reasonably inferred from the passage?

    1. The Supreme Court was complicit in curbing judicial independence during the Emergency.
    2. Public interest litigations have, post-Emergency, led to the judiciary overreaching into the realm of legislature.
    3. The Indian Judiciary ought not indulge in general supervisory jurisdiction to correct actions and policies of government.
    4. The Indian judiciary must be equipped with technical competence and institutional capacity to ensure compliance to orders passed in relation to public interest litigations.
    Choice D

  4. The word “egregious” in the passage is farthest in meaning to :

    1. outrageous
    2. flagitious
    3. distinguished
    4. arrant
    Choice C

  5. Which of the following is the author least likely to agree with?

    1. The rise in judicial activism is in danger making the Supreme Court diffuse and ineffective, encroaching into the functions of government.
    2. Where the Supreme Court is only moved for better governance and administration, which does not involve the exercise of any proper judicial function, it should refrain from acting.
    3. Adoption, police reform and environment issues are the remit of the judiciary.
    4. The Indian judicial system needs to focus on clearing the massive backlog of cases to re-establish its credibility.
    Choice C

Passage 3 : CAT Reading Comprehension: Upholding the Law

Sound the alarm! The kingdom of letters has admitted Trojan horses: James Frey, JT Leroy, Misha Defonseca, Margaret B. Jones, Herman Rosenblat, and now Matt McCarthy, portions of whose baseball memoir, the New York Times reports, are “incorrect, embellished or impossible.” The watchmen have let down their guards.

I write: Hold your horses. In the rush to diagnose these fake memoirs as symptoms of a diseased culture, we have failed to consider an equally plausible alternative. What if the exposure of fake memoirists is not due to an increased frequency of lying, but rather to our increased ability to root out liars and hold them accountable for their verisimilitudes? Perhaps the outings of these hoaxes mark not a blurring of the line between fact and fiction, but a further demarcation.

Indeed, it may be helpful to remember that the novel was born from exactly such confusion. One of the standards by which the earliest novels were judged was their ability to convince readers that their narratives were, in fact, real. Authors deployed several tricks to scaffold the illusion. 'Robinson Crusoe' was “written by himself,” according to the novel’s title page, which omitted Daniel Defoe’s name. Samuel Richardson’s novel 'Pamela', an attempt to instruct in good conduct through entertainment, was written as a series of letters penned by the heroine. In his preface to the novel, which excluded his name altogether, Richardson included several real letters from friends to whom he had shown the manuscript, but he changed the salutation from “Dear Author” to “Dear Editor” and even, writing under the guise of “editor,” praised “Pamela’s” letters. However, this was a lie, but not a hoax. Richardson wanted his novels to be read with "Historical Faith", since they contained, he believed, "the truth of the possible- the truth of human nature". Richardson’s authorship was revealed shortly after Pamela’s publication, but rather than serving time on Oprah’s couch, he was hailed as an innovator of the novelistic form.

Whereas novels were unashamedly fake memoirs at their conception, our recent hoaxes suggest that the line between the genres, once drawn, cannot easily be erased. This is in no small part due to the Internet’s surveillance. All along, historians had raised questions about Misha Defonseca, who claimed to have survived the Holocaust by living with a pack of wolves, but the engine of her downfall was her former publisher Jane Daniel’s blog. James Frey’s sine qua non of the fudged-memoir genre, A Million Little Pieces, was debunked by the website The Smoking Gun, which posted his actual arrest records and compared them to Frey’s embellished retellings. Deborah Lipstadt used her blog to gather evidence against Herman Rosenblat’s memoir.

If anything, you could argue that the fact-checkers are doing too good a job. There seems to be some risk that, in attempting to hold memoirs to journalistic standards of factuality, the watchdogs miss the forest for the trees, fixating on minor details in books whose general pictures are correct. The New York Times includes in its dossier against Matt McCarthy disputations by teammates who McCarthy alleges threatened children and made fun of Hispanics, as though their denials of having said such self-incriminating things were more trustworthy than McCarthy’s accusations. When Jose Canseco published his baseball memoirs Juiced and Vindicated, reviewers caviled over minor details and unsubstantiated claims, including that Alex Rodriguez had used steroids. Recent events have proven the gist of Canseco’s memoirs largely correct.

Indeed, it seems unlikely that, say, every claim in Casanova’s The Story of My Life would hold up to such scrutiny. And yet, if we knew this were the case, would we excise it from the canon? Writers’ enormous talents can sometimes render moot questions of their works’ factuality; our fraudsters, meanwhile, attempted to compensate for their meager talents by actually inhabiting their bloated fictions. They suffer not an excess of imagination, which can illuminate even the most mundane experiences, but a retreat from it. And yet simply because they lost their handles on the truth does not mean that the culture also has. Maybe the symptom of our age is not the fake memoirists themselves, but the catching of fake memoirists. In which case: Sound the church bells! The traitors are routed! The watchmen won!

  1. Which of the following is a suitable title for the passage?

    1. How to write a memoir
    2. The age of literary fraud
    3. Who is afraid of fake memoirists
    4. Writing in the age of the internet
    Choice C

  2. Which of the following is the author unlikely to agree with?

    1. There isn’t more literary fraud in our age. More fraud is coming to light due to the Internet’s surveillance.
    2. The line dividing novels and fake memoirs was never clear.
    3. As long as the main or essential part of a memoir is correct, it does not matter if lesser details do not stand up to verification.
    4. There exists now a widespread, diseased culture of literary fraud.
    Choice D

  3. With regard to the novel ‘Pamela’, the author states that Richardson’s artifice “ was a lie, but not a hoax”. What does he mean?

    1. It was an unintentional deception that contained the truth of human nature and was hence acceptable to readers.
    2. It was just a ploy to capture the imagination of the readers with the truth of the possible.
    3. It was a deception perpetrated simply to make money.
    4. It was a mere prank, and did not generate public interest.
    Choice B

  4. The word ‘verisimilitude’ in the passage is farthest in meaning to

    1. absurdity
    2. plausibility
    3. authenticity
    4. credibleness
    Choice A

Passage 4 : CAT Reading Comprehension: Power in language

Considered amongst the greatest works of Western literature, the Iliad, paired with its sequel, the Odyssey, is attributed to Homer.

However, that the author of the Iliad was not the same as the compiler of the fantastic tales in the Odyssey is arguable on several scores. The two epics belong to different literary types; the Iliad is essentially dramatic in its confrontation of opposing warriors who converse like the actors in Attic tragedy, while the Odyssey is cast as a novel narrated in more everyday human speech. In their physical structure, also, the two epics display an equally pronounced difference. The Odyssey is composed in six distinct cantos of four chapters ("books") each, whereas the Iliad moves unbrokenly forward with only one irrelevant episode in its tightly woven plot. Readers who examine psychological nuances see in the two works some distinctly different human responses and behavioral attitudes. For example, the Iliad voices admiration for the beauty and speed of horses, while the Odyssey shows no interest in these animals. The Iliad dismisses dogs as mere scavengers, while the poet of the Odyssey reveals a modern sentimental sympathy for Odysseus's faithful old hound, Argos.

But the most cogent argument for separating the two poems by assigning them to different authors is the archeological criterion of implied chronology. In the Iliad the Phoenicians are praised as skilled craftsmen working in metal and weavers of elaborate, much-prized garments. The shield which the metalworking god Hephaistos forges for Achilles in the Iliad seems inspired by the metal bowls with inlaid figures in action made by the Phoenicians and introduced by them into Greek and Etruscan commerce in the 8th century B.C. In contrast, in the Odyssey Greek sentiment toward the Phoenicians has undergone a drastic change. Although they are still regarded as clever craftsmen, in place of the Iliad's laudatory polydaidaloi ("of manifold skills") the epithet is parodied into polypaipaloi ("of manifold scurvy tricksters"), reflecting the competitive penetration into Greek commerce by traders from Phoenician Carthage in the 7th century B.C.

One thing, however, is certain: both epics were created without recourse to writing. Between the decline of Mycenaean and the emergence of classical Greek civilization—which is to say, from the late 12th to the mid-8th century B.C.—the inhabitants of the Greek lands had lost all knowledge of the syllabic script of their Mycenaean fore-bears and had not yet acquired from the easternmost shore of the Mediterranean that familiarity with Phoenician alphabetic writing from which classical Greek literacy (and in turn, Etruscan, Roman, and modern European literacy) derived. The same conclusion of illiterate composition may be reached from a critical inspection of the poems themselves. Among many races and in many different periods there has existed (and still exists sporadically) a form of purely oral and unwritten poetic speech, distinguishable from normal and printed literature by special traits that are readily recognizable and specifically distinctive. To this class the Homeric epics conform. Hence it would seem an inevitable inference that they must have been created either before the end of the 8th century B.C. or so shortly after that date that the use of alphabetic writing had not yet been developed sufficiently to record lengthy compositions. It is this illiterate environment that explains the absence of all contemporary historical record of the authors of the two great epics.

It is probable that Homer's name was applied to two distinct individuals differing in temperament and artistic accomplishment, born perhaps as much as a century apart, but practicing the same traditional craft of oral composition and recitation. Although each became known as "Homer, " it may be (as one ancient source asserts) that “homros “was a dialectal word for a blind man and so came to be used generically of the old and often sightless wandering reciters of heroic legends in the traditional meter of unrhymed dactylic hexameters. Thus there could have been many Homers. The two epics ascribed to Homer, however, have been as highly prized in modern as in ancient times for their marvelous vividness of expression, their keenness of personal characterization, their unflagging interest, whether in narration of action or in animated dramatic dialogue.

  1. Which of the following cannot be reasonably inferred from the passage?

    1. Before the 12th century BC , the use of syllabic writing existed in Ancient Greece.
    2. Phoenician traders flourished in Greece at the time the Homeric epics were composed.
    3. Greek, Roman and modern European literacy can be traced back to the Phoenicians.
    4. Iliad and Odyssey are purely oral poetic speech, set to rhyme.
    Choice D

  2. Which of the following can be characterized as the main idea of the passage?

    1. There could have been many Homers, old and often sightless wandering reciters of heroic legends.
    2. Attributing the composition of the Iliad and Odyssey to one Homer is erroneous.
    3. Both Iliad and Odyssey were created without recourse to writing.
    4. The Iliad and the Odyssey are of distinct literary types, physical structure and style.
    Choice B

  3. The term epithet as used in the passage is farthest in meaning to

    1. Sobriquet
    2. Moniker
    3. Nickname
    4. Jargon
    Choice D

The following questions are from IPMAT Rohtak and Indore sample papers. If you want to take these questions as a mock please click below.

IPMAT Rohtak Sample Paper Mock
IPMAT Indore Sample Paper Mock

Please note that the explanation button will take you to the IPMAT solution page.


Passage 5 : IPMAT Reading Comprehension

Read the passage below and answer the questions that follow :

Harold a professional man who had worked in an office for many years had a fearful dream. In it, he found himself in a land where small slug-like animals with slimy tentacles lived on people's bodies. The people tolerated the loathsome creatures because after many years they grew into elephants which then became the nation's system of transport, carrying everyone wherever he wanted to go. Harold suddenly realised that he himself was covered with these things, and he woke upscreaming. In a vivid sequence of pictures this dream dramatised for Harold what he had never been able to put in to words; he saw himself as letting society feed on his body in his early years so that it would carry him when he retired. He later threw off the "security bug" and took upfreelance work.

  1. IPMAT 2020 Sample Paper - IPM Rohtak Verbal Ability

    In his dream Harold found the loathsome creatures

    1. in his village
    2. in his own house
    3. in a different land
    4. in his office
    Choice C
    in a different land

  2. IPMAT 2020 Sample Paper - IPM Rohtak Verbal Ability

    Which one of the following phrases best helps to bring out the precise meaning of 'loathsome creatures'?

    1. Security bug and slimy tentacles
    2. Fearful dream and slug-like animals
    3. Slimy tentacles and slug-like animals
    4. slug-like animals and security bug
    Choice C
    Slimy tentacles and slug-like animals

  3. IPMAT 2020 Sample Paper - IPM Rohtak Verbal Ability

    The statement that 'he later threw off the security bug' means that

    1. Harold succeeded in overcoming the need for security
    2. Harold stopped giving much importance to dreams
    3. Harold started tolerating social victimisation
    4. Harold killed all the bugs troubled him
    Choice A
    Harold succeeded in overcoming the need for security

  4. IPMAT 2020 Sample Paper - IPM Rohtak Verbal Ability

    Harold's dream was fearful because

    1. it brought him face to face with reality
    2. it was full of vivid pictures of snakes
    3. he saw huge elephant in it
    4. in it he saw slimy creatures feeding on people's bodies
    Choice D
    in it he saw slimy creatures feeding on people's bodies

Passage 6 : IPMAT Reading Comprehension

Read the passage below and answer the questions that follow :

Concussions are brain injuries that occur when a person receives a blow to the head, face, or neck. Although most people who suffer a concussion experience initial bouts of dizziness, nausea, and drowsiness, these symptoms often disappear after a few days. The long-term effects of  concussions,however, are less understood and far more severe. 

Recent studies suggest that people who suffer multiple concussions are at significant risk for developing chronic traumatic encephalopathy (CTE), a degenerative brain disorder that causes a variety of dangerous mental and emotional problems to arise weeks, months, or even years after the initial injury. These psychological problems can include depression, anxiety, memory loss, inability to concentrate, and aggression. In extreme cases, people suffering from CTE have even committed suicide or homicide. The majority of people who developthese issues are athletes who participate in popular high-impact sports, especially football. Although new sports regulations and improvements in helmet technology can helpprotect players, amateur leagues, the sports media, and fans all bear some of the responsibility for reducing the incidence of these devastating injuries.Improvements in diagnostic technology have provided substantial evidence to link severe and often fatal psychological disorders to the head injuries that players receive while on the field. 

Recent autopsies performed on the brains of football players who have committed suicide have shown advanced cases of CTE in every single victim.In response to the growing understanding of this danger, the National Football League (NFL) has revised its safety regulations. Players who have suffered a head injury on the field must undergo a concussion sideline assessmenta series of mental and physical fitness tests before being allowed back in the game.

In an effort to diminish the amount of head and neck injuries on the field, NFL officials began enforcing stricter penalty calls for helmet-to-helmet contact, leading with the head, and hitting a defenseless player. Furthermore, as of 2010, if a player's helmet is accidentally wrenched from his head during play, the ball is immediately whistled dead. It is hoped that these new regulations, coupled with advances in helmet design, will reduce the number of concussions, and thus curb further cases of CTE. Efforts by the NFL and other professional sports leagues are certainly laudable; we should commend every attempt to protect the mental and physical health of players. However, new regulations at the professional level cannot protect amateur players, especially young people.

Fatal cases of CTE have been reported in victims as young as 21. Proper tackling form using the arms and shoulders to aim for a player's midsection should be taught at an early age. Youth, high school, and college leagues should also adopt safety rules even more stringent than those of the NFL. Furthermore, young athletes should be educated about the serious dangers of head injuries at an early age. Perhaps the most important factor in reducing the number of traumatic brain injuries, however, lies not with the players, the coaches, or the administrators, but with the media and fans. 

Sports media producers have become accustomed to showcasing the most aggressive tackles and the most intense plays. NFL broadcasts often replay especially violent collisions while the commentators marvel at the players physical prowess. Some sports highlights television programs even feature weekly countdowns of the hardest hits. When the media exalts such dangerous behavior, professionals are rewarded for injuring each other on the field and amateurs become more likely to try to imitate their favorite NFL athletes. Announcers, commentators, television producers, and sportswriters should engage in a collective effort to cease glorifying brutal plays. In turn, fans should stopexpecting their favorite players to put their lives on  the line for the purposes of entertainment. Players must not be encouraged to trade their careers, their health, their happiness, and even their lives for the sake of a game.

  1. IPMAT 2020 Sample Paper - IPM Rohtak Verbal Ability

    Based on information in the passage, it can be inferred that all of the following statements are true except

    1. tackling is not always dangerous; however, players who use improper tackling form may injure others
    2. scientists have established a definitive link between players who die untimely deaths and the onset of CTE
    3. NFL officials have done little to address the problem of CTE
    4. athletes who are praised for exceptionally brutal hits are likely to continue engaging in such dangerous behavior
    Choice C
    NFL officials have done little to address the problem of CTE

  2. IPMAT 2020 Sample Paper - IPM Rohtak Verbal Ability

    According to the passage, which of the following factors contribute(s) to the incidence of CTE in
    amateur players?
    I. inconsistent application of safety regulations for all levels
    II. lack of education about the dangers of head injuries
    III. amateur players' desire to emulate professionals

    1. l only
    2. II only
    3. I and II only
    4. II and III only
    Choice C
    I and II only

  3. IPMAT 2020 Sample Paper - IPM Rohtak Verbal Ability

    As used in paragraph 3, which is the best synonym for laudable?

    1. praiseworthy
    2. ineffectual
    3. memorable
    4. audacious
    Choice A
    praiseworthy

  4. IPMAT 2020 Sample Paper - IPM Rohtak Verbal Ability

    The author's tone in the final paragraph can best be described as

    1. remorseful
    2. hopeless
    3. perplexed
    4. insistent
    Choice D
    insistent

  5. IPMAT 2020 Sample Paper - IPM Rohtak Verbal Ability

    As used in the final paragraph, which is the best antonym for exalts?

    1. mitigates
    2. venerates
    3. mollifies
    4. expedites
    Choice A
    mitigates

  6. IPMAT 2020 Sample Paper - IPM Rohtak Verbal Ability

    In describing the sports media, the author emphasizes its

    1. responsibility
    2. entertainment value
    3. senselessness
    4. danger
    Choice A
    responsibility

  7. IPMAT 2020 Sample Paper - IPM Rohtak Verbal Ability

    In the final paragraph, the author mentions sports highlights television programs as an example of how
    I. the media glorifies violence
    II. amateurs learn to mimic professional athletes
    III. professional athletes gain approval

    1. I only
    2. II only
    3. I and II only
    4. II and III only
    Choice A
    I only

Passage 7 : IPMAT Reading Comprehension

Read the passage below and answer the questions that follow :

The ravages [of the storm] were terrible in America, Europe, and Asia. Towns were overthrown, forests uprooted, coasts devastated by the mountains of water which were precipitated on them, vessels cast on the shore, whole districts leveled by waterspouts, several thousand people crushed on land or drowned at sea; such were the traces of its (468) ________, left by this devastating tempest.
                       -Ralph Waldo Emerson

  1. IPMAT 2020 Sample Paper - IPM Rohtak Verbal Ability

    What is the meaning of the word over throwning the context of this passage?

    1. surrendered
    2. devastated
    3. capitulated
    4. Yielded
    Choice B
    devastated

  2. IPMAT 2020 Sample Paper - IPM Rohtak Verbal Ability

    What is the meaning of the word leveled as it is used in the passage?

    1. razed
    2. marked
    3. spread
    4. raised
    Choice A
    razed

  3. IPMAT 2020 Sample Paper - IPM Rohtak Verbal Ability

    Which word, if inserted in the blank, makes the most sense in the context of the passage?

    1. Velocity
    2. Fury
    3. reward
    4. benevolence
    Choice B
    Fury

Passage 8 : IPMAT Reading Comprehension

Read the following passage and choose the answer that is closest to each of the questions that are based on the passage.

Supposing half a dozen or a dozen men were cast ashore from a wreck on an uninhabited island and left to their own resources, one of course, according to his capacity, would be set to one business and one to another; the strongest to dig and to cut wood, and to build huts for the rest: the most dexterous to make shoes out of bark and coats out of skins; the best educated to look for iron or lead in the rocks, and to plan the channels for the irrigation of the fields. But though their labours were thus naturally severed, that small group of shipwrecked men would understand well enough that the speediest progress was to be made by helping each other-not by opposing each other; and they would know that this help could only be properly given so long as they were frank and open in their relations, and the difficulties which each lay under properly explained to the rest. So that any appearance of secrecy or separateness in the actions of any of them would instantly, and justly, be looked upon with suspicion by the rest, as the sign of some selfish or foolish proceeding on the part of the individual. If, for instance, the scientific man were found to have gone out at night, unknown to the rest, to alter the sluices, the others would think, and in all probability rightly think, that he wanted to get the best supply of water to his own field; and if the shoemaker refused to show them where the bark grew which he made the sandals of, they would naturally think, and in all probability rightly think, that he didn't want them to see how much there was of it, and that he meant to ask from them more corn and potatoes in exchange for his sandals than the trouble of making them deserved. And thus, although each man would have a portion of time to himself in which he was allowed to do what he chose without let or inquiry - so long as he was working in that particular business which he had undertaken for the common benefit, any secrecy on his part would be immediately supposed to mean mischief; and would require to be accounted for, or put an end to: and this all the more because, whatever the work might be, certainly there would be difficulties about it which, when once they were well explained, might be more or less done away with by the help of the rest; so that assuredly every one of them would advance with his labour not only more happily, but more profitably and quickly, by having no secrets, and by frankly bestowing, and frankly receiving, such help as lay in his way to get or to give. 

  1. IPMAT 2020 Question Paper - IPM Indore Verbal

    When a dozen men are cast away on an imaginary island, the best educated would look for metals in rocks because

    1. metals can be used to make weapons.
    2. such an island probably has unexploited resources.
    3. he may find it beneath him to dig or cut or make shoes.
    4. he is suited for such work.
    Choice D
    he is suited for such work.

  2. IPMAT 2020 Question Paper - IPM Indore Verbal

    The author states that any appearance of secrecy or separateness would instantly and justly be looked upon with suspicion. From this statement we may infer that

    1. what is secret is not what is separate
    2. secrecy is not exactly the same as separateness
    3. it is natural to be suspicious of secrecy
    4. it only takes an instant for a relationship to deteriorate
    Choice D
    it only takes an instant for a relationship to deteriorate

  3. IPMAT 2020 Question Paper - IPM Indore Verbal

    The instance of the shoemaker who refuses to show his source and asks for more corn and potatoes, is an example of

    1. a strong bargain.
    2. unfair practice.
    3. the system of barter.
    4. the intent to make trouble.
    Choice B
    unfair practice.

  4. IPMAT 2020 Question Paper - IPM Indore Verbal

    According to the author, whatever one's work might be

    1. hardships are going to be part of it.
    2. one cannot keep complaining.
    3. one should expect others to assure of help and advance our labours.
    4. one must offer help to others in order to receive help.
    Choice D
    one must offer help to others in order to receive help.

  5. IPMAT 2020 Question Paper - IPM Indore Verbal

    The author's belief is that for progress to happen

    1. a team should consist of people with multiple talents.
    2. co-operation among team members is essential.
    3. one must deal with those who are secretive.
    4. transparency among all concerned is mandatory.
    Choice D
    transparency among all concerned is mandatory.

  6. IPMAT 2020 Question Paper - IPM Indore Verbal

    The writer makes a hypothesis, which can be related to

    1. communities in general.
    2. an imaginary island, rich with resources.
    3. an ideal world of talented people.
    4. a primitive and unsophisticated world.
    Choice A
    communities in general.

Passage 9 : IPMAT Reading Comprehension

Read the following passage and choose the answer that is closest to each of the questions that are based on the passage.

The perennial debate over gender differences threatens to remain inconclusive. Stereotypes pertaining to male superiority and female submissiveness could be traced to earlier ages where assigned roles were needed as survival measures. But, can we today see a swing away from these stereotypes, or have they established a stranglehold on our perceptions? In this gendered world, we continue to live with notions that one's gender determines one's skills and preferences, from toys and colours to career choices. So the girl child will be presented with a Barbie doll, while the boy child will receive a Lego set.

Does that mean that our brains are different? This myth has been exploded by a British professor of cognitive neuroimaging. Her research attempts to establish how these stereotypes mould our ideas of ourselves. She examines how science has been misinterpreted or misused to ask the wrong questions, instead of challenging the status quo. She urges us to move beyond a binary view of people's brains and instead to see these as highly individualised, profoundly adaptable, and full of unbounded potential. Her conclusive findings establish that no brain differences can be found that are solely gender related. In other words, modern neuroscientists have identified no decisive category-defining differences between the brains of men and women.

As a result of these findings we owe it to ourselves to dump the myths and look at ourselves afresh. We need to recognise that the male and female brain debate is a distraction, besides being based on inaccuracies. It is possibly harmful too, because it can be used as a hook to justify saying there is no point in girls doing science because they do not have a science brain; or compelling boys to opt for science because their brains are shaped for that subject. It can also condemn boys for being emotional, as this is seen as a feminine trait. And, most dangerous of all, to proclaim that boys, not girls, are meant to lead.

  1. IPMAT 2020 Question Paper - IPM Indore Verbal

    The research of a British professor of cognitive neuroimaging has succeeded in establishing that

    1. the brains of men and women are alike.
    2. science needs to challenge the status quo.
    3. society must break away from attempts at stereotyping gender issues.
    4. the potential of a human brain is not directly linked to gender.
    Choice D
    the potential of a human brain is not directly linked to gender.

  2. IPMAT 2020 Question Paper - IPM Indore Verbal

    By referring to the world as "gendered' the writer wants to convey that

    1. gender differences can be detected right from childhood.
    2. society continues to be fixated on gender stereotypes.
    3. one's gender is bound to determine one's abilities.
    4. the debate on gender differences will never be resolved.
    Choice B
    society continues to be fixated on gender stereotypes.

  3. IPMAT 2020 Question Paper - IPM Indore Verbal

    One of the dangers in adopting a binary view of the human brain is that it can

    1. promote the notion of feminine and masculine traits.
    2. determine as well as limit academic choices.
    3. lead to the distortion and misinterpretation of scientific data.
    4. be used to encourage male dominance and superiority.
    Choice A
    promote the notion of feminine and masculine traits.

  4. IPMAT 2020 Question Paper - IPM Indore Verbal

    The writer of this passage wants to emphasise the need to

    1. use new insights provided by scientific research for a better understanding of human abilities.
    2. continuously debate issues of gender differences to achieve human progress.
    3. question the findings of scientific inquiry into the functioning of the human brain.
    4. accept gender differences as essential to the survival of the human species.
    Choice A
    use new insights provided by scientific research for a better understanding of human abilities.

  5. IPMAT 2020 Question Paper - IPM Indore Verbal

    The synonym for 'stranglehold' (Para 1) is

    1. asphyxiation
    2. containment
    3. prohibiting entry
    4. overwhelming control
    Choice D
    overwhelming control

  6. IPMAT 2020 Question Paper - IPM Indore Verbal

    The antonym for "unbounded' (Para 2) is

    1. imprisoned
    2. aggressive
    3. restricted
    4. fearful
    Choice C
    restricted

The Questions that follow, are from actual CAT papers. If you wish to take them separately or plan to solve actual CAT papers at a later point in time, It would be a good idea to stop here.


CAT VARC : CAT 2020 Question Paper Slot 1

The passage below is accompanied by a set of questions. Choose the best answer to each question.

The word ‘anarchy’ comes from the Greek anarkhia, meaning contrary to authority or without a ruler, and was used in a derogatory sense until 1840, when it was adopted by Pierre-Joseph Proudhon to describe his political and social ideology. Proudhon argued that organization without government was both possible and desirable. In the evolution of political ideas, anarchism can be seen as an ultimate projection of both liberalism and socialism, and the differing strands of anarchist thought can be related to their emphasis on one or the other of these. 

Historically, anarchism arose not only as an explanation of the gulf between the rich and the poor in any community, and of the reason why the poor have been obliged to fight for their share of a common inheritance, but as a radical answer to the question ‘What went wrong?’ that followed the ultimate outcome of the French Revolution. It had ended not only with a reign of terror and the emergence of a newly rich ruling caste, but with a new adored emperor, Napoleon Bonaparte, strutting through his conquered territories.

The anarchists and their precursors were unique on the political Left in affirming that workers and peasants, grasping the chance that arose to bring an end to centuries of exploitation and tyranny, were inevitably betrayed by the new class of politicians, whose first priority was to re-establish a centralized state power. After every revolutionary uprising, usually won at a heavy cost for ordinary populations, the new rulers had no hesitation in applying violence and terror, a secret police, and a professional army to maintain their control.

For anarchists the state itself is the enemy, and they have applied the same interpretation to the outcome of every revolution of the 19th and 20th centuries. This is not merely because every state keeps a watchful and sometimes punitive eye on its dissidents, but because every state protects the privileges of the powerful.

The mainstream of anarchist propaganda for more than a century has been anarchist-communism, which argues that property in land, natural resources, and the means of production should be held in mutual control by local communities, federating for innumerable joint purposes with other communes. It differs from state socialism in opposing the concept of any central authority. Some anarchists prefer to distinguish between anarchist-communism and collectivist anarchism in order to stress the obviously desirable freedom of an individual or family to possess the resources needed for living, while not implying the right to own the resources needed by others. . . . 

There are, unsurprisingly, several traditions of individualist anarchism, one of them deriving from the ‘conscious egoism’ of the German writer Max Stirner (1806–56), and another from a remarkable series of 19th-century American figures who argued that in protecting our own autonomy and associating with others for common advantages, we are promoting the good of all. These thinkers differed from free-market liberals in their absolute mistrust of American capitalism, and in their emphasis on mutualism. 

  1. CAT 2020 Question Paper Slot 1

    Which one of the following best expresses the similarity between American individualist anarchists and free-market liberals as well as the difference between the former and the latter?

    1. Both reject the regulatory power of the state; but the former favour a people’s state, while the latter favour state intervention in markets.
    2. Both prioritise individual autonomy; but the former also emphasise mutual dependence, while the latter do not do so.
    3. Both are sophisticated arguments for capitalism; but the former argue for a morally upright capitalism, while the latter argue that the market is the only morality.
    4. Both are founded on the moral principles of altruism; but the latter conceive of the market as a force too mystical for the former to comprehend.
    Choice B
    Both prioritise individual autonomy; but the former also emphasise mutual dependence, while the latter do not do so.

  2. CAT 2020 Question Paper Slot 1

    The author makes all of the following arguments in the passage, EXCEPT:

    1. Individualist anarchism is actually constituted of many streams, all of which focus on the autonomy of the individual.
    2. The popular perception of anarchism as espousing lawlessness and violence comes from a mainstream mistrust of collectivism.
    3. For anarchists, the state is the enemy because all states apply violence and terror to maintain their control.
    4. The failure of the French Revolution was because of its betrayal by the new class of politicians who emerged from it.
    Choice B
    The popular perception of anarchism as espousing lawlessness and violence comes from a mainstream mistrust of collectivism.

  3. CAT 2020 Question Paper Slot 1

    According to the passage, what is the one idea that is common to all forms of anarchism?

    1. There is no idea common to all forms of anarchism; that is why it is anarchic.
    2. They all focus on the primacy of the power of the individual.
    3. They all derive from the work of Pierre-Joseph Proudhon.
    4. They are all opposed to the centralisation of power in the state.
    Choice D
    They are all opposed to the centralisation of power in the state.

  4. CAT 2020 Question Paper Slot 1

    The author believes that the new ruling class of politicians betrayed the principles of the French Revolution, but does not specify in what way. In the context of the passage, which statement below is the likeliest explanation of that betrayal?

    1. The new ruling class rode to power on the strength of the workers’ revolutionary anger, but then turned to oppress that very class.
    2. The anarchists did not want a new ruling class, but were not politically strong enough to stop them.
    3. The new ruling class was constituted mainly of anarchists who were against the destructive impact of the Revolution on the market.
    4. The new ruling class struck a deal with the old ruling class to share power between them.
    Choice A
    The new ruling class rode to power on the strength of the workers’ revolutionary anger, but then turned to oppress that very class.

  5. CAT 2020 Question Paper Slot 1

    Of the following sets of concepts, identify the set that is conceptually closest to the concerns of the passage.

    1. Anarchism, Betrayal, Power, State.
    2. Revolution, State, Strike, Egoism.
    3. Revolution, State, Protection, Liberals.
    4. Anarchism, State, Individual, Freedom.
    Choice D
    Anarchism, State, Individual, Freedom.

The passage below is accompanied by a set of questions. Choose the best answer to each question.

In the late 1960s, while studying the northern-elephant-seal population along the coasts of Mexico and California, Burney Le Boeuf and his colleagues couldn’t help but notice that the threat calls of males at some sites sounded different from those of males at other sites. . . . That was the first time dialects were documented in a nonhuman mammal. . . .

All the northern elephant seals that exist today are descendants of the small herd that survived on Isla Guadalupe [after the near extinction of the species in the nineteenth century]. As that tiny population grew, northern elephant seals started to recolonize former breeding locations. It was precisely on the more recently colonized islands where Le Boeuf found that the tempos of the male vocal displays showed stronger differences to the ones from Isla Guadalupe, the founder colony. 

In order to test the reliability of these dialects over time, Le Boeuf and other researchers visited Año Nuevo Island in California—the island where males showed the slowest pulse rates in their calls—every winter from 1968 to 1972. “What we found is that the pulse rate increased, but it still remained relatively slow compared to the other colonies we had measured in the past” Le Boeuf told me.

At the individual level, the pulse of the calls stayed the same: A male would maintain his vocal signature throughout his lifetime. But the average pulse rate was changing. Immigration could have been responsible for this increase, as in the early 1970s, 43 percent of the males on Año Nuevo had come from southern rookeries that had a faster pulse rate. This led Le Boeuf and his collaborator, Lewis Petrinovich, to deduce that the dialects were, perhaps, a result of isolation over time, after the breeding sites had been recolonized. For instance, the first settlers of Año Nuevo could have had, by chance, calls with low pulse rates. At other sites, where the scientists found faster pulse rates, the opposite would have happened—seals with faster rates would have happened to arrive first.

As the population continued to expand and the islands kept on receiving immigrants from the original population, the calls in all locations would have eventually regressed to the average pulse rate of the founder colony. In the decades that followed, scientists noticed that the geographical variations reported in 1969 were not obvious anymore. . . . In the early 2010s, while studying northern elephant seals on Año Nuevo Island, [researcher Caroline] Casey noticed, too, that what Le Boeuf had heard decades ago was not what she heard now. . . . By performing more sophisticated statistical analyses on both sets of data, [Casey and Le Boeuf] confirmed that dialects existed back then but had vanished. Yet there are other differences between the males from the late 1960s and their great-great-grandsons: Modern males exhibit more individual diversity, and their calls are more complex. While 50 years ago the drumming pattern was quite simple and the dialects denoted just a change in tempo, Casey explained, the calls recorded today have more complex structures, sometimes featuring doublets or triplets. . . .

  1. CAT 2020 Question Paper Slot 1

    Which one of the following conditions, if true, could have ensured that male northern elephant seal dialects did not disappear?

    1. Besides Isla Guadalupe, there was one more surviving colony with the same average male call tempo from which no migration took place.
    2. The call tempo of individual male seals in host colonies changed to match the average call tempo of immigrant male seals.
    3. Besides Isla Guadalupe, there was one more founder colony with the same average male call tempo from which male seals migrated to various other colonies.
    4. The call tempo of individual immigrant male seals changed to match the average tempo of resident male seals in the host colony.
    Choice D
    The call tempo of individual immigrant male seals changed to match the average tempo of resident male seals in the host colony.

  2. CAT 2020 Question Paper Slot 1

    All of the following can be inferred from Le Boeuf’s study as described in the passage EXCEPT that:

    1. changes in population and migration had no effect on the call pulse rate of individual male northern elephant seals.
    2. the influx of new northern elephant seals into Año Nuevo Island would have soon made the call pulse rate of its male seals exceed that of those at Isla Guadalupe.
    3. male northern elephant seals might not have exhibited dialects had they not become nearly extinct in the nineteenth century.
    4. the average call pulse rate of male northern elephant seals at Año Nuevo Island increased from the early 1970s till the disappearance of dialects.
    Choice B
    the influx of new northern elephant seals into Año Nuevo Island would have soon made the call pulse rate of its male seals exceed that of those at Isla Guadalupe.

  3. CAT 2020 Question Paper Slot 1

    Which one of the following best sums up the overall history of transformation of male northern elephant seal calls?

    1. Owing to migrations in the aftermath of near species extinction, the calls have transformed from exhibiting complex composition, less individual variety, and great regional variety to simple composition, less individual variety, and great regional variety.
    2. The calls have transformed from exhibiting simple composition, great individual variety, and less regional variety to complex composition, less individual variety, and great regional variety.
    3. Owing to migrations in the aftermath of near species extinction, the average call pulse rates in the recolonised breeding locations exhibited a gradual increase until they matched the tempo at the founding colony.
    4. The calls have transformed from exhibiting simple composition, less individual variety, and great regional variety to complex composition, great individual variety, and less regional variety.
    Choice D
    The calls have transformed from exhibiting simple composition, less individual variety, and great regional variety to complex composition, great individual variety, and less regional variety.

  4. CAT 2020 Question Paper Slot 1

    From the passage it can be inferred that the call pulse rate of male northern elephant seals in the southern rookeries was faster because:

    1. a large number of male northern elephant seals migrated from the southern rookeries to Año Nuevo Island in the early 1970s.
    2. the male northern elephant seals of Isla Guadalupe with faster call pulse rates might have been the original settlers of the southern rookeries.
    3. the calls of male northern elephant seals in the southern rookeries have more sophisticated structures, containing doublets and triplets.
    4. a large number of male northern elephant seals from Año Nuevo Island might have migrated to the southern rookeries to recolonise them.
    Choice B
    the male northern elephant seals of Isla Guadalupe with faster call pulse rates might have been the original settlers of the southern rookeries.

The passage below is accompanied by a set of questions. Choose the best answer to each question.

Few realise that the government of China, governing an empire of some 60 million people during the Tang dynasty (618–907), implemented a complex financial system that recognised grain, coins and textiles as money. . . . Coins did have certain advantages: they were durable, recognisable and provided a convenient medium of exchange, especially for smaller transactions. However, there were also disadvantages. A continuing shortage of copper meant that government mints could not produce enough coins for the entire empire, to the extent that for most of the dynasty’s history, coins constituted only a tenth of the money supply. One of the main objections to calls for taxes to be paid in coin was that peasant producers who could weave cloth or grow grain – the other two major currencies of the Tang – would not be able to produce coins, and therefore would not be able to pay their taxes. . . . 

As coins had advantages and disadvantages, so too did textiles. If in circulation for a long period of time, they could show signs of wear and tear. Stained, faded and torn bolts of textiles had less value than a brand new bolt. Furthermore, a full bolt had a particular value. If consumers cut textiles into smaller pieces to buy or sell something worth less than a full bolt, that, too, greatly lessened the value of the textiles. Unlike coins, textiles could not be used for small transactions; as [an official] noted, textiles could not “be exchanged by the foot and the inch” . . . 

But textiles had some advantages over coins. For a start, textile production was widespread and there were fewer problems with the supply of textiles. For large transactions, textiles weighed less than their equivalent in coins since a string of coins . . .  could weigh as much as 4 kg. Furthermore, the dimensions of a bolt of silk held remarkably steady from the third to the tenth century: 56 cm wide and 12 m long . . . The values of different textiles were also more stable than the fluctuating values of coins. . . .  

The government also required the use of textiles for large transactions. Coins, on the other hand, were better suited for smaller transactions, and possibly, given the costs of transporting coins, for a more local usage. Grain, because it rotted easily, was not used nearly as much as coins and textiles, but taxpayers were required to pay grain to the government as a share of their annual tax obligations, and official salaries were expressed in weights of grain. . . . 

In actuality, our own currency system today has some similarities even as it is changing in front of our eyes. . . . We have cash – coins for small transactions like paying for parking at a meter, and banknotes for other items; cheques and debit/credit cards for other, often larger, types of payments. At the same time, we are shifting to electronic banking and making payments online. Some young people never use cash [and] do not know how to write a cheque . . . 

  1. CAT 2020 Question Paper Slot 1

    In the context of the passage, which one of the following can be inferred with regard to the use of currency during the Tang era?

    1. Currency usage was similar to that of modern times.
    2. Currency that deteriorated easily was not used for official work.
    3. Copper coins were more valuable and durable than textiles.
    4. Grains were the most used currency because of government requirements.
    Choice A
    Currency usage was similar to that of modern times.

  2. CAT 2020 Question Paper Slot 1

    According to the passage, the modern currency system shares all the following features with that of the Tang, EXCEPT that:

    1. it uses different materials as currency.
    2. its currencies fluctuate in value over time. 
    3. it uses different currencies for different situations.
    4. it is undergoing transformation.
    Choice D
    it is undergoing transformation.

  3. CAT 2020 Question Paper Slot 1

    When discussing textiles as currency in the Tang period, the author uses the words “steady” and “stable” to indicate all of the following EXCEPT:

    1. reliable supply.
    2. reliable measurements.
    3. reliable quality.
    4. reliable transportation.
    Choice D
    reliable transportation.

  4. CAT 2020 Question Paper Slot 1

    During the Tang period, which one of the following would not be an economically sound decision for a small purchase in the local market that is worth one-eighth of a bolt of cloth?

    1. Cutting one-eighth of the fabric from a new bolt to pay the amount.
    2. Making the payment with the appropriate weight of grain.
    3. Using coins issued by the government to make the payment.
    4. Paying with a faded bolt of cloth that has approximately the same value.
    Choice A
    Cutting one-eighth of the fabric from a new bolt to pay the amount.

The passage below is accompanied by a set of questions. Choose the best answer to each question.

Vocabulary used in speech or writing organizes itself in seven parts of speech (eight, if you count interjections such as Oh! and Gosh! and Fuhgeddaboudit!). Communication composed of these parts of speech must be organized by rules of grammar upon which we agree. When these rules break down, confusion and misunderstanding result. Bad grammar produces bad sentences. My favorite example from Strunk and White is this one: “As a mother of five, with another one on the way, my ironing board is always up.”

Nouns and verbs are the two indispensable parts of writing. Without one of each, no group of words can be a sentence, since a sentence is, by definition, a group of words containing a subject (noun) and a predicate (verb); these strings of words begin with a capital letter, end with a period, and combine to make a complete thought which starts in the writer’s head and then leaps to the reader’s.

Must you write complete sentences each time, every time? Perish the thought. If your work consists only of fragments and floating clauses, the Grammar Police aren’t going to come and take you away. Even William Strunk, that Mussolini of rhetoric, recognized the delicious pliability of language. “It is an old observation,” he writes, “that the best writers sometimes disregard the rules of rhetoric.” Yet he goes on to add this thought, which I urge you to consider: “Unless he is certain of doing well, [the writer] will probably do best to follow the rules.”

The telling clause here is Unless he is certain of doing well. If you don’t have a rudimentary grasp of how the parts of speech translate into coherent sentences, how can you be certain that you are doing well? How will you know if you’re doing ill, for that matter? The answer, of course, is that you can’t, you won’t. One who does grasp the rudiments of grammar finds a comforting simplicity at its heart, where there need be only nouns, the words that name, and verbs, the words that act.

Take any noun, put it with any verb, and you have a sentence. It never fails. Rocks explode. Jane transmits. Mountains float. These are all perfect sentences. Many such thoughts make little rational sense, but even the stranger ones (Plums deify!) have a kind of poetic weight that’s nice. The simplicity of noun-verb construction is useful—at the very least it can provide a safety net for your writing. Strunk and White caution against too many simple sentences in a row, but simple sentences provide a path you can follow when you fear getting lost in the tangles of rhetoric—all those restrictive and nonrestrictive clauses, those modifying phrases, those appositives and compound-complex sentences. If you start to freak out at the sight of such unmapped territory (unmapped by you, at least), just remind yourself that rocks explode, Jane transmits, mountains float, and plums deify. Grammar is . . . the pole you grab to get your thoughts up on their feet and walking.

  1. CAT 2020 Question Paper Slot 1

    Which one of the following quotes best captures the main concern of the passage?

    1. “Nouns and verbs are the two indispensable parts of writing. Without one of each, no group of words can be a sentence . . .”
    2. “Strunk and White caution against too many simple sentences in a row, but simple sentences provide a path you can follow when you fear getting lost in the tangles of rhetoric . . .”
    3. “The telling clause here is Unless he is certain of doing well.”
    4. “Bad grammar produces bad sentences.”
    Choice D
    “Bad grammar produces bad sentences.”

  2. CAT 2020 Question Paper Slot 1

    Which one of the following statements, if false, could be seen as supporting the arguments in the passage?

    1. An understanding of grammar helps a writer decide if she/he is writing well or not.
    2. Perish the thought that complete sentences necessarily need nouns and verbs!
    3. It has been observed that writers sometimes disregard the rules of rhetoric.
    4. Regarding grammar, women writers tend to be more attentive to method and accuracy.
    Choice B
    Perish the thought that complete sentences necessarily need nouns and verbs!

  3. CAT 2020 Question Paper Slot 1

    All of the following statements can be inferred from the passage EXCEPT that:

    1. the subject–predicate relation is the same as the noun–verb relation.
    2. the primary purpose of grammar is to ensure that sentences remain simple.
    3. sentences do not always have to be complete.
    4. “Grammar Police” is a metaphor for critics who focus on linguistic rules.
    Choice B
    the primary purpose of grammar is to ensure that sentences remain simple.

  4. CAT 2020 Question Paper Slot 1

    “Take any noun, put it with any verb, and you have a sentence. It never fails. Rocks explode. Jane transmits. Mountains float.” None of the following statements can be seen as similar EXCEPT:

    1. A collection of people with the same sports equipment is a sports team.
    2. Take an apple tree, plant it in a field, and you have an orchard.
    3. A group of nouns arranged in a row becomes a sentence.
    4. Take any vegetable, put some spices in it, and you have a dish.
    Choice D
    Take any vegetable, put some spices in it, and you have a dish.

  5. CAT 2020 Question Paper Slot 1

    Inferring from the passage, the author could be most supportive of which one of the following practices?

    1. A Creative Writing course that focuses on how to avoid the use of rhetoric.
    2. The critique of standardised rules of punctuation and capitalisation.
    3. A campaign demanding that a writer’s creative license should allow the breaking of grammatical rules.
    4. The availability of language software that will standardise the rules of grammar as an aid to writers.
    Choice D
    The availability of language software that will standardise the rules of grammar as an aid to writers.

CAT VARC : CAT 2020 Question Paper Slot 2

The passage below is accompanied by a set of questions. Choose the best answer to each question.

The claims advanced here may be condensed into two assertions: [first, that visual] culture is what images, acts of seeing, and attendant intellectual, emotional, and perceptual sensibilities do to build, maintain, or transform the worlds in which people live. [And second, that the] study of visual culture is the analysis and interpretation of images and the ways of seeing (or gazes) that configure the agents, practices, conceptualities, and institutions that put images to work. . . .

Accordingly, the study of visual culture should be characterized by several concerns. First, scholars of visual culture need to examine any and all imagery – high and low, art and nonart. . . . They must not restrict themselves to objects of a particular beauty or aesthetic value. Indeed, any kind of imagery may be found to offer up evidence of the visual construction of reality. . . . 

Second, the study of visual culture must scrutinize visual practice as much as images themselves, asking what images do when they are put to use. If scholars engaged in this enterprise inquire what makes an image beautiful or why this image or that constitutes a masterpiece or a work of genius, they should do so with the purpose of investigating an artist’s or a work’s contribution to the experience of beauty, taste, value, or genius. No amount of social analysis can account fully for the existence of Michelangelo or Leonardo. They were unique creators of images that changed the way their contemporaries thought and felt and have continued to shape the history of art, artists, museums, feeling, and aesthetic value. But study of the critical, artistic, and popular reception of works by such artists as Michelangelo and Leonardo can shed important light on the meaning of these artists and their works for many different people. And the history of meaning-making has a great deal to do with how scholars as well as lay audiences today understand these artists and their achievements. 

Third, scholars studying visual culture might properly focus their interpretative work on lifeworlds by examining images, practices, visual technologies, taste, and artistic style as constitutive of social relations. The task is to understand how artifacts contribute to the construction of a world. . . . Important methodological implications follow: ethnography and reception studies become productive forms of gathering information, since these move beyond the image as a closed and fixed meaning-event. . . .

Fourth, scholars may learn a great deal when they scrutinize the constituents of vision, that is, the structures of perception as a physiological process as well as the epistemological frameworks informing a system of visual representation. Vision is a socially and a biologically constructed operation, depending on the design of the human body and how it engages the interpretive devices developed by a culture in order to see intelligibly. . . . Seeing . . . operates on the foundation of covenants with images that establish the conditions for meaningful visual experience. 

Finally, the scholar of visual culture seeks to regard images as evidence for explanation, not as epiphenomena.  

  1. CAT 2020 Question Paper Slot 2

    Which one of the following best describes the word “epiphenomena” in the last sentence of the passage?

    1. Phenomena amenable to analysis.
    2. Phenomena supplemental to the evidence.
    3. Overarching collections of images.
    4. Visual phenomena of epic proportions.
    Choice B
    Phenomena supplemental to the evidence.

  2. CAT 2020 Question Paper Slot 2

    All of the following statements may be considered valid inferences from the passage, EXCEPT:

    1. artifacts are meaningful precisely because they help to construct the meanings of the world for us.
    2. studying visual culture requires institutional structures without which the structures of perception cannot be analysed.
    3. understanding the structures of perception is an important part of understanding how visual cultures work.
    4. visual culture is not just about how we see, but also about how our visual practices can impact and change the world.
    Choice B
    studying visual culture requires institutional structures without which the structures of perception cannot be analysed.

  3. CAT 2020 Question Paper Slot 2

    “No amount of social analysis can account fully for the existence of Michelangelo or Leonardo.” In light of the passage, which one of the following interpretations of this sentence is the most accurate?

    1. Social analytical accounts of people like Michelangelo or Leonardo cannot explain their genius.
    2. Michelangelo or Leonardo cannot be subjected to social analysis because of their genius.
    3. No analyses exist of Michelangelo’s or Leonardo’s social accounts.
    4. Socially existing beings cannot be analysed, unlike the art of Michelangelo or Leonardo which can.
    Choice A
    Social analytical accounts of people like Michelangelo or Leonardo cannot explain their genius.

  4. CAT 2020 Question Paper Slot 2

    “Seeing . . . operates on the foundation of covenants with images that establish the conditions for meaningful visual experience.” In light of the passage, which one of the following statements best conveys the meaning of this sentence?

    1. Sight becomes a meaningful visual experience because of covenants of meaningfulness that we establish with the images we see.
    2. The way we experience sight is through images operated on by meaningful covenants.
    3. Images are meaningful visual experiences when they have a foundation of covenants seeing them.
    4. Sight as a meaningful visual experience is possible when there is a foundational condition established in images of covenants.
    Choice A
    Sight becomes a meaningful visual experience because of covenants of meaningfulness that we establish with the images we see.

  5. CAT 2020 Question Paper Slot 2

    Which set of keywords below most closely captures the arguments of the passage?

    1. Visual Culture, Aesthetic Value, Lay Audience, Visual Experience.
    2. Scholars, Social Analysis, Michelangelo and Leonardo, Interpretive Devices.
    3. Imagery, Visual Practices, Lifeworlds, Structures of Perception.
    4. Visual Construction of Reality, Work of Genius, Ethnography, Epiphenomena.
    Choice C
    Imagery, Visual Practices, Lifeworlds, Structures of Perception.

The passage below is accompanied by a set of questions. Choose the best answer to each question.

174 incidents of piracy were reported to the International Maritime Bureau last year, with Somali pirates responsible for only three. The rest ranged from the discreet theft of coils of rope in the Yellow Sea to the notoriously ferocious Nigerian gunmen attacking and hijacking oil tankers in the Gulf of Guinea, as well as armed robbery off Singapore and the Venezuelan coast and kidnapping in the Sundarbans in the Bay of Bengal. For [Dr. Peter] Lehr, an expert on modern-day piracy, the phenomenon’s history should be a source of instruction rather than entertainment, piracy past offering lessons for piracy present. . . . 

But . . . where does piracy begin or end? According to St Augustine, a corsair captain once told Alexander the Great that in the forceful acquisition of power and wealth at sea, the difference between an emperor and a pirate was simply one of scale. By this logic, European empire-builders were the most successful pirates of all time. A more eclectic history might have included the conquistadors, Vasco da Gama and the East India Company. But Lehr sticks to the disorganised small fry, making comparisons with the renegades of today possible. 

The main motive for piracy has always been a combination of need and greed. Why toil away as a starving peasant in the 16th century when a successful pirate made up to £4,000 on each raid? Anyone could turn to freebooting if the rewards were worth the risk . . . .

Increased globalisation has done more to encourage piracy than suppress it. European colonialism weakened delicate balances of power, leading to an influx of opportunists on the high seas. A rise in global shipping has meant rich pickings for freebooters. Lehr writes: “It quickly becomes clear that in those parts of the world that have not profited from globalisation and modernisation, and where abject poverty and the daily struggle for survival are still a reality, the root causes of piracy are still the same as they were a couple of hundred years ago.” . . . 

Modern pirate prevention has failed. After the French yacht Le Gonant was ransomed for $2 million in 2008, opportunists from all over Somalia flocked to the coast for a piece of the action. . . . A consistent rule, even today, is there are never enough warships to patrol pirate-infested waters. Such ships are costly and only solve the problem temporarily; Somali piracy is bound to return as soon as the warships are withdrawn. Robot shipping, eliminating hostages, has been proposed as a possible solution; but as Lehr points out, this will only make pirates switch their targets to smaller carriers unable to afford the technology.

His advice isn’t new. Proposals to end illegal fishing are often advanced but they are difficult to enforce. Investment in local welfare put a halt to Malaysian piracy in the 1970s, but was dependent on money somehow filtering through a corrupt bureaucracy to the poor on the periphery. Diplomatic initiatives against piracy are plagued by mutual distrust: the Russians execute pirates, while the EU and US are reluctant to capture them for fear they’ll claim asylum. 

  1. CAT 2020 Question Paper Slot 2

    “A more eclectic history might have included the conquistadors, Vasco da Gama and the East India Company. But Lehr sticks to the disorganised small fry . . .” From this statement we can infer that the author believes that:

    1. colonialism should be considered an organised form of piracy.
    2. the disorganised piracy of today is no match for the organised piracy of the past.
    3. Lehr does not assign adequate blame to empire builders for their past deeds.
    4. Vasco da Gama and the East India Company laid the ground for modern piracy.   
    Choice A
    colonialism should be considered an organised form of piracy.

  2. CAT 2020 Question Paper Slot 2

    We can deduce that the author believes that piracy can best be controlled in the long run:

    1. through international cooperation in enforcing stringent deterrents.
    2. if we eliminate poverty and income disparities in affected regions.
    3. through the extensive deployment of technology to track ships and cargo. 
    4. through lucrative welfare schemes to improve the lives of people in affected regions.
    Choice B
    if we eliminate poverty and income disparities in affected regions.

  3. CAT 2020 Question Paper Slot 2

    “Why toil away as a starving peasant in the 16th century when a successful pirate made up to £4,000 on each raid?” In this sentence, the author’s tone can best be described as being:

    1. facetious, about the hardships of peasant life in medieval England.
    2. ironic, about the reasons why so many took to piracy in medieval times.
    3. analytical, to explain the contrasts between peasant and pirate life in medieval England.
    4. indignant, at the scale of wealth successful pirates could amass in medieval times. 
    Choice B
    ironic, about the reasons why so many took to piracy in medieval times.

  4. CAT 2020 Question Paper Slot 2

    The author ascribes the rise in piracy today to all of the following factors EXCEPT:

    1. the growth in international shipping with globalisation.
    2. colonialism’s disruption of historic ties among countries. 
    3. the high rewards via ransoms for successful piracy attempts. 
    4. decreased surveillance of the high seas.
    Choice D
    decreased surveillance of the high seas.

The passage below is accompanied by a set of questions. Choose the best answer to each question.

In a low-carbon world, renewable energy technologies are hot business. For investors looking to redirect funds, wind turbines and solar panels, among other technologies, seem a straightforward choice. But renewables need to be further scrutinized before being championed as forging a path toward a low-carbon future. Both the direct and indirect impacts of renewable energy must be examined to ensure that a climate-smart future does not intensify social and environmental harm. As renewable energy production requires land, water, and labor, among other inputs, it imposes costs on people and the environment. Hydropower projects, for instance, have led to community dispossession and exclusion . . . Renewable energy supply chains are also intertwined with mining, and their technologies contribute to growing levels of electronic waste . . . Furthermore, although renewable energy can be produced and distributed through small-scale, local systems, such an approach might not generate the high returns on investment needed to attract capital.

Although an emerging sector, renewables are enmeshed in long-standing resource extraction through their dependence on minerals and metals . . . Scholars document the negative consequences of mining . . . even for mining operations that commit to socially responsible practices[:] “many of the world’s largest reservoirs of minerals like cobalt, copper, lithium, [and] rare earth minerals”—the ones needed for renewable technologies—“are found in fragile states and under communities of marginalized peoples in Africa, Asia, and Latin America.” Since the demand for metals and minerals will increase substantially in a renewable-powered future . . . this intensification could exacerbate the existing consequences of extractive activities.

Among the connections between climate change and waste, O’Neill . . . highlights that “devices developed to reduce our carbon footprint, such as lithium batteries for hybrid and electric cars or solar panels[,] become potentially dangerous electronic waste at the end of their productive life.” The disposal of toxic waste has long perpetuated social injustice through the flows of waste to the Global South and to marginalized communities in the Global North . . .

While renewable energy is a more recent addition to financial portfolios, investments in the sector must be considered in light of our understanding of capital accumulation. As agricultural finance reveals, the concentration of control of corporate activity facilitates profit generation. For some climate activists, the promise of renewables rests on their ability not only to reduce emissions but also to provide distributed, democratized access to energy . . . But Burke and Stephens . . . caution that “renewable energy systems offer a possibility but not a certainty for more democratic energy futures.” Small-scale, distributed forms of energy are only highly profitable to institutional investors if control is consolidated somewhere in the financial chain. Renewable energy can be produced at the household or neighborhood level. However, such small-scale, localized production is unlikely to generate high returns for investors. For financial growth to be sustained and expanded by the renewable sector, production and trade in renewable energy technologies will need to be highly concentrated, and large asset management firms will likely drive those developments.

  1. CAT 2020 Question Paper Slot 2

    All of the following statements, if true, could be seen as supporting the arguments in the passage, EXCEPT:

    1. Marginalised people in Africa, Asia and Latin America have often been the main sufferers of corporate mineral extraction projects.
    2. The example of agricultural finance helps us to see how to concentrate corporate activity in the renewable energy sector.
    3. One reason for the perpetuation of social injustice lies in the problem of the disposal of toxic waste.
    4. The possible negative impacts of renewable energy need to be studied before it can be offered as a financial investment opportunity.
    Choice D
    The possible negative impacts of renewable energy need to be studied before it can be offered as a financial investment opportunity.

  2. CAT 2020 Question Paper Slot 2

    Which one of the following statements, if false, could be seen as best supporting the arguments in the passage?

    1. Renewable energy systems are not as profitable as non-renewable energy systems.
    2. Renewable energy systems are as expensive as non-renewable energy systems.
    3. The production and distribution of renewable energy through small-scale, local systems is not economically sustainable.
    4. Renewable energy systems have little or no environmental impact.
    Choice D
    Renewable energy systems have little or no environmental impact.

  3. CAT 2020 Question Paper Slot 2

    Which one of the following statements, if true, could be an accurate inference from the first paragraph of the passage?

    1. The author’s only reservation is about the profitability of renewable energy systems.
    2. The author does not think renewable energy systems can be as efficient as non-renewable energy systems.
    3. The author has reservations about the consequences of non-renewable energy systems.
    4. The author has reservations about the consequences of renewable energy systems.
    Choice D
    The author has reservations about the consequences of renewable energy systems.

  4. CAT 2020 Question Paper Slot 2

    Which one of the following statements best captures the main argument of the last paragraph of the passage?

    1. Renewable energy systems are not democratic unless they are corporate-controlled.
    2. Renewable energy produced at the household or neighbourhood level is more efficient than mass-produced forms of energy.
    3. The development of the renewable energy sector is a double-edged sword.
    4. Most forms of renewable energy are not profitable investments for institutional investors.
    Choice C
    The development of the renewable energy sector is a double-edged sword.

  5. CAT 2020 Question Paper Slot 2

    Based on the passage, we can infer that the author would be most supportive of which one of the following practices?

    1. The study of the coexistence of marginalised people with their environments.
    2. Encouragement for the development of more environment-friendly carbon-based fuels.
    3. More stringent global policies and regulations to ensure a more just system of toxic waste disposal.
    4. The localised, small-scale development of renewable energy systems.
    Choice C
    More stringent global policies and regulations to ensure a more just system of toxic waste disposal.

The passage below is accompanied by a set of questions. Choose the best answer to each question.

Aggression is any behavior that is directed toward injuring, harming, or inflicting pain on another living being or group of beings. Generally, the victim(s) of aggression must wish to avoid such behavior in order for it to be considered true aggression. Aggression is also categorized according to its ultimate intent. Hostile aggression is an aggressive act that results from anger, and is intended to inflict pain or injury because of that anger. Instrumental aggression is an aggressive act that is regarded as a means to an end other than pain or injury. For example, an enemy combatant may be subjected to torture in order to extract useful intelligence, though those inflicting the torture may have no real feelings of anger or animosity toward their subject. The concept of aggression is very broad, and includes many categories of behavior (e.g., verbal aggression, street crime, child abuse, spouse abuse, group conflict, war, etc.). A number of theories and models of aggression have arisen to explain these diverse forms of behavior, and these theories/models tend to be categorized according to their specific focus. The most common system of categorization groups the various approaches to aggression into three separate areas, based upon the three key variables that are present whenever any aggressive act or set of acts is committed. The first variable is the aggressor him/herself. The second is the social situation or circumstance in which the aggressive act(s) occur. The third variable is the target or victim of aggression.

Regarding theories and research on the aggressor, the fundamental focus is on the factors that lead an individual (or group) to commit aggressive acts. At the most basic level, some argue that aggressive urges and actions are the result of inborn, biological factors. Sigmund Freud (1930) proposed that all individuals are born with a death instinct that predisposes us to a variety of aggressive behaviors, including suicide (self directed aggression) and mental illness (possibly due to an unhealthy or unnatural suppression of aggressive urges). Other influential perspectives supporting a biological basis for aggression conclude that humans evolved with an abnormally low neural inhibition of aggressive impulses (in comparison to other species), and that humans possess a powerful instinct for property accumulation and territorialism. It is proposed that this instinct accounts for hostile behaviors ranging from minor street crime to world wars. Hormonal factors also appear to play a significant role in fostering aggressive tendencies. For example, the hormone testosterone has been shown to increase aggressive behaviors when injected into animals. Men and women convicted of violent crimes also possess significantly higher levels of testosterone than men and women convicted of non violent crimes. Numerous studies comparing different age groups, racial/ethnic groups, and cultures also indicate that men, overall, are more likely to engage in a variety of aggressive behaviors (e.g., sexual assault, aggravated assault, etc.) than women. One explanation for higher levels of aggression in men is based on the assumption that, on average, men have higher levels of testosterone than women.

  1. CAT 2020 Question Paper Slot 2

    “[A]n enemy combatant may be subjected to torture in order to extract useful intelligence, though those inflicting the torture may have no real feelings of anger or animosity toward their subject.” Which one of the following best explicates the larger point being made by the author here?

    1. In certain kinds of aggression, inflicting pain is not the objective, and is no more than a utilitarian means to achieve another end.
    2. When an enemy combatant refuses to reveal information, the use of torture can sometimes involve real feelings of hostility.
    3. Information revealed by subjecting an enemy combatant to torture is not always reliable because of the animosity involved.
    4. The use of torture to extract information is most effective when the torturer is not emotionally involved in the torture.
    Choice A
    In certain kinds of aggression, inflicting pain is not the objective, and is no more than a utilitarian means to achieve another end.

  2. CAT 2020 Question Paper Slot 2

    All of the following statements can be seen as logically implied by the arguments of the passage EXCEPT:

    1. Freud’s theory of aggression proposes that aggression results from the suppression of aggressive urges.
    2. the Freudian theory of suicide as self-inflicted aggression implies that an aggressive act need not be sought to be avoided in order for it to be considered aggression.
    3. a common theory of aggression is that it is the result of an abnormally low neural regulation of testosterone.
    4. if the alleged aggressive act is not sought to be avoided, it cannot really be considered aggression.
    Choice C
    a common theory of aggression is that it is the result of an abnormally low neural regulation of testosterone.

  3. CAT 2020 Question Paper Slot 2

    The author identifies three essential factors according to which theories of aggression are most commonly categorised. Which of the following options is closest to the factors identified by the author?

    1. Extreme – Moderate – Mild.
    2. Hostile – Instrumental – Hormonal.
    3. Aggressor – Circumstances of aggression – Victim.
    4. Psychologically – Sociologically – Medically.
    Choice C
    Aggressor – Circumstances of aggression – Victim.

  4. CAT 2020 Question Paper Slot 2

    The author discusses all of the following arguments in the passage EXCEPT that:

    1. aggression in most societies is kept under control through moderating the death instinct identified by Freud.
    2. the nature of aggression can vary depending on several factors, including intent.
    3. men in general are believed to be more hormonally driven to exhibit violence than women.
    4. several studies indicate that aggression may have roots in the biological condition of humanity.
    Choice A
    aggression in most societies is kept under control through moderating the death instinct identified by Freud.

CAT VARC : CAT 2020 Question Paper Slot 3

The passage below is accompanied by a set of questions. Choose the best answer to each question.

Mode of transportation affects the travel experience and thus can produce new types of travel writing and perhaps even new “identities.” Modes of transportation determine the types and duration of social encounters; affect the organization and passage of space and time; . . . and also affect perception and knowledge—how and what the traveler comes to know and write about. The completion of the first U.S. transcontinental highway during the 1920s . . . for example, inaugurated a new genre of travel literature about the United States—the automotive or road narrative. Such narratives highlight the experiences of mostly male protagonists “discovering themselves” on their journeys, emphasizing the independence of road travel and the value of rural folk traditions.

Travel writing’s relationship to empire building— as a type of “colonialist discourse”—has drawn the most attention from academicians. Close connections have been observed between European (and American) political, economic, and administrative goals for the colonies and their manifestations in the cultural practice of writing travel books. Travel writers’ descriptions of foreign places have been analyzed as attempts to validate, promote, or challenge the ideologies and practices of colonial or imperial domination and expansion. Mary Louise Pratt’s study of the genres and conventions of 18th- and 19th-century exploration narratives about South America and Africa (e.g., the “monarch of all I survey” trope) offered ways of thinking about travel writing as embedded within relations of power between metropole and periphery, as did Edward Said’s theories of representation and cultural imperialism. Particularly Said’s book, Orientalism, helped scholars understand ways in which representations of people in travel texts were intimately bound up with notions of self, in this case, that the Occident defined itself through essentialist, ethnocentric, and racist representations of the Orient. Said’s work became a model for demonstrating cultural forms of imperialism in travel texts, showing how the political, economic, or administrative fact of dominance relies on legitimating discourses such as those articulated through travel writing. . . .

Feminist geographers’ studies of travel writing challenge the masculinist history of geography by questioning who and what are relevant subjects of geographic study and, indeed, what counts as geographic knowledge itself. Such questions are worked through ideological constructs that posit men as explorers and women as travelers—or, conversely, men as travelers and women as tied to the home. Studies of Victorian women who were professional travel writers, tourists, wives of colonial administrators, and other (mostly) elite women who wrote narratives about their experiences abroad during the 19th century have been particularly revealing. From a “liberal” feminist perspective, travel presented one means toward female liberation for middle- and upper-class Victorian women. Many studies from the 1970s onward demonstrated the ways in which women’s gendered identities were negotiated differently “at home” than they were “away,” thereby showing women’s self-development through travel. The more recent poststructural turn in studies of Victorian travel writing has focused attention on women’s diverse and fragmented identities as they narrated their travel experiences, emphasizing women’s sense of themselves as women in new locations, but only as they worked through their ties to nation, class, whiteness, and colonial and imperial power structures.

  1. CAT 2020 Question Paper Slot 3

    From the passage, we can infer that feminist scholars’ understanding of the experiences of Victorian women travellers is influenced by all of the following EXCEPT scholars':

    1. awareness of the ways in which identity is formed.
    2. perspective that they bring to their research.
    3. knowledge of class tensions in Victorian society.
    4. awareness of gender issues in Victorian society.
    Choice C
    knowledge of class tensions in Victorian society.

  2. CAT 2020 Question Paper Slot 3

    From the passage, we can infer that travel writing is most similar to:

    1. political journalism.
    2. historical fiction.
    3. autobiographical writing.
    4. feminist writing.
    Choice C
    autobiographical writing.

  3. CAT 2020 Question Paper Slot 3

    From the passage, it can be inferred that scholars argue that Victorian women experienced self-development through their travels because:

    1. their identity was redefined when they were away from home.
    2. they were from the progressive middle- and upper-classes of society.
    3. they were on a quest to discover their diverse identities.
    4. they developed a feminist perspective of the world.
    Choice A
    their identity was redefined when they were away from home.

  4. CAT 2020 Question Paper Slot 3

    American travel literature of the 1920s:

    1. celebrated the freedom that travel gives.
    2. developed the male protagonists’ desire for independence.
    3. presented travellers’ discovery of their identity as different from others.
    4. showed participation in local traditions.
    Choice A
    celebrated the freedom that travel gives.

  5. CAT 2020 Question Paper Slot 3

    According to the passage, Said’s book, “Orientalism”:

    1. explained the difference between the representation of people and the actual fact.
    2. demonstrated how cultural imperialism was used to justify colonial domination.
    3. argued that cultural imperialism was more significant than colonial domination.
    4. illustrated how narrow minded and racist westerners were.
    Choice B
    demonstrated how cultural imperialism was used to justify colonial domination.

The passage below is accompanied by a set of questions. Choose the best answer to each question.

Although one of the most contested concepts in political philosophy, human nature is something on which most people seem to agree. By and large, according to Rutger Bregman in his new book Humankind, we have a rather pessimistic view – not of ourselves exactly, but of everyone else. We see other people as selfish, untrustworthy and dangerous and therefore we behave towards them with defensiveness and suspicion. This was how the 17th-century philosopher Thomas Hobbes conceived our natural state to be, believing that all that stood between us and violent anarchy was a strong state and firm leadership.

But in following Hobbes, argues Bregman, we ensure that the negative view we have of human nature is reflected back at us. He instead puts his faith in Jean-Jacques Rousseau, the 18th-century French thinker, who famously declared that man was born free and it was civilisation – with its coercive powers, social classes and restrictive laws – that put him in chains.

Hobbes and Rousseau are seen as the two poles of the human nature argument and it’s no surprise that Bregman strongly sides with the Frenchman. He takes Rousseau’s intuition and paints a picture of a prelapsarian idyll in which, for the better part of 300,000 years, Homo sapiens lived a fulfilling life in harmony with nature . . . Then we discovered agriculture and for the next 10,000 years it was all property, war, greed and injustice. . . . 

It was abandoning our nomadic lifestyle and then domesticating animals, says Bregman, that brought about infectious diseases such as measles, smallpox, tuberculosis, syphilis, malaria, cholera and plague. This may be true, but what Bregman never really seems to get to grips with is that pathogens were not the only things that grew with agriculture – so did the number of humans. It’s one thing to maintain friendly relations and a property-less mode of living when you’re 30 or 40 hunter-gatherers following the food. But life becomes a great deal more complex and knowledge far more extensive when there are settlements of many thousands. 

“Civilisation has become synonymous with peace and progress and wilderness with war and decline,” writes Bregman. “In reality, for most of human existence, it was the other way around.” Whereas traditional history depicts the collapse of civilisations as “dark ages” in which everything gets worse, modern scholars, he claims, see them more as a reprieve, in which the enslaved gain their freedom and culture flourishes. Like much else in this book, the truth is probably somewhere between the two stated positions.

In any case, the fear of civilisational collapse, Bregman believes, is unfounded. It’s the result of what the Dutch biologist Frans de Waal calls “veneer theory” – the idea that just below the surface, our bestial nature is waiting to break out. . . . There’s a great deal of reassuring human decency to be taken from this bold and thought-provoking book and a wealth of evidence in support of the contention that the sense of who we are as a species has been deleteriously distorted. But it seems equally misleading to offer the false choice of Rousseau and Hobbes when, clearly, humanity encompasses both.

  1. CAT 2020 Question Paper Slot 3

    The author has differing views from Bregman regarding:

    1. the role of pathogens in the spread of infectious diseases.
    2. a civilised society being coercive and unjust.
    3. a property-less mode of living being socially harmonious.
    4. the role of agriculture in the advancement of knowledge.
    Choice B
    a civilised society being coercive and unjust.

  2. CAT 2020 Question Paper Slot 3

    According to the passage, the “collapse of civilisations” is viewed by Bregman as:

    1. a sign of regression in society’s trajectory. 
    2. resulting from a breakdown in the veneer of human nature.  
    3. a time that enables changes in societies and cultures.
    4. a temporary phase which can be rectified by social action.
    Choice C
    a time that enables changes in societies and cultures.

  3. CAT 2020 Question Paper Slot 3

    None of the following views is expressed in the passage EXCEPT that:

    1. Hobbes and Rousseau disagreed on the fundamental nature of humans, but both believed in the need for a strong state. 
    2. Bregman agrees with Hobbes that firm leadership is needed to ensure property rights and regulate strife.
    3. the author of the review believes in the veneer theory of human nature.   
    4. most people agree with Hobbes’ pessimistic view of human nature as being intrinsically untrustworthy and selfish.   
    Choice D
    most people agree with Hobbes’ pessimistic view of human nature as being intrinsically untrustworthy and selfish.

  4. CAT 2020 Question Paper Slot 3

    According to the author, the main reason why Bregman contrasts life in pre-agricultural societies with agricultural societies is to:

    1. advocate the promotion of less complex societies as a basis for greater security and prosperity.
    2. bolster his argument that people are basically decent, but progress as we know it can make them selfish.
    3. highlight the enormous impact that settled farming had on population growth.
    4. make the argument that an environmentally conscious lifestyle is a more harmonious way of living.
    Choice C
    bolster his argument that people are basically decent, but progress as we know it can make them selfish.

The passage below is accompanied by a set of questions. Choose the best answer to each question.

[There is] a curious new reality: Human contact is becoming a luxury good. As more screens appear in the lives of the poor, screens are disappearing from the lives of the rich. The richer you are, the more you spend to be off-screen. . . .  

The joy — at least at first — of the internet revolution was its democratic nature. Facebook is the same Facebook whether you are rich or poor. Gmail is the same Gmail. And it’s all free. There is something mass market and unappealing about that. And as studies show that time on these advertisement-support platforms is unhealthy, it all starts to seem déclassé, like drinking soda or smoking cigarettes, which wealthy people do less than poor people.  The wealthy can afford to opt out of having their data and their attention sold as a product. The poor and middle class don’t have the same kind of resources to make that happen.

Screen exposure starts young. And children who spent more than two hours a day looking at a screen got lower scores on thinking and language tests, according to early results of a landmark study on brain development of more than 11,000 children that the National Institutes of Health is supporting. Most disturbingly, the study is finding that the brains of children who spend a lot of time on screens are different. For some kids, there is premature thinning of their cerebral cortex. In adults, one study found an association between screen time and depression. . . .

Tech companies worked hard to get public schools to buy into programs that required schools to have one laptop per student, arguing that it would better prepare children for their screen-based future. But this idea isn’t how the people who actually build the screen-based future raise their own children. In Silicon Valley, time on screens is increasingly seen as unhealthy. Here, the popular elementary school is the local Waldorf School, which promises a back-to-nature, nearly screen-free education. So as wealthy kids are growing up with less screen time, poor kids are growing up with more. How comfortable someone is with human engagement could become a new class marker.

Human contact is, of course, not exactly like organic food . . . . But with screen time, there has been a concerted effort on the part of Silicon Valley behemoths to confuse the public. The poor and the middle class are told that screens are good and important for them and their children. There are fleets of psychologists and neuroscientists on staff at big tech companies working to hook eyes and minds to the screen as fast as possible and for as long as possible. And so human contact is rare. . . . 

There is a small movement to pass a “right to disconnect” bill, which would allow workers to turn their phones off, but for now a worker can be punished for going offline and not being available. There is also the reality that in our culture of increasing isolation, in which so many of the traditional gathering places and social structures have disappeared, screens are filling a crucial void.

  1. CAT 2020 Question Paper Slot 3

    Which of the following statements about the negative effects of screen time is the author least likely to endorse?

    1. It can cause depression in viewers.
    2. It increases human contact as it fills an isolation void.
    3. It is shown to have adverse effects on young children’s learning.
    4. It is designed to be addictive.
    Choice B
    It increases human contact as it fills an isolation void.

  2. CAT 2020 Question Paper Slot 3

    The statement “The richer you are, the more you spend to be off-screen” is supported by which other line from the passage?

    1. “. . . studies show that time on these advertisement-support platforms is unhealthy . . .”
    2. “Gmail is the same Gmail. And it’s all free.” 
    3. “How comfortable someone is with human engagement could become a new class marker.”
    4.  “. . . screens are filling a crucial void.”
    Choice C
    “How comfortable someone is with human engagement could become a new class marker.”

  3. CAT 2020 Question Paper Slot 3

    The author is least likely to agree with the view that the increase in screen-time is fuelled by the fact that:

    1. screens provide social contact in an increasingly isolating world.
    2. some workers face punitive action if they are not online.
    3. with falling costs, people are streaming more content on their devices.
    4. there is a growth in computer-based teaching in public schools.
    Choice C
    with falling costs, people are streaming more content on their devices.

  4. CAT 2020 Question Paper Slot 3

    The author claims that Silicon Valley tech companies have tried to “confuse the public” by:

    1. promoting screen time in public schools while opting for a screen-free education for their own children. 
    2. developing new work-efficiency programmes while lobbying for the “right to disconnect” bill.
    3. concealing the findings of psychologists and neuroscientists on screen-time use from the public.
    4. pushing for greater privacy while working with advertisement-support platforms to mine data.
    Choice A
    promoting screen time in public schools while opting for a screen-free education for their own children.

The passage below is accompanied by a set of questions. Choose the best answer to each question.

I’ve been following the economic crisis for more than two years now. I began working on the subject as part of the background to a novel, and soon realized that I had stumbled across the most interesting story I’ve ever found. While I was beginning to work on it, the British bank Northern Rock blew up, and it became clear that, as I wrote at the time, “If our laws are not extended to control the new kinds of super-powerful, super-complex, and potentially super-risky investment vehicles, they will one day cause a financial disaster of global-systemic proportions.” . . . I was both right and too late, because all the groundwork for the crisis had already been done—though the sluggishness of the world’s governments, in not preparing for the great unraveling of autumn 2008, was then and still is stupefying. But this is the first reason why I wrote this book: because what’s happened is extraordinarily interesting. It is an absolutely amazing story, full of human interest and drama, one whose byways of mathematics, economics, and psychology are both central to the story of the last decades and mysteriously unknown to the general public. We have heard a lot about “the two cultures” of science and the arts—we heard a particularly large amount about it in 2009, because it was the fiftieth anniversary of the speech during which C. P. Snow first used the phrase. But I’m not sure the idea of a huge gap between science and the arts is as true as it was half a century ago—it’s certainly true, for instance, that a general reader who wants to pick up an education in the fundamentals of science will find it easier than ever before. It seems to me that there is a much bigger gap between the world of finance and that of the general public and that there is a need to narrow that gap, if the financial industry is not to be a kind of priesthood, administering to its own mysteries and feared and resented by the rest of us. Many bright, literate people have no idea about all sorts of economic basics, of a type that financial insiders take as elementary facts of how the world works. I am an outsider to finance and economics, and my hope is that I can talk across that gulf.

My need to understand is the same as yours, whoever you are. That’s one of the strangest ironies of this story: after decades in which the ideology of the Western world was personally and economically individualistic, we’ve suddenly been hit by a crisis which shows in the starkest terms that whether we like it or not—and there are large parts of it that you would have to be crazy to like—we’re all in this together. The aftermath of the crisis is going to dominate the economics and politics of our societies for at least a decade to come and perhaps longer.

  1. CAT 2020 Question Paper Slot 3

    Which one of the following, if false, could be seen as supporting the author’s claims?

    1. The global economic crisis lasted for more than two years. 
    2. The huge gap between science and the arts has steadily narrowed over time.
    3. The economic crisis was not a failure of collective action to rectify economic problems.
    4. Most people are yet to gain any real understanding of the workings of the financial world.
    Choice C
    The economic crisis was not a failure of collective action to rectify economic problems.

  2. CAT 2020 Question Paper Slot 3

    Which one of the following, if true, would be an accurate inference from the first sentence of the passage?

    1. The author’s preoccupation with the economic crisis is not less than two years old.
    2. The economic crisis outlasted the author’s preoccupation with it.
    3. The author is preoccupied with the economic crisis because he is being followed.
    4. The author has witnessed many economic crises by travelling a lot for two years.
    Choice A
    The author’s preoccupation with the economic crisis is not less than two years old.

  3. CAT 2020 Question Paper Slot 3

    Which one of the following best captures the main argument of the last paragraph of the passage?

    1. Whoever you are, you would be crazy to think that there is no crisis.
    2. In the decades to come, other ideologies will emerge in the aftermath of the crisis.
    3. The ideology of individualism must be set aside in order to deal with the crisis.
    4. The aftermath of the crisis will strengthen the central ideology of individualism in the Western world.
    Choice C
    The ideology of individualism must be set aside in order to deal with the crisis.

  4. CAT 2020 Question Paper Slot 3

    All of the following, if true, could be seen as supporting the arguments in the passage, EXCEPT:

    1. The story of the economic crisis is also one about international relations, global financial security, and mass psychology.
    2. Economic crises could be averted by changing prevailing ideologies and beliefs.
    3. The failure of economic systems does not necessarily mean the failure of their ideologies.
    4. The difficulty with understanding financial matters is that they have become so arcane.
    Choice C
    The failure of economic systems does not necessarily mean the failure of their ideologies.

  5. CAT 2020 Question Paper Slot 3

    According to the passage, the author is likely to be supportive of which one of the following programmes?

    1. An educational curriculum that promotes economic research.
    2. An educational curriculum that promotes developing financial literacy in the masses.
    3. The complete nationalisation of all financial institutions.
    4. Economic policies that are more sensitively calibrated to the fluctuations of the market.
    Choice B
    An educational curriculum that promotes developing financial literacy in the masses.

CAT VARC : CAT 2019 Question Paper Slot 1

Tale of Aladdin

In the past, credit for telling the tale of Aladdin has often gone to Antoine Galland . . . the first European translator of . . . Arabian Nights [which] started as a series of translations of an incomplete manuscript of a medieval Arabic story collection. . . But, though those tales were of medieval origin, Aladdin may be a more recent invention. Scholars have not found a manuscript of the story that predates the version published in 1712 by Galland, who wrote in his diary that he first heard the tale from a Syrian storyteller from Aleppo named Hanna Diyab . . .

Despite the fantastical elements of the story, scholars now think the main character may actually be based on a real person’s real experiences. . . . Though Galland never credited Diyab in his published translations of the Arabian Nights stories, Diyab wrote something of his own: a travelogue penned in the mid-18th century. In it, he recalls telling Galland the story of Aladdin [and] describes his own hard-knocks upbringing and the way he marveled at the extravagance of Versailles. The descriptions he uses were very similar to the descriptions of the lavish palace that ended up in Galland’s version of the Aladdin story. [Therefore, author Paulo Lemos] Horta believes that “Aladdin might be the young Arab Maronite from Aleppo, marveling at the jewels and riches of Versailles.” . . .

For 300 years, scholars thought that the rags-to-riches story of Aladdin might have been inspired by the plots of French fairy tales that came out around the same time, or that the story was invented in that 18th century period as a byproduct of French Orientalism, a fascination with stereotypical exotic Middle Eastern luxuries that was prevalent then. The idea that Diyab might have based it on his own life — the experiences of a Middle Eastern man encountering the French, not vice-versa — flips the script. [According to Horta,] “Diyab was ideally placed to embody the overlapping world of East and West, blending the storytelling traditions of his homeland with his youthful observations of the wonder of 18th-century France.” . . .

To the scholars who study the tale, its narrative drama isn’t the only reason storytellers keep finding reason to return to Aladdin. It reflects not only “a history of the French and the Middle East, but also [a story about] Middle Easterners coming to Paris and that speaks to our world today,” as Horta puts it. “The day Diyab told the story of Aladdin to Galland, there were riots due to food shortages during the winter and spring of 1708 to 1709, and Diyab was sensitive to those people in a way that Galland is not. When you read this diary, you see this solidarity among the Arabs who were in Paris at the time. . . . There is little in the writings of Galland that would suggest that he was capable of developing a character like Aladdin with sympathy, but Diyab’s memoir reveals a narrator adept at capturing the distinctive psychology of a young protagonist, as well as recognizing the kinds of injustices and opportunities that can transform the path of any youthful adventurer.”

  1. CAT 2019 Question Paper - Slot 1

    All of the following serve as evidence for the character of Aladdin being based on Hanna Diyab EXCEPT:

    1. Diyab’s cosmopolitanism and cross-cultural experience.
    2. Diyab’s humble origins and class struggles, as recounted in his travelogue.
    3. Diyab’s description of the wealth of Versailles in his travelogue.
    4. Diyab’s narration of the original story to Galland.
    Choice D
    Diyab’s narration of the original story to Galland.

  2. CAT 2019 Question Paper - Slot 1

    The author of the passage is most likely to agree with which of the following explanations for the origins of the story of Aladdin?

    1. Galland derived the story of Aladdin from Diyab’s travelogue in which he recounts his fascination with the wealth of Versailles.
    2. The story of Aladdin has its origins in an undiscovered, incomplete manuscript of a medieval Arabic collection of stories.
    3. Basing it on his own life experiences, Diyab transmitted the story of Aladdin to Galland who included it in Arabian Nights.
    4. Galland received the story of Aladdin from Diyab who, in turn, found it in an incomplete medieval manuscript.
    Choice C
    Basing it on his own life experiences, Diyab transmitted the story of Aladdin to Galland who included it in Arabian Nights.

  3. CAT 2019 Question Paper - Slot 1

    Which of the following, if true, would invalidate the inversion that the phrase “flips the script” refers to?

    1. Diyab’s travelogue described the affluence of the French city of Bordeaux, instead of Versailles.
    2. The French fairy tales of the eighteenth century did not have rags-to-riches plot lines like that of the tale of Aladdin.
    3. The description of opulence in Hanna Diyab’s and Antoine Galland’s narratives bore no resemblance to each other.
    4. Galland acknowledged in the published translations of Arabian Nights that he heard the story of Aladdin from Diyab.
    Choice C
    The description of opulence in Hanna Diyab’s and Antoine Galland’s narratives bore no resemblance to each other.

  4. CAT 2019 Question Paper - Slot 1

    Which of the following is the primary reason for why storytellers are still fascinated by the story of Aladdin?

    1. The traveller's experience that inspired the tale of Aladdin resonates even today.
    2. The tale of Aladdin documents the history of Europe and Middle East.
    3. The archetype of the rags-to-riches story of Aladdin makes it popular even today.
    4. The story of Aladdin is evidence of the eighteenth century French Orientalist.
    Choice A
    The traveller's experience that inspired the tale of Aladdin resonates even today.

  5. CAT 2019 Question Paper - Slot 1

    Which of the following does not contribute to the passage’s claim about the authorship of Aladdin?

    1. The narrative sensibility of Diyab’s travelogue.
    2. The depiction of the affluence of Versailles in Diyab’s travelogue.
    3. Galland’s acknowledgment of Diyab in his diary.
    4. The story-line of many French fairy tales of the 18th century.
    Choice D
    The story-line of many French fairy tales of the 18th century.

Choice Fatigue

Contemporary internet shopping conjures a perfect storm of choice anxiety. Research has consistently held that people who are presented with a few options make better, easier decisions than those presented with many. . . . Helping consumers figure out what to buy amid an endless sea of choice online has become a cottage industry unto itself. Many brands and retailers now wield marketing buzzwords such as curation, differentiation, and discovery as they attempt to sell an assortment of stuff targeted to their ideal customer. Companies find such shoppers through the data gold mine of digital advertising, which can catalog people by gender, income level, personal interests, and more. Since Americans have lost the ability to sort through the sheer volume of the consumer choices available to them, a ghost now has to be in the retail machine, whether it’s an algorithm, an influencer, or some snazzy ad tech to help a product follow you around the internet. Indeed, choice fatigue is one reason so many people gravitate toward lifestyle influencers on Instagram—the relentlessly chic young moms and perpetually vacationing 20-somethings—who present an aspirational worldview, and then recommend the products and services that help achieve it. . . .

For a relatively new class of consumer-products start-ups, there’s another method entirely. Instead of making sense of a sea of existing stuff, these companies claim to disrupt stuff as Americans know it. Casper (mattresses), Glossier (makeup), Away (suitcases), and many others have sprouted up to offer consumers freedom from choice: The companies have a few aesthetically pleasing and supposedly highly functional options, usually at mid-range prices. They’re selling nice things, but maybe more importantly, they’re selling a confidence in those things, and an ability to opt out of the stuff rat race. . . .

One-thousand-dollar mattresses and $300 suitcases might solve choice anxiety for a certain tier of consumer, but the companies that sell them, along with those that attempt to massage the larger stuff economy into something navigable, are still just working within a consumer market that’s broken in systemic ways. The presence of so much stuff in America might be more valuable if it were more evenly distributed, but stuff’s creators tend to focus their energy on those who already have plenty. As options have expanded for people with disposable income, the opportunity to buy even basic things such as fresh food or quality diapers has contracted for much of America’s lower classes.

For start-ups that promise accessible simplicity, their very structure still might eventually push them toward overwhelming variety. Most of these companies are based on hundreds of millions of dollars of venture capital, the investors of which tend to expect a steep growth rate that can’t be achieved by selling one great mattress or one great sneaker. Casper has expanded into bedroom furniture and bed linens. Glossier, after years of marketing itself as no-makeup makeup that requires little skill to apply, recently launched a full line of glittering color cosmetics. There may be no way to opt out of stuff by buying into the right thing.

  1. CAT 2019 Question Paper - Slot 1

    Which of the following hypothetical statements would add the least depth to the author’s prediction of the fate of start-ups offering few product options?

    1. An exponential surge in their sales enables start-ups to meet their desired profit goals without expanding their product catalogue
    2. With the motive of promoting certain rival companies, the government decides to double the tax-rates for these start-ups.
    3. With Casper and Glossier venturing into new product ranges, their regular customers start losing trust in the companies and their products.
    4. Start-ups with few product options are no exception to the American consumer market that is deeply divided along class lines.
    Choice A
    An exponential surge in their sales enables start-ups to meet their desired profit goals without expanding their product catalogue

  2. CAT 2019 Question Paper - Slot 1

    Which one of the following best sums up the overall purpose of the examples of Casper and Glossier in the passage?

    1. They are increasing the purchasing power of poor Americans.
    2. They are exceptions to a dominant trend in consumer markets.
    3. They are facilitating a uniform distribution of commodities in the market.
    4. They might transform into what they were exceptions to.
    Choice D
    They might transform into what they were exceptions to.

  3. CAT 2019 Question Paper - Slot 1

    A new food brand plans to launch a series of products in the American market. Which of the following product plans is most likely to be supported by the author of the passage?

    1. A range of 10 products priced between $5 and $10.
    2. A range of 25 products priced between $10 and $25.
    3. A range of 25 products priced between $5 and $10.
    4. A range of 10 products priced between $10 and $25.
    Choice A
    A range of 10 products priced between $5 and $10.

  4. CAT 2019 Question Paper - Slot 1

    All of the following, IF TRUE, would weaken the author’s claims EXCEPT:

    1. product options increased market competition, bringing down the prices of commodities, which, in turn, increased purchasing power of the poor.
    2. the annual sales growth of companies with fewer product options were higher than that of companies which curated their products for target consumers.
    3. the annual sale of companies that hired lifestyle influencers on Instagram for marketing their products were 40% less than those that did not.
    4. the empowerment felt by purchasers in buying a commodity were directly proportional to the number of options they could choose from.
    Choice B
    the annual sales growth of companies with fewer product options were higher than that of companies which curated their products for target consumers.

  5. CAT 2019 Question Paper - Slot 1

    Based on the passage, all of the following can be inferred about consumer behaviour EXCEPT that:

    1. too many options have made it difficult for consumers to trust products.
    2. consumers are susceptible to marketing images that they see on social media.
    3. having too many product options can be overwhelming for consumers.
    4. consumers tend to prefer products by start-ups over those by established companies.
    Choice D
    consumers tend to prefer products by start-ups over those by established companies.

Emperor Penguins

Scientists recently discovered that Emperor Penguins—one of Antarctica’s most celebrated species—employ a particularly unusual technique for surviving the daily chill. As detailed in an article published today in the journal Biology Letters, the birds minimize heat loss by keeping the outer surface of their plumage below the temperature of the surrounding air. At the same time, the penguins’ thick plumage insulates their body and keeps it toasty. . . .

The researchers analyzed thermographic images . . . taken over roughly a month during June 2008. During that period, the average air temperature was 0.32 degrees Fahrenheit. At the same time, the majority of the plumage covering the penguins’ bodies was even colder: the surface of their warmest body part, their feet, was an average 1.76 degrees Fahrenheit, but the plumage on their heads, chests and backs were -1.84, -7.24 and -9.76 degrees Fahrenheit respectively. Overall, nearly the entire outer surface of the penguins’ bodies was below freezing at all times, except for their eyes and beaks. The scientists also used a computer simulation to determine how much heat was lost or gained from each part of the body—and discovered that by keeping their outer surface below air temperature, the birds might paradoxically be able to draw very slight amounts of heat from the air around them. The key to their trick is the difference between two different types of heat transfer: radiation and convection.

The penguins do lose internal body heat to the surrounding air through thermal radiation, just as our bodies do on a cold day. Because their bodies (but not surface plumage) are warmer than the surrounding air, heat gradually radiates outward over time, moving from a warmer material to a colder one. To maintain body temperature while losing heat, penguins, like all warm-blooded animals, rely on the metabolism of food. The penguins, though, have an additional strategy. Since their outer plumage is even colder than the air, the simulation showed that they might gain back a little of this heat through thermal convection—the transfer of heat via the movement of a fluid (in this case, the air). As the cold Antarctic air cycles around their bodies, slightly warmer air comes into contact with the plumage and donates minute amounts of heat back to the penguins, then cycles away at a slightly colder temperature.

Most of this heat, the researchers note, probably doesn’t make it all the way through the plumage and back to the penguins’ bodies, but it could make a slight difference. At the very least, the method by which a penguin’s plumage wicks heat from the bitterly cold air that surrounds it helps to cancel out some of the heat that’s radiating from its interior. And given the Emperors’ unusually demanding breeding cycle, every bit of warmth counts. . . . Since [penguins trek as far as 75 miles to the coast to breed and male penguins] don’t eat anything during [the incubation period of 64 days], conserving calories by giving up as little heat as possible is absolutely crucial.

  1. CAT 2019 Question Paper - Slot 1

    In the last sentence of paragraph 3, “slightly warmer air” and “at a slightly colder temperature” refer to ______ AND ______ respectively:

    1. the air inside penguins’ bodies kept warm because of metabolism of food AND the fall in temperature of the body air after it transfers some heat to the plumage.
    2. the cold Antarctic air which becomes warmer because of the heat radiated out from penguins’ bodies AND the fall in temperature of the surrounding air after thermal convection.
    3. the air trapped in the plumage which is warmer than the Antarctic air AND the fall in temperature of the trapped plumage air after it radiates out some heat.
    4. the cold Antarctic air whose temperature is higher than that of the plumage AND the fall in temperature of the Antarctic air after it has transmitted some heat to the plumage.
    Choice D
    the cold Antarctic air whose temperature is higher than that of the plumage AND the fall in temperature of the Antarctic air after it has transmitted some heat to the plumage.

  2. CAT 2019 Question Paper - Slot 1

    Which of the following best explains the purpose of the word “paradoxically” as used by the author?

    1. Keeping their body colder helps penguins keep their plumage warmer.
    2. Keeping a part of their body colder helps penguins keep their bodies warmer.
    3. Heat gain through radiation happens despite the heat loss through convection.
    4. Heat loss through radiation happens despite the heat gain through convection.
    Choice B
    Keeping a part of their body colder helps penguins keep their bodies warmer.

  3. CAT 2019 Question Paper - Slot 1

    All of the following, if true, would negate the findings of the study reported in the passage EXCEPT:

    1. the penguins’ plumage were made of a material that did not allow any heat transfer through convection or radiation.
    2. the average air temperature recorded during the month of June 2008 in the area of study were –10 degrees Fahrenheit.
    3. the temperature of the plumage on the penguins’ heads, chests and backs were found to be 1.84, 7.24 and 9.76 degrees Fahrenheit respectively.
    4. the average temperature of the feet of penguins in the month of June 2008 were found to be 2.76 degrees Fahrenheit.
    Choice D
    the average temperature of the feet of penguins in the month of June 2008 were found to be 2.76 degrees Fahrenheit.

  4. CAT 2019 Question Paper - Slot 1

    Which of the following can be responsible for Emperor Penguins losing body heat?

    1. Food metabolism.
    2. Reproduction process.
    3. Plumage.
    4. Thermal convection.
    Choice B
    Reproduction process.

Folk Music

"Free of the taint of manufacture" – that phrase, in particular, is heavily loaded with the ideology of what the Victorian socialist William Morris called the "anti-scrape", or an anti- capitalist conservationism (not conservatism) that solaced itself with the vision of a pre- industrial golden age. In Britain, folk may often appear a cosy, fossilised form, but when you look more closely, the idea of folk – who has the right to sing it, dance it, invoke it, collect it, belong to it or appropriate it for political or cultural ends – has always been contested territory. . . .

In our own time, though, the word "folk" . . . has achieved the rare distinction of occupying fashionable and unfashionable status simultaneously. Just as the effusive floral prints of the radical William Morris now cover genteel sofas, so the revolutionary intentions of many folk historians and revivalists have led to music that is commonly regarded as parochial and conservative. And yet – as newspaper columns periodically rejoice – folk is hip again, influencing artists, clothing and furniture designers, celebrated at music festivals, awards ceremonies and on TV, reissued on countless record labels. Folk is a sonic "shabby chic", containing elements of the uncanny and eerie, as well as an antique veneer, a whiff of Britain's heathen dark ages. The very obscurity and anonymity of folk music's origins open up space for rampant imaginative fancies. . . .

[Cecil Sharp, who wrote about this subject, believed that] folk songs existed in constant transformation, a living example of an art form in a perpetual state of renewal. "One man sings a song, and then others sing it after him, changing what they do not like" is the most concise summary of his conclusions on its origins. He compared each rendition of a ballad to an acorn falling from an oak tree; every subsequent iteration sows the song anew. But there is tension in newness. In the late 1960s, purists were suspicious of folk songs recast in rock idioms. Electrification, however, comes in many forms. For the early-20th-century composers such as Vaughan Williams and Holst, there were thunderbolts of inspiration from oriental mysticism, angular modernism and the body blow of the first world war, as well as input from the rediscovered folk tradition itself.

For the second wave of folk revivalists, such as Ewan MacColl and AL Lloyd, starting in the 40s, the vital spark was communism's dream of a post-revolutionary New Jerusalem. For their younger successors in the 60s, who thronged the folk clubs set up by the old guard, the lyrical freedom of Dylan and the unchained melodies of psychedelia created the conditions for folk- rock's own golden age, a brief Indian summer that lasted from about 1969 to 1971. . . . Four decades on, even that progressive period has become just one more era ripe for fashionable emulation and pastiche. The idea of a folk tradition being exclusively confined to oral transmission has become a much looser, less severely guarded concept. Recorded music and television, for today's metropolitan generation, are where the equivalent of folk memories are seeded. . . .

  1. CAT 2019 Question Paper - Slot 1

    The author says that folk “may often appear a cosy, fossilised form” because:

    1. of its nostalgic association with a pre-industrial past.
    2. it has been arrogated for various political and cultural purposes.
    3. folk is a sonic “shabby chic” with an antique veneer.
    4. the notion of folk has led to several debates and disagreements.
    Choice A
    of its nostalgic association with a pre-industrial past.

  2. CAT 2019 Question Paper - Slot 1

    All of the following are causes for plurality and diversity within the British folk tradition EXCEPT:

    1. the fluidity of folk forms owing to their history of oral mode of transmission.
    2. paradoxically, folk forms are both popular and unpopular.
    3. that British folk forms can be traced to the remote past of the country.
    4. that British folk continues to have traces of pagan influence from the dark ages.
    Choice B
    paradoxically, folk forms are both popular and unpopular.

  3. CAT 2019 Question Paper - Slot 1

    At a conference on folk forms, the author of the passage is least likely to agree with which one of the following views?

    1. The power of folk resides in its contradictory ability to influence and be influenced by the present while remaining rooted in the past.
    2. Folk forms, despite their archaic origins, remain intellectually relevant in contemporary times.
    3. Folk forms, in their ability to constantly adapt to the changing world, exhibit an unusual poise and homogeneity with each change.
    4. The plurality and democratising impulse of folk forms emanate from the improvisation that its practitioners bring to it.
    Choice C
    Folk forms, in their ability to constantly adapt to the changing world, exhibit an unusual poise and homogeneity with each change.

  4. CAT 2019 Question Paper - Slot 1

    The primary purpose of the reference to William Morris and his floral prints is to show:

    1. the pervasive influence of folk on contemporary art, culture, and fashion.
    2. that what is once regarded as radical in folk, can later be seen as conformist.
    3. that what was once derided as genteel is now considered revolutionary.
    4. that despite its archaic origins, folk continues to remain a popular tradition.
    Choice B
    that what is once regarded as radical in folk, can later be seen as conformist.

  5. CAT 2019 Question Paper - Slot 1

    Which of the following statements about folk revivalism of the 1940s and 1960s cannot be inferred from the passage?

    1. Freedom and rebellion were popular themes during the second wave of folk revivalism.
    2. Electrification of music would not have happened without the influence of rock music.
    3. Even though it led to folk-rock’s golden age, it wasn’t entirely free from critique.
    4. It reinforced Cecil Sharp’s observation about folk’s constant transformation.
    Choice B
    Electrification of music would not have happened without the influence of rock music.

Topophilia

As defined by the geographer Yi-Fu Tuan, topophilia is the affective bond between people and place. His 1974 book set forth a wide-ranging exploration of how the emotive ties with the material environment vary greatly from person to person and in intensity, subtlety, and mode of expression. Factors influencing one’s depth of response to the environment include cultural background, gender, race, and historical circumstance, and Tuan also argued that there is a biological and sensory element. Topophilia might not be the strongest of human emotions— indeed, many people feel utterly indifferent toward the environments that shape their lives— but when activated it has the power to elevate a place to become the carrier of emotionally charged events or to be perceived as a symbol.

Aesthetic appreciation is one way in which people respond to the environment. A brilliantly colored rainbow after gloomy afternoon showers, a busy city street alive with human interaction—one might experience the beauty of such landscapes that had seemed quite ordinary only moments before or that are being newly discovered. This is quite the opposite of a second topophilic bond, namely that of the acquired taste for certain landscapes and places that one knows well. When a place is home, or when a space has become the locus of memories or the means of gaining a livelihood, it frequently evokes a deeper set of attachments than those predicated purely on the visual. A third response to the environment also depends on the human senses but may be tactile and olfactory, namely a delight in the feel and smell of air, water, and the earth.

Topophilia—and its very close conceptual twin, sense of place—is an experience that, however elusive, has inspired recent architects and planners. Most notably, new urbanism seeks to counter the perceived placelessness of modern suburbs and the decline of central cities through neo-traditional design motifs. Although motivated by good intentions, such attempts to create places rich in meaning are perhaps bound to disappoint. As Tuan noted, purely aesthetic responses often are suddenly revealed, but their intensity rarely is long- lasting. Topophilia is difficult to design for and impossible to quantify, and its most articulate interpreters have been self-reflective philosophers such as Henry David Thoreau, evoking a marvelously intricate sense of place at Walden Pond, and Tuan, describing his deep affinity for the desert.

Topophilia connotes a positive relationship, but it often is useful to explore the darker affiliations between people and place. Patriotism, literally meaning the love of one’s terra patria or homeland, has long been cultivated by governing elites for a range of nationalist projects, including war preparation and ethnic cleansing. Residents of upscale residential developments have disclosed how important it is to maintain their community’s distinct identity, often by casting themselves in a superior social position and by reinforcing class and racial differences. And just as a beloved landscape is suddenly revealed, so too may landscapes of fear cast a dark shadow over a place that makes one feel a sense of dread or anxiety—or topophobia.

  1. CAT 2019 Question Paper - Slot 1

    The word “topophobia” in the passage is used:

    1. to represent a feeling of dread towards particular spaces and places.
    2. as a metaphor expressing the failure of the homeland to accommodate non-citizens.
    3. to signify the fear of studying the complex discipline of topography.
    4. to signify feelings of fear or anxiety towards topophilic people.
    Choice A
    to represent a feeling of dread towards particular spaces and places.

  2. CAT 2019 Question Paper - Slot 1

    In the last paragraph, the author uses the example of “Residents of upscale residential developments” to illustrate the:

    1. introduction of nationalist projects by such elites to produce a sense of dread or topophobia.
    2. manner in which environments are designed to minimise the social exclusion of their clientele.
    3. sensitive response to race and class problems in upscale residential developments.
    4. social exclusivism practised by such residents in order to enforce a sense of racial or class superiority.
    Choice D
    social exclusivism practised by such residents in order to enforce a sense of racial or class superiority.

  3. CAT 2019 Question Paper - Slot 1

    Which one of the following best captures the meaning of the statement, “Topophilia is difficult to design for and impossible to quantify . . .”?

    1. Philosopher-architects are uniquely suited to develop topophilic design.
    2. People’s responses to their environment are usually subjective and so cannot be rendered in design.
    3. Architects have to objectively quantify spaces and hence cannot be topophilic.
    4. The deep anomie of modern urbanisation led to new urbanism’s intricate sense of place.
    Choice B
    People’s responses to their environment are usually subjective and so cannot be rendered in design.

  4. CAT 2019 Question Paper - Slot 1

    Which one of the following comes closest in meaning to the author’s understanding of topophilia?

    1. The French are not overly patriotic, but they will refuse to use English as far as possible, even when they know it well.
    2. The tendency of many cultures to represent their land as “motherland” or “fatherland” may be seen as an expression of their topophilia.
    3. Scientists have found that most creatures, including humans, are either born with or cultivate a strong sense of topography.
    4. Nomadic societies are known to have the least affinity for the lands through which they traverse because they tend to be topophobic.
    Choice B
    The tendency of many cultures to represent their land as “motherland” or “fatherland” may be seen as an expression of their topophilia.

  5. CAT 2019 Question Paper - Slot 1

    Which of the following statements, if true, could be seen as not contradicting the arguments in the passage?

    1. Generally speaking, in a given culture, the ties of the people to their environment vary little in significance or intensity.
    2. Patriotism, usually seen as a positive feeling, is presented by the author as a darker form of topophilia.
    3. New Urbanism succeeded in those designs where architects collaborated with their clients.
    4. The most important, even fundamental, response to our environment is our tactile and olfactory response.
    Choice B
    Patriotism, usually seen as a positive feeling, is presented by the author as a darker form of topophilia.

CAT VARC : CAT 2019 Question Paper Slot 2

Bureaucracy

Around the world, capital cities are disgorging bureaucrats. In the post-colonial fervour of the 20th century, coastal capitals picked by trade-focused empires were spurned for “regionally neutral” new ones . . . . But decamping wholesale is costly and unpopular; governments these days prefer piecemeal dispersal. The trend reflects how the world has changed. In past eras, when information travelled at a snail’s pace, civil servants had to cluster together. But now desk-workers can ping emails and video-chat around the world. Travel for face-to-face meetings may be unavoidable, but transport links, too, have improved. . . .

Proponents of moving civil servants around promise countless benefits. It disperses the risk that a terrorist attack or natural disaster will cripple an entire government. Wonks in the sticks will be inspired by new ideas that walled-off capitals cannot conjure up. Autonomous regulators perform best far from the pressure and lobbying of the big city. Some even hail a cure for ascendant cynicism and populism. The unloved bureaucrats of faraway capitals will become as popular as firefighters once they mix with regular folk.

Beyond these sunny visions, dispersing central-government functions usually has three specific aims: to improve the lives of both civil servants and those living in clogged capitals; to save money; and to redress regional imbalances. The trouble is that these goals are not always realised.

The first aim—improving living conditions—has a long pedigree. After the second world war Britain moved thousands of civil servants to “agreeable English country towns” as London was rebuilt. But swapping the capital for somewhere smaller is not always agreeable. Attrition rates can exceed 80%. . . . The second reason to pack bureaucrats off is to save money. Office space costs far more in capitals. . . . Agencies that are moved elsewhere can often recruit better workers on lower salaries than in capitals, where well-paying multinationals mop up talent.

The third reason to shift is to rebalance regional inequality. . . . Norway treats federal jobs as a resource every region deserves to enjoy, like profits from oil. Where government jobs go, private ones follow. . . . Sometimes the aim is to fulfil the potential of a country’s second-tier cities. Unlike poor, remote places, bigger cities can make the most of relocated government agencies, linking them to local universities and businesses and supplying a better-educated workforce. The decision in 1946 to set up America’s Centres for Disease Control in Atlanta rather than Washington, D.C., has transformed the city into a hub for health-sector research and business.

The dilemma is obvious. Pick small, poor towns, and areas of high unemployment get new jobs, but it is hard to attract the most qualified workers; opt for larger cities with infrastructure and better-qualified residents, and the country’s most deprived areas see little benefit. . . .

Others contend that decentralisation begets corruption by making government agencies less accountable. . . . A study in America found that state-government corruption is worse when the state capital is isolated—journalists, who tend to live in the bigger cities, become less watchful of those in power.

  1. CAT 2019 Question Paper - Slot 2

    According to the passage, colonial powers located their capitals:

    1. to showcase their power and prestige.
    2. where they had the densest populations.
    3. based on political expediency.
    4. to promote their trading interests.
    Choice D
    to promote their trading interests.

  2. CAT 2019 Question Paper - Slot 2

    The “dilemma” mentioned in the passage refers to:

    1. keeping government agencies in the largest city with good infrastructure or moving them to a remote area with few amenities.
    2. concentrating on decongesting large cities or focusing on boosting employment in relatively larger cities.
    3. encouraging private enterprises to relocate to smaller towns or not incentivising them in order to keep government costs in those towns low.
    4. relocating government agencies to boost growth in remote areas with poor amenities or to relatively larger cities with good amenities.
    Choice D
    relocating government agencies to boost growth in remote areas with poor amenities or to relatively larger cities with good amenities.

  3. CAT 2019 Question Paper - Slot 2

    People who support decentralising central government functions are LEAST likely to cite which of the following reasons for their view?

    1. More independence could be enjoyed by regulatory bodies located away from political centres.
    2. Policy makers may benefit from fresh thinking in a new environment.
    3. It reduces expenses as infrastructure costs and salaries are lower in smaller cities.
    4. It could weaken the nexus between bureaucrats and media in the capital.
    Choice D
    It could weaken the nexus between bureaucrats and media in the capital.

  4. CAT 2019 Question Paper - Slot 2

    The “long pedigree” of the aim to shift civil servants to improve their living standards implies that this move:

    1. is not a new idea and has been tried in the past.
    2. has become common practice in several countries worldwide.
    3. is supported by politicians and the ruling elites.
    4. takes a long time to achieve its intended outcomes.
    Choice A
    is not a new idea and has been tried in the past.

  5. CAT 2019 Question Paper - Slot 2

    According to the author, relocating government agencies has not always been a success for all of the following reasons EXCEPT:

    1. a rise in pollution levels and congestion in the new locations.
    2. the difficulty of attracting talented, well-skilled people in more remote areas.
    3. increased avenues of corruption away from the capital city.
    4. high staff losses, as people may not be prepared to move to smaller towns.
    Choice A
    a rise in pollution levels and congestion in the new locations.

Preservation

War, natural disasters and climate change are destroying some of the world's most precious cultural sites. Google is trying to help preserve these archaeological wonders by allowing users access to 3D images of these treasures through its site.
But the project is raising questions about Google's motivations and about who should own the digital copyrights. Some critics call it a form of "digital colonialism."
When it comes to archaeological treasures, the losses have been mounting. ISIS blew up parts of the ancient city of Palmyra in Syria and an earthquake hit Bagan, an ancient city in Myanmar, damaging dozens of temples, in 2016. In the past, all archaeologists and historians had for restoration and research were photos, drawings, remnants and intuition.
But that's changing. Before the earthquake at Bagan, many of the temples on the site were scanned. . . . [These] scans . . . are on Google's Arts & Culture site. The digital renditions allow viewers to virtually wander the halls of the temple, look up-close at paintings and turn the building over, to look up at its chambers. . . . [Google Arts & Culture] works with museums and other nonprofits . . . to put high-quality images online.
The images of the temples in Bagan are part of a collaboration with CyArk, a nonprofit that creates the 3D scanning of historic sites. . . . Google . . . says [it] doesn't make money off this website, but it fits in with Google's mission to make the world's information available and useful.
Critics say the collaboration could be an attempt by a large corporation to wrap itself in the sheen of culture. Ethan Watrall, an archaeologist, professor at Michigan State University and a member of the Society for American Archaeology, says he's not comfortable with the arrangement between CyArk and Google. . . . Watrall says this project is just a way for Google to promote Google. "They want to make this material accessible so people will browse it and be filled with wonder by it," he says. "But at its core, it's all about advertisements and driving traffic." Watrall says these images belong on the site of a museum or educational institution, where there is serious scholarship and a very different mission. . . .
[There's] another issue for some archaeologists and art historians. CyArk owns the copyrights of the scans — not the countries where these sites are located. That means the countries need CyArk's permission to use these images for commercial purposes.
Erin Thompson, a professor of art crime at John Jay College of Criminal Justice in New York City, says it's the latest example of a Western nation appropriating a foreign culture, a centuries-long battle. . . . CyArk says it copyrights the scans so no one can use them in an inappropriate way. The company says it works closely with authorities during the process, even training local people to help. But critics like Thompson are not persuaded. . . . She would prefer the scans to be owned by the countries and people where these sites are located.

  1. CAT 2019 Question Paper - Slot 2

    Based on his views mentioned in the passage, one could best characterise Dr. Watrall as being:

    1. opposed to the use of digital technology in archaeological and cultural sites in developing countries.
    2. dismissive of laypeople’s access to specialist images of archaeological and cultural sites.
    3. uneasy about the marketing of archaeological images for commercial use by firms such as Google and CyArk.
    4. critical about the links between a non-profit and a commercial tech platform for distributing archaeological images.
    Choice D
    critical about the links between a non-profit and a commercial tech platform for distributing archaeological images.

  2. CAT 2019 Question Paper - Slot 2

    By “digital colonialism”, critics of the CyArk–Google project are referring to the fact that:

    1. CyArk and Google have been scanning images without copyright permission from host countries.
    2. the scanning process can damage delicate frescos and statues at the sites.
    3. countries where the scanned sites are located do not own the scan copyrights.
    4. CyArk and Google have not shared the details of digitisation with the host countries.
    Choice C
    countries where the scanned sites are located do not own the scan copyrights.

  3. CAT 2019 Question Paper - Slot 2

    Which of the following, if true, would most strongly invalidate Dr. Watrall’s objections?

    1. Google takes down advertisements on its website hosting CyArk’s scanned images.
    2. There is a ban on CyArk scanning archeological sites located in other countries.
    3. CyArk does not own the copyright on scanned images of archaeological sites.
    4. CyArk uploads its scanned images of archaeological sites onto museum websites only.
    Choice D
    CyArk uploads its scanned images of archaeological sites onto museum websites only.

  4. CAT 2019 Question Paper - Slot 2

    In Dr. Thompson’s view, CyArk owning the copyright of its digital scans of archaeological sites is akin to:

    1. tourists uploading photos of monuments onto social media.
    2. the seizing of ancient Egyptian artefacts by a Western museum.
    3. the illegal downloading of content from the internet.
    4. digital platforms capturing users’ data for market research.
    Choice B
    the seizing of ancient Egyptian artefacts by a Western museum.

  5. CAT 2019 Question Paper - Slot 2

    Of the following arguments, which one is LEAST likely to be used by the companies that digitally scan cultural sites?

    1. It enables people who cannot physically visit these sites to experience them.
    2. It helps preserve precious images in case the sites are damaged or destroyed.
    3. It allows a large corporation to project itself as a protector of culture.
    4. It provides images free of cost to all users.
    Choice C
    It allows a large corporation to project itself as a protector of culture.

Urban Settlements

The magic of squatter cities is that they are improved steadily and gradually by their residents. To a planner’s eye, these cities look chaotic. I trained as a biologist and to my eye, they look organic. Squatter cities are also unexpectedly green. They have maximum density—1 million people per square mile in some areas of Mumbai—and have minimum energy and material use. People get around by foot, bicycle, rickshaw, or the universal shared taxi.

Not everything is efficient in the slums, though. In the Brazilian favelas where electricity is stolen and therefore free, people leave their lights on all day. But in most slums recycling is literally a way of life. The Dharavi slum in Mumbai has 400 recycling units and 30,000 ragpickers. Six thousand tons of rubbish are sorted every day. In 2007, the Economist reported that in Vietnam and Mozambique, “Waves of gleaners sift the sweepings of Hanoi’s streets, just as Mozambiquan children pick over the rubbish of Maputo’s main tip. Every city in Asia and Latin America has an industry based on gathering up old cardboard boxes.” . . .

In his 1985 article, Calthorpe made a statement that still jars with most people: “The city is the most environmentally benign form of human settlement. Each city dweller consumes less land, less energy, less water, and produces less pollution than his counterpart in settlements of lower densities.” “Green Manhattan” was the inflammatory title of a 2004 New Yorker article by David Owen. “By the most significant measures,” he wrote, “New York is the greenest community in the United States, and one of the greenest cities in the world . . . The key to New York’s relative environmental benignity is its extreme compactness. . . . Placing one and a half million people on a twenty-three-square-mile island sharply reduces their opportunities to be wasteful.” He went on to note that this very compactness forces people to live in the world’s most energy-efficient apartment buildings. . . .

Urban density allows half of humanity to live on 2.8 per cent of the land. . . . Consider just the infrastructure efficiencies. According to a 2004 UN report: “The concentration of population and enterprises in urban areas greatly reduces the unit cost of piped water, sewers, drains, roads, electricity, garbage collection, transport, health care, and schools.” . . .

[T]he nationally subsidised city of Manaus in northern Brazil “answers the question” of how to stop deforestation: give people decent jobs. Then they can afford houses, and gain security. One hundred thousand people who would otherwise be deforesting the jungle around Manaus are now prospering in town making such things as mobile phones and televisions. . . .

Of course, fast-growing cities are far from an unmitigated good. They concentrate crime, pollution, disease and injustice as much as business, innovation, education and entertainment. . . . But if they are overall a net good for those who move there, it is because cities offer more than just jobs. They are transformative: in the slums, as well as the office towers and leafy suburbs, the progress is from hick to metropolitan to cosmopolitan . . .

  1. CAT 2019 Question Paper - Slot 2

    Which one of the following statements would undermine the author’s stand regarding the greenness of cities?

    1. The compactness of big cities in the West increases the incidence of violent crime.
    2. Sorting through rubbish contributes to the rapid spread of diseases in the slums.
    3. The high density of cities leads to an increase in carbon dioxide and global warming.
    4. Over the last decade the cost of utilities has been increasing for city dwellers.
    Choice C
    The high density of cities leads to an increase in carbon dioxide and global warming.

  2. CAT 2019 Question Paper - Slot 2

    According to the passage, squatter cities are environment-friendly for all of the following reasons EXCEPT:

    1. their transportation is energy efficient.
    2. they recycle material.
    3. they sort out garbage.
    4. their streets are kept clean.
    Choice D
    their streets are kept clean.

  3. CAT 2019 Question Paper - Slot 2

    We can infer that Calthorpe’s statement “still jars” with most people because most people:

    1. regard cities as places of disease and crime.
    2. do not consider cities to be eco-friendly places.
    3. do not regard cities as good places to live in.
    4. consider cities to be very crowded and polluted.
    Choice B
    do not consider cities to be eco-friendly places.

  4. CAT 2019 Question Paper - Slot 2

    In the context of the passage, the author refers to Manaus in order to:

    1. explain how urban areas help the environment.
    2. describe the infrastructure efficiencies of living in a city.
    3. explain where cities source their labour for factories.
    4. promote cities as employment hubs for people.
    Choice A
    explain how urban areas help the environment.

  5. CAT 2019 Question Paper - Slot 2

    From the passage it can be inferred that cities are good places to live in for all of the following reasons EXCEPT that they:

    1. offer employment opportunities.
    2. help prevent destruction of the environment.
    3. contribute to the cultural transformation of residents.
    4. have suburban areas as well as office areas.
    Choice D
    have suburban areas as well as office areas.

Linguistics

For two years, I tracked down dozens of . . . Chinese in Upper Egypt [who were] selling lingerie. In a deeply conservative region, where Egyptian families rarely allow women to work or own businesses, the Chinese flourished because of their status as outsiders. They didn’t gossip, and they kept their opinions to themselves. In a New Yorker article entitled “Learning to Speak Lingerie,” I described the Chinese use of Arabic as another non-threatening characteristic. I wrote, “Unlike Mandarin, Arabic is inflected for gender, and Chinese dealers, who learn the language strictly by ear, often pick up speech patterns from female customers. I’ve come to think of it as the lingerie dialect, and there’s something disarming about these Chinese men speaking in the feminine voice.” . . .

When I wrote about the Chinese in the New Yorker, most readers seemed to appreciate the unusual perspective. But as I often find with topics that involve the Middle East, some people had trouble getting past the black-and-white quality of a byline. “This piece is so orientalist I don’t know what to do,” Aisha Gani, a reporter who worked at The Guardian, tweeted. Another colleague at the British paper, Iman Amrani, agreed: “I wouldn’t have minded an article on the subject written by an Egyptian woman—probably would have had better insight.” . . .

As an MOL (man of language), I also take issue with this kind of essentialism. Empathy and understanding are not inherited traits, and they are not strictly tied to gender and race. An individual who wrestles with a difficult language can learn to be more sympathetic to outsiders and open to different experiences of the world. This learning process—the embarrassments, the frustrations, the gradual sense of understanding and connection—is invariably transformative. In Upper Egypt, the Chinese experience of struggling to learn Arabic and local culture had made them much more thoughtful. In the same way, I was interested in their lives not because of some kind of voyeurism, but because I had also experienced Egypt and Arabic as an outsider. And both the Chinese and the Egyptians welcomed me because I spoke their languages. My identity as a white male was far less important than my ability to communicate.

And that easily lobbed word—“Orientalist”—hardly captures the complexity of our interactions. What exactly is the dynamic when a man from Missouri observes a Zhejiang native selling lingerie to an Upper Egyptian woman? . . . If all of us now stand beside the same river, speaking in ways we all understand, who’s looking east and who’s looking west? Which way is Oriental?

For all of our current interest in identity politics, there’s no corresponding sense of identity linguistics. You are what you speak—the words that run throughout your mind are at least as fundamental to your selfhood as is your ethnicity or your gender. And sometimes it’s healthy to consider human characteristics that are not inborn, rigid, and outwardly defined. After all, you can always learn another language and change who you are.

  1. CAT 2019 Question Paper - Slot 2

    Which of the following can be inferred from the author’s claim, “Which way is Oriental?”

    1. Learning another language can mitigate cultural hierarchies and barriers.
    2. Globalisation has mitigated cultural hierarchies and barriers.
    3. Goodwill alone mitigates cultural hierarchies and barriers.
    4. Orientalism is a discourse of the past, from colonial times, rarely visible today.
    Choice A
    Learning another language can mitigate cultural hierarchies and barriers.

  2. CAT 2019 Question Paper - Slot 2

    A French ethnographer decides to study the culture of a Nigerian tribe. Which of the following is most likely to be the view of the author of the passage?

    1. The author would encourage the ethnographer, but ask him/her to first learn the language of the Nigerian tribe s/he wishes to study.
    2. The author would encourage the ethnographer, but ask him/her to be mindful of his/her racial and gender identity in the process.
    3. The author would discourage the ethnographer from conducting the study as Nigerian ethnographers can better understand the tribe.
    4. The author would encourage the ethnographer and recommend him/her to hire a good translator for the purpose of holding interviews.
    Choice A
    The author would encourage the ethnographer, but ask him/her to first learn the language of the Nigerian tribe s/he wishes to study.

  3. CAT 2019 Question Paper - Slot 2

    The author’s critics would argue that:

    1. Empathy can overcome identity politics.
    2. Language is insufficient to bridge cultural barriers.
    3. Linguistic politics can be erased.
    4. Orientalism cannot be practiced by Egyptians.
    Choice B
    Language is insufficient to bridge cultural barriers.

  4. CAT 2019 Question Paper - Slot 2

    According to the passage, which of the following is not responsible for language’s ability to change us?

    1. The ups and downs involved in the course of learning a language.
    2. Language’s intrinsic connection to our notions of self and identity.
    3. Language’s ability to mediate the impact of identity markers one is born with.
    4. The twists and turns in the evolution of language over time.
    Choice D
    The twists and turns in the evolution of language over time.

British Colonial Policy

British colonial policy . . . went through two policy phases, or at least there were two strategies between which its policies actually oscillated, sometimes to its great advantage. At first, the new colonial apparatus exercised caution, and occupied India by a mix of military power and subtle diplomacy, the high ground in the middle of the circle of circles. This, however, pushed them into contradictions. For, whatever their sense of the strangeness of the country and the thinness of colonial presence, the British colonial state represented the great conquering discourse of Enlightenment rationalism, entering India precisely at the moment of its greatest unchecked arrogance. As inheritors and representatives of this discourse, which carried everything before it, this colonial state could hardly adopt for long such a self-denying attitude. It had restructured everything in Europe—the productive system, the political regimes, the moral and cognitive orders—and would do the same in India, particularly as some empirically inclined theorists of that generation considered the colonies a massive laboratory of utilitarian or other theoretical experiments. Consequently, the colonial state could not settle simply for eminence at the cost of its marginality; it began to take initiatives to introduce the logic of modernity into Indian society. But this modernity did not enter a passive society. Sometimes, its initiatives were resisted by pre-existing structural forms. At times, there was a more direct form of collective resistance. Therefore the map of continuity and discontinuity that this state left behind at the time of independence was rather complex and has to be traced with care.

Most significantly, of course, initiatives for . . . modernity came to assume an external character. The acceptance of modernity came to be connected, ineradicably, with subjection. This again points to two different problems, one theoretical, the other political. Theoretically, because modernity was externally introduced, it is explanatorily unhelpful to apply the logical format of the ‘transition process’ to this pattern of change. Such a logical format would be wrong on two counts. First, however subtly, it would imply that what was proposed to be built was something like European capitalism. (And, in any case, historians have forcefully argued that what it was to replace was not like feudalism, with or without modificatory adjectives.) But, more fundamentally, the logical structure of endogenous change does not apply here. Here transformation agendas attack as an external force. This externality is not something that can be casually mentioned and forgotten. It is inscribed on every move, every object, every proposal, every legislative act, each line of causality. It comes to be marked on the epoch itself. This repetitive emphasis on externality should not be seen as a nationalist initiative that is so well rehearsed in Indian social science. . . .

Quite apart from the externality of the entire historical proposal of modernity, some of its contents were remarkable. . . . Economic reforms, or rather alterations . . . did not foreshadow the construction of a classical capitalist economy, with its necessary emphasis on extractive and transport sectors. What happened was the creation of a degenerate version of capitalism—what early dependency theorists called the ‘development of underdevelopment’.

  1. CAT 2019 Question Paper - Slot 2

    All of the following statements about British colonialism can be inferred from the first paragraph, EXCEPT that it:

    1. was at least partly an outcome of Enlightenment rationalism.
    2. faced resistance from existing structural forms of Indian modernity.
    3. was at least partly shaped by the project of European modernity.
    4. allowed the treatment of colonies as experimental sites.
    Choice B
    faced resistance from existing structural forms of Indian modernity.

  2. CAT 2019 Question Paper - Slot 2

    All of the following statements, if true, could be seen as supporting the arguments in the passage, EXCEPT:

    1. the introduction of capitalism in India was not through the transformation of feudalism, as happened in Europe.
    2. modernity was imposed upon India by the British and, therefore, led to underdevelopment.
    3. throughout the history of colonial conquest, natives have often been experimented on by the colonisers.
    4. the change in British colonial policy was induced by resistance to modernity in Indian society.
    Choice D
    the change in British colonial policy was induced by resistance to modernity in Indian society.

  3. CAT 2019 Question Paper - Slot 2

    “Consequently, the colonial state could not settle simply for eminence at the cost of its marginality; it began to take initiatives to introduce the logic of modernity into Indian society.” Which of the following best captures the sense of this statement?

    1. The colonial state’s eminence was unsettled by its marginal position; therefore, it developed Indian society by modernising it.
    2. The colonial enterprise was a costly one; so to justify the cost it began to take initiatives to introduce the logic of modernity into Indian society.
    3. The colonial state felt marginalised from Indian society because of its own modernity; therefore, it sought to address that marginalisation by bringing its modernity to change Indian society.
    4. The cost of the colonial state’s eminence was not settled; therefore, it took the initiative of introducing modernity into Indian society.
    Choice C
    The colonial state felt marginalised from Indian society because of its own modernity; therefore, it sought to address that marginalisation by bringing its modernity to change Indian society.

  4. CAT 2019 Question Paper - Slot 2

    Which one of the following 5-word sequences best captures the flow of the arguments in the passage?

    1. Military power—arrogance—laboratory—modernity—capitalism.
    2. Colonial policy—Enlightenment—external modernity—subjection—underdevelopment.
    3. Colonial policy—arrogant rationality—resistance—independence—development.
    4. Military power—colonialism—restructuring—feudalism—capitalism.
    Choice B
    Colonial policy—Enlightenment—external modernity—subjection—underdevelopment.

  5. CAT 2019 Question Paper - Slot 2

    Which of the following observations is a valid conclusion to draw from the author’s statement that “the logical structure of endogenous change does not apply here. Here transformation agendas attack as an external force”?

    1. The endogenous logic of colonialism can only bring change if it attacks and transforms external forces.
    2. Indian society is not endogamous; it is more accurately characterised as aggressively exogamous.
    3. Colonised societies cannot be changed through logic; they need to be transformed with external force.
    4. The transformation of Indian society did not happen organically, but was forced by colonial agendas.
    Choice D
    The transformation of Indian society did not happen organically, but was forced by colonial agendas.

CAT VARC : CAT 2018 Question Paper Slot 1

Plastic Pollution

The only thing worse than being lied to is not knowing you’re being lied to. It’s true that plastic pollution is a huge problem, of planetary proportions. And it’s true we could all do more to reduce our plastic footprint. The lie is that blame for the plastic problem is wasteful consumers and that changing our individual habits will fix it.

Recycling plastic is to saving the Earth what hammering a nail is to halting a falling skyscraper. You struggle to find a place to do it and feel pleased when you succeed. But your effort is wholly inadequate and distracts from the real problem of why the building is collapsing in the first place. The real problem is that single-use plastic—the very idea of producing plastic items like grocery bags, which we use for an average of 12 minutes but can persist in the environment for half a millennium—is an incredibly reckless abuse of technology. Encouraging individuals to recycle more will never solve the problem of a massive production of single-use plastic that should have been avoided in the first place.

As an ecologist and evolutionary biologist, I have had a disturbing window into the accumulating literature on the hazards of plastic pollution. Scientists have long recognized that plastics biodegrade slowly, if at all, and pose multiple threats to wildlife through entanglement and consumption. More recent reports highlight dangers posed by absorption of toxic chemicals in the water and by plastic odors that mimic some species’ natural food. Plastics also accumulate up the food chain, and studies now show that we are likely ingesting it ourselves in seafood. . . .

Beginning in the 1950s, big beverage companies like Coca-Cola and Anheuser-Busch, along with Phillip Morris and others, formed a non-profit called Keep America Beautiful. Its mission is/was to educate and encourage environmental stewardship in the public. . . . At face value, these efforts seem benevolent, but they obscure the real problem, which is the role that corporate polluters play in the plastic problem. This clever misdirection has led journalist and author Heather Rogers to describe Keep America Beautiful as the first corporate greenwashing front, as it has helped shift the public focus to consumer recycling behavior and actively thwarted legislation that would increase extended producer responsibility for waste management. . . . [T]he greatest success of Keep America Beautiful has been to shift the onus of environmental responsibility onto the public while simultaneously becoming a trusted name in the environmental movement. . . .

So what can we do to make responsible use of plastic a reality? First: reject the lie. Litterbugs are not responsible for the global ecological disaster of plastic. Humans can only function to the best of their abilities, given time, mental bandwidth and systemic constraints. Our huge problem with plastic is the result of a permissive legal framework that has allowed the uncontrolled rise of plastic pollution, despite clear evidence of the harm it causes to local communities and the world’s oceans. Recycling is also too hard in most parts of the U.S. and lacks the proper incentives to make it work well.

  1. CAT 2018 Question Paper - Slot 1

    Which of the following interventions would the author most strongly support:

    1. recycling all plastic debris in the seabed.
    2. having all consumers change their plastic consumption habits.
    3. completely banning all single-use plastic bags.
    4. passing regulations targeted at producers that generate plastic products.
    Choice D
    passing regulations targeted at producers that generate plastic products.

  2. CAT 2018 Question Paper - Slot 1

    The author lists all of the following as negative effects of the use of plastics EXCEPT the:

    1. slow pace of degradation or non-degradation of plastics in the environment.
    2. air pollution caused during the process of recycling plastics.
    3. poisonous chemicals released into the water and food we consume.
    4. adverse impacts on the digestive systems of animals exposed to plastic.
    Choice B
    air pollution caused during the process of recycling plastics.

  3. CAT 2018 Question Paper - Slot 1

    In the first paragraph, the author uses “lie” to refer to the:

    1. blame assigned to consumersfor indiscriminate use of plastics.
    2. understatement of the enormity of the plastics pollution problem.
    3. fact that people do not know they have been lied to.
    4. understatement of the effects of recycling plastics.
    Choice A
    blame assigned to consumersfor indiscriminate use of plastics.

  4. CAT 2018 Question Paper - Slot 1

    In the second paragraph, the phrase “what hammering a nail is to halting a falling skyscraper” means:

    1. relying on emerging technologies to mitigate the ill-effects of plastic pollution.
    2. encouraging the responsible production of plastics by firms.
    3. focusing on consumer behaviour to tackle the problem of plastics pollution.
    4. focusing on single-use plastic bags to reduce the plastics footprint.
    Choice C
    focusing on consumer behaviour to tackle the problem of plastics pollution.

  5. CAT 2018 Question Paper - Slot 1

    It can be inferred that the author considers the Keep America Beautiful organisation:

    1. a "greenwash" because it was a benevolent attempt to improve public recycling habits.
    2. a sham as it diverted attention away from the role of corporates in plastics pollution.
    3. an important step in sensitising producers to the need to tackle plastics pollution.
    4. an innovative example of a collaborative corporate social responsibility initiative.
    Choice B
    a sham as it diverted attention away from the role of corporates in plastics pollution.

Human-Elephant Conflict

“Everybody pretty much agrees that the relationship between elephants and people has dramatically changed,” [says psychologist Gay] Bradshaw. “Where for centuries humans and elephants lived in relatively peaceful coexistence, there is now hostility and violence. Now, I use the term ‘violence’ because of the intentionality associated with it, both in the aggression of humans and, at times, the recently observed behavior of elephants.”

Typically, elephant researchers have cited, as a cause of aggression, the high levels of testosterone in newly matured male elephants or the competition for land and resources between elephants and humans. But, Bradshaw and several colleagues argue that today’s elephant populations are suffering from a form of chronic stress, a kind of species-wide trauma. Decades of poaching and culling and habitat loss, they claim, have so disrupted the intricate web of familial and societal relations by which young elephants have traditionally been raised in the wild, and by which established elephant herds are governed, that what we are now witnessing is nothing less than a precipitous collapse of elephant culture.

Elephants, when left to their own devices, are profoundly social creatures. young elephants are raised within an extended, multi-tiered network of doting female caregivers that includes the birth mother, grandmothers, aunts and friends. These relations are maintained over a life span as long as 70 years. Studies of established herds have shown that young elephants stay within 15 feet of their mothers for nearly all of their first eight years of life, after which young females are socialized into the matriarchal network, while young males go off for a time into an all-male social group before coming back into the fold as mature adults.

This fabric of elephant society, Bradshaw and her colleagues [demonstrate], ha[s] effectively been frayed by years of habitat loss and poaching, along with systematic culling by government agencies to control elephant numbers and translocations of herds to different habitats. As a result of such social upheaval, calves are now being born to and raised by ever younger and inexperienced mothers. Young orphaned elephants, meanwhile, that have witnessed the death of a parent at the hands of poachers are coming of age in the absence of the support system that defines traditional elephant life. “The loss of elephant elders,” [says] Bradshaw "and the traumatic experience of witnessing the massacres of their family, impairs normal brain and behavior development in young elephants.”

What Bradshaw and her colleagues describe would seem to be an extreme form of anthropocentric conjecture if the evidence that they’ve compiled from various elephant researchers weren’t so compelling. The elephants of decimated herds, especially orphans who’ve watched the death of their parents and elders from poaching and culling, exhibit behavior typically associated with post-traumatic stress disorder and other trauma-related disorders in humans: abnormal startle response, unpredictable asocial behavior, inattentive mothering and hyper-aggression.

[According to Bradshaw], “Elephants are suffering and behaving in the same ways that we recognize in ourselves as a result of violence. Except perhaps for a few specific features, brain organization and early development of elephants and humans are extremely similar.”

  1. CAT 2018 Question Paper - Slot 1

    Which of the following statements best expresses the overall argument of this passage?

    1. Elephants, like the humans they are in conflict with, are profoundly social creatures.
    2. The relationship between elephants and humans has changed from one of coexistence to one of hostility
    3. Recent elephant behaviour could be understood as a form of species-wide trauma-related response.
    4. The brain organisation and early development of Aelephants and humans are extremely similar
    Choice C
    Recent elephant behaviour could be understood as a form of species-wide trauma-related response.

  2. CAT 2018 Question Paper - Slot 1

    In the first paragraph, Bradshaw uses the term "violence" to describe the recent change in the human-elephant relationship because, according to him:

    1. Both humans and elephants have killed members of each other's species.
    2. There is a purposefulness in human and elephant aggression towards each other.
    3. Human-elephant interactions have changed their character over time.
    4. Elephant herds and their habitat have been systematically destroyed by humans.
    Choice B
    There is a purposefulness in human and elephant aggression towards each other.

  3. CAT 2018 Question Paper - Slot 1

    The passage makes all of the following claims EXCEPT

    1. Elephant mothers are evolving newer ways of rearing their calves to adapt to emerging threats.
    2. The elephant response to deeply disturbing experiences is similar to that of humans.
    3. Elephants establish e[tended and enduring familial relationships as do humans.
    4. Human actions such as poaching and culling have created stressful conditions for elephant communities.
    Choice A
    Elephant mothers are evolving newer ways of rearing their calves to adapt to emerging threats.

  4. CAT 2018 Question Paper - Slot 1

    Which of the following measures is Bradshaw most likely to support to address the problem of elephant aggression?

    1. The development of treatment programmes for elephants drawing on insights gained from treating post-traumatic stress disorder in humans.
    2. Funding of more studies to better understand the impact of testosterone on male elephant aggression.
    3. Studying the impact of isolating elephant calves on their early brain development, behaviour and aggression.
    4. Increased funding for research into the similarity of humans and other animals drawing on insights gained from human-elephant similarities.
    Choice A
    The development of treatment programmes for elephants drawing on insights gained from treating post-traumatic stress disorder in humans.

  5. CAT 2018 Question Paper - Slot 1

    In paragraph 4, the phrase, “The fabric of elephant society . . . has(s) effectively been frayed by . . .” is:

    1. an exaggeration aimed at bolstering Bradshaw's claims.
    2. an accurate description of the condition of elephant herds today.
    3. an ode to the fragility of elephant society today.
    4. a metaphor for the effect of human activity on elephant communities.
    Choice D
    a metaphor for the effect of human activity on elephant communities.

India and the World War

The Indian government has announced an international competition to design a National War Memorial in New Delhi, to honour all of the Indian soldiers who served in the various wars and counter-insurgency campaigns from 1947 onwards. The terms of the competition also specified that the new structure would be built adjacent to the India Gate – a memorial to the Indian soldiers who died in the First World War. Between the old imperialist memorial and the proposed nationalist one, India’s contribution to the Second World War is airbrushed out of existence.

The Indian government’s conception of the war memorial was not merely absentminded. Rather, it accurately reflected the fact that both academic history and popular memory have yet to come to terms with India’s Second World War, which continues to be seen as little more than mood music in the drama of India’s advance towards independence and partition in 1947. Further, the political trajectory of the postwar subcontinent has militated against popular remembrance of the war. With partition and the onset of the India-Pakistan rivalry, both of the new nations needed fresh stories for self-legitimisation rather than focusing on shared wartime experiences.

However, the Second World War played a crucial role in both the independence and partition of India. The Indian army recruited, trained and deployed some 2.5 million men, almost 90,000 of which were killed and many more injured. Even at the time, it was recognised as the largest volunteer force in the war.

India’s material and financial contribution to the war was equally significant. India emerged as a major military-industrial and logistical base for Allied operations in south-east Asia and the Middle East. This led the United States to take considerable interest in the country’s future, and ensured that this was no longer the preserve of the British government. Other wartime developments pointed in the direction of India’s independence. In a stunning reversal of its long-standing financial relationship with Britain, India finished the war as one of the largest creditors to the imperial power.

Such extraordinary mobilization for war was achieved at great human cost, with the Bengal famine the most extreme manifestation of widespread wartime deprivation. The costs on India’s home front must be counted in millions of lives.

Indians signed up to serve on the war and home fronts for a variety of reasons. Many were convinced that their contribution would open the doors to India’s freedom. The political and social churn triggered by the war was evident in the massive waves of popular protest and unrest that washed over rural and urban India in the aftermath of the conflict. This turmoil was crucial in persuading the Attlee government to rid itself of the incubus of ruling India. Seventy years on, it is time that India engaged with the complex legacies of the Second World War. Bringing the war into the ambit of the new national memorial would be a fitting – if not overdue – recognition that this was India’s War.

  1. CAT 2018 Question Paper - Slot 1

    In the first paragraph, the author laments the fact that

    1. the new war memorial will be built right next to India Gate.
    2. there is no recognition of the Indian soldiers who served in the Second World War.
    3. funds will be wasted on another war memorial when we already have the India Gate memorial.
    4. India lost thousands of human lives during the Second World War
    Choice B
    there is no recognition of the Indian soldiers who served in the Second World War.

  2. CAT 2018 Question Paper - Slot 1

    The author lists all of the following as outcomes of the Second World War EXCEPT:

    1. US recognition of India's strategic location and role in the war.
    2. Large-scale deaths in Bengal as a result of deprivation and famine.
    3. Independence of the subcontinent and its partition into two countries.
    4. The large financial debt India owed to Britain after the war.
    Choice D
    The large financial debt India owed to Britain after the war.

  3. CAT 2018 Question Paper - Slot 1

    The phrase “mood music” is used in the second paragraph to indicate that the Second World War is viewed as:

    1. Setting the stage for the emergence of the India–Pakistan rivalry in the subcontinent.
    2. a backdrop to the subsequent independence and partition of the region.
    3. a part of the narrative on the ill-effects of colonial rule on India.
    4. a tragic period in terms of loss of lives and national wealth.
    Choice B
    a backdrop to the subsequent independence and partition of the region.

  4. CAT 2018 Question Paper - Slot 1

    The author suggests that a major reason why India has not so far acknowledged its role in the Second World War is that it:

    1. wants to forget the human and financial toll of the War on the country
    2. has been focused on building an independent, non-colonial political identity.
    3. views the War as a predominantly Allied effort, with India playing only a supporting role.
    4. blames the War for leading to the momentous partition of the country.
    Choice B
    has been focused on building an independent, non-colonial political identity.

  5. CAT 2018 Question Paper - Slot 1

    The author claims that omitting mention of Indians who served in the Second World War from the new National War Memorial is:

    1. a reflection of misplaced priorities of the post-independence Indian governments.
    2. a reflection of the academic and popular view of India’s role in the War.
    3. appropriate as their names can always be included in the India Gate memorial.
    4. is something which can be rectified in future by constructing a separate memorial.
    Choice B
    a reflection of the academic and popular view of India’s role in the War

Economy and Happiness

Economists have spent most of the 20th century ignoring psychology, positive or otherwise. But today there is a great deal of emphasis on how happiness can shape global economies, or — on a smaller scale — successful business practice. This is driven, in part, by a trend in "measuring" positive emotions, mostly so they can be optimized. Neuroscientists, for example, claim to be able to locate specific emotions, such as happiness or disappointment, in particular areas of the brain. Wearable technologies, such as Spire, offer data-driven advice on how to reduce stress.

We are no longer just dealing with "happiness" in a philosophical or romantic sense — it has become something that can be monitored and measured, including by our behavior, use of social media and bodily indicators such as pulse rate and facial expressions. There is nothing automatically sinister about this trend. But it is disquieting that the businesses and experts driving the quantification of happiness claim to have our best interests at heart, often concealing their own agendas in the process. In the workplace, happy workers are viewed as a "win-win." Work becomes more pleasant, and employees, more productive. But this is now being pursued through the use of performance-evaluating wearable technology, such as Humanyze or Virgin Pulse, both of which monitor physical signs of stress and activity toward the goal of increasing productivity.

Cities such as Dubai, which has pledged to become the "happiest city in the world," dream up ever-more elaborate and intrusive ways of collecting data on well-being — to the point where there is now talk of using CCTV cameras to monitor facial expressions in public spaces. New ways of detecting emotions are hitting the market all the time: One company, Beyond Verbal, aims to calculate moods conveyed in a phone conversation, potentially without the knowledge of at least one of the participants. And Facebook [has] demonstrated that it could influence our emotions through tweaking our news feeds — opening the door to ever-more targeted manipulation in advertising and influence.

As the science grows more sophisticated and technologies become more intimate with our thoughts and bodies, a clear trend is emerging. Where happiness indicators were once used as a basis to reform society, challenging the obsession with money that G.D.P. measurement entrenches, they are increasingly used as a basis to transform or discipline individuals.

Happiness becomes a personal project, that each of us must now work on, like going to the gym. Since the 1970s, depression has come to be viewed as a cognitive or neurological defect in the individual, and never a consequence of circumstances. All of this simply escalates the sense of responsibility each of us feels for our own feelings, and with it, the sense of failure when things go badly. A society that deliberately removed certain sources of misery, such as precarious and exploitative employment, may well be a happier one. But we won't get there by making this single, often fleeting emotion, the over-arching goal.

  1. CAT 2018 Question Paper - Slot 1

    According to the author, wearable technologies and social media are contributing most to:

    1. happiness as a “personal project”.
    2. depression as a thing of the past.
    3. disciplining individualsto be happy.
    4. making individuals aware of stress in their lives.
    Choice C
    disciplining individualsto be happy.

  2. CAT 2018 Question Paper - Slot 1

    The author’s view would be undermined by which of the following research findings?

    1. There is a definitive move towards the adoption of wearable technology that taps into emotions.
    2. Stakeholders globally are moving away from collecting data on the well-being of individuals.
    3. A proliferation of gyms that are collecting data on customer well-being.
    4. Individuals worldwide are utilising technologies to monitor and increase their wellbeing.
    Choice B
    Stakeholders globally are moving away from collecting data on the well-being of individuals.

  3. CAT 2018 Question Paper - Slot 1

    According to the author, Dubai:

    1. is on its way to becoming one of the world’s happiest cities.
    2. collaborates with Facebook to selectively influence its inhabitants’ moods.
    3. develops sophisticated technologies to monitor its inhabitants’ states of mind.
    4. incentivises companies that prioritise worker welfare.
    Choice C
    develops sophisticated technologies to monitor its inhabitants’ states of mind.

  4. CAT 2018 Question Paper - Slot 1

    In the author's opinion, the shift in thinking in the 1970s:

    1. was a welcome change from the earlier view that depression could be cured by changing circumstances.
    2. introduced greater stress into people’s lives as they were expected to be responsible for their own happiness.
    3. put people in touch with their own feelings rather than depending on psychologists.
    4. reflected the emergence of neuroscience as the authority on human emotions.
    Choice B
    introduced greater stress into people’s lives as they were expected to be responsible for their own happiness.

  5. CAT 2018 Question Paper - Slot 1

    From the passage we can infer that the author would like economists to:

    1. incorporate psychological findings into their research cautiously
    2. correlate measurements of happiness with economic indicators
    3. measure the effectiveness of Facebook and social media advertising
    4. work closely with neuroscientists to understand human behaviour.
    Choice A
    incorporate psychological findings into their research cautiously

Acquired Characterisitcs

When researchers at Emory University in Atlanta trained mice to fear the smell of almonds (by pairing it with electric shocks), they found, to their consternation, that both the children and grandchildren of these mice were spontaneously afraid of the same smell. That is not supposed to happen. Generations of schoolchildren have been taught that the inheritance of acquired characteristics is impossible. A mouse should not be born with something its parents have learned during their lifetimes, any more than a mouse that loses its tail in an accident should give birth to tailless mice.

Modern evolutionary biology dates back to a synthesis that emerged around the 1940s60s, which married Charles Darwin’s mechanism of natural selection with Gregor Mendel’s discoveries of how genes are inherited. The traditional, and still dominant, view is that adaptations – from the human brain to the peacock’s tail – are fully and satisfactorily explained by natural selection (and subsequent inheritance). Yet [new evidence] from genomics, epigenetics and developmental biology [indicates] that evolution is more complex than we once assumed.

In his book On Human Nature (1978), the evolutionary biologist Edward O Wilson claimed that human culture is held on a genetic leash. The metaphor [needs revision]. Imagine a dogwalker (the genes) struggling to retain control of a brawny mastiff (human culture). The pair’s trajectory (the pathway of evolution) reflects the outcome of the struggle. Now imagine the same dog-walker struggling with multiple dogs, on leashes of varied lengths, with each dog tugging in different directions. All these tugs represent the influence of developmental factors, including epigenetics, antibodies and hormones passed on by parents, as well as the ecological legacies and culture they bequeath.

The received wisdom is that parental experiences can’t affect the characters of their offspring. Except they do. The way that genes are expressed to produce an organism’s phenotype– the actual characteristics it ends up with – is affected by chemicals that attach to them. Everything from diet to air pollution to parental behaviour can influence the addition or removal of these chemical marks, which switches genes on or off. Usually these so-called ‘epigenetic’ attachments are removed during the production of sperm and eggs cells, but it turns out that some escape the resetting process and are passed on to the next generation, along with the genes. This is known as ‘epigenetic inheritance’, and more and more studies are confirming that it really happens. Let’s return to the almond-fearing mice. The inheritance of an epigenetic mark transmitted in the sperm is what led the mice’s offspring to acquire an inherited fear.

Epigenetics is only part of the story. Through culture and society, [humans and other animals] inherit knowledge and skills acquired by [their] parents. All this complexity points to an evolutionary process in which genomes (over hundreds to thousands of generations), epigenetic modifications and inherited cultural factors (over several, perhaps tens or hundreds of generations), and parental effects (over single-generation timespans) collectively informb how organisms adapt. These extra-genetic kinds of inheritance give organisms the flexibility to make rapid adjustments to environmental challenges, dragging genetic change in their wake – much like a rowdy pack of dogs.

  1. CAT 2018 Question Paper - Slot 1

    The passage uses the metaphor of a dog walker to argue that evolutionary adaptation is most comprehensively understood as being determined by:

    1. genetic, epigenetic, developmental factors, and ecological legacies.
    2. socio-cultural, genetic, epigenetic, and genomic legacies.
    3. ecological, hormonal, extra genetic and genetic legacies.
    4. extra genetic, genetic, epigenetic and genomic legacies.
    Choice A
    genetic, epigenetic, developmental factors, and ecological legacies.

  2. CAT 2018 Question Paper - Slot 1

    Which of the following options best describes the author's argument?

    1. Darwin’s theory of natural selection cannot fully explain evolution.
    2. Mendel’s theory of inheritance is unfairly underestimated in explaining evolution.
    3. Darwin’s and Mendel’s theories together best explain evolution.
    4. Wilson’s theory of evolution is scientifically superior to either Darwin’s or Mendel’s.
    Choice A
    Darwin’s theory of natural selection cannot fully explain evolution.

  3. CAT 2018 Question Paper - Slot 1

    The Emory University experiment with mice points to the inheritance of:

    1. acquired characteristics
    2. psychological markers
    3. personality traits
    4. acquired parental fears
    Choice A
    acquired characteristics

  4. CAT 2018 Question Paper - Slot 1

    Which of the following, if found to be true, would negate the main message of the passage?

    1. A study indicating the primacy of ecological impact on human adaptation.
    2. A study highlighting the criticality of epigenetic inheritance to evolution.
    3. A study affirming the sole influence of natural selection and inheritance on evolution.
    4. A study affirming the influence of socio-cultural markers on evolutionary processes.
    Choice C
    A study affirming the sole influence of natural selection and inheritance on evolution.

CAT VARC : CAT 2018 Question Paper Slot 2

E-Governance

Will a day come when India’s poor can access government services as easily as drawing cash from an ATM? No country in the world has made accessing education or health or policing or dispute resolution as easy as an ATM, because the nature of these activities requires individuals to use their discretion in a positive way. Technology can certainly facilitate this in a variety of ways if it is seen as one part of an overall approach, but the evidence so far in education, for instance, is that just adding computers alone doesn’t make education any better.

The dangerous illusion of technology is that it can create stronger, top down accountability of service providers in implementation-intensive services within existing public sector organisations. One notion is that electronic management information systems (EMIS) keep better track of inputs and those aspects of personnel that are ‘EMIS visible’ can lead to better services. A recent study examined attempts to increase attendance of Auxiliary Nurse Midwife (ANMs) at clinics in Rajasthan, which involved high-tech time clocks to monitor attendance. The study’s title says it all: Band-Aids on a Corpse. E-governance can be just as bad as any other governance when the real issue is people and their motivation.

For services to improve, the people providing the services have to want to do a better job with the skills they have. A study of medical care in Delhi found that even though providers, in the public sector had much better skills than private sector providers their provision of care in actual practice was much worse.

In implementation-intensive services the key to success is face-to-face interactions between a teacher, a nurse, a policeman, an extension agent and a citizen. This relationship is about power. Amartya Sen’s report on education in West Bengal had a supremely telling anecdote in which the villagers forced the teacher to attend school, but then, when the parents went off to work, the teacher did not teach, but forced the children to massage his feet. As long as the system empowers providers over citizens, technology is irrelevant.

The answer to successfully providing basic services is to create systems that provide both autonomy and accountability. In basic education for instance, the answer to poor teaching is not controlling teachers more. The key is to hire teachers who want to teach and let them teach, expressing their professionalism and vocation as a teacher through autonomy in the classroom. This autonomy has to be matched with accountability for results—not just narrowly measured through test scores, but broadly for the quality of the education they provide.

A recent study in Uttar Pradesh showed that if, somehow, all civil service teachers could be replaced with contract teachers, the state could save a billion dollars a year in revenue and double student learning. Just the additional autonomy and accountability of contracts through local groups—even without complementary system changes in information and empowerment—led to that much improvement. The first step to being part of the solution is to create performance information accessible to those outside of the government.

  1. CAT 2018 Question Paper - Slot 2

    In the context of the passage, we can infer that the title “Band Aids on a Corpse” (in paragraph 2) suggests that:

    1. the nurses who attended the clinics were too poorly trained to provide appropriate medical care.
    2. the electronic monitoring system was a superficial solution to a serious problem.
    3. the nurses attended the clinics, but the clinics were ill-equipped.
    4. the clinics were better funded, but performance monitoring did not result in any improvement.
    Choice B
    the electronic monitoring system was a superficial solution to a serious problem.

  2. CAT 2018 Question Paper - Slot 2

    According to the author, service delivery in Indian education can be improved in all of the following ways EXCEPT through:

    1. use of technology.
    2. recruitment of motivated teachers.
    3. access to information on the quality of teaching.
    4. elimination of government involvement.
    Choice D
    elimination of government involvement.

  3. CAT 2018 Question Paper - Slot 2

    Which of the following, IF TRUE, would undermine the passage’s main argument?

    1. If absolute instead of moderate technological surveillance is exercised over the performance of service providers.
    2. Empowerment of service providers leads to increased complacency and rigged performance results.
    3. If it were proven that increase in autonomy of service providers leads to an exponential increase in their work ethic and sense of responsibility.
    4. If it were proven that service providers in the private sector have better skills than those in the public sector.
    Choice B
    Empowerment of service providers leads to increased complacency and rigged performance results.

  4. CAT 2018 Question Paper - Slot 2

    The author questions the use of monitoring systems in services that involve face-to-face interaction between service providers and clients because such systems:

    1. do not improve services that need committed service providers.
    2. are not as effective in the public sector as they are in the private sector.
    3. improve the skills but do not increase the motivation of service providers
    4. are ineffective because they are managed by the government.
    Choice A
    do not improve services that need committed service providers.

  5. CAT 2018 Question Paper - Slot 2

    The main purpose of the passage is to:

    1. critique the government’s involvement in educational activities and other implementation-intensive services.
    2. argue that some types of services can be improved by providing independence and requiring accountability.
    3. analyse the shortcomings of government-appointed nurses and their management through technology.
    4. find a solution to the problem of poor service delivery in education by examining different strategies.
    Choice B
    argue that some types of services can be improved by providing independence and requiring accountability.

White-lipped Grove Snails

Grove snails as a whole are distributed all over Europe, but a specific variety of the snail, with a distinctive white-lipped shell, is found exclusively in Ireland and in the Pyrenees mountains that lie on the border between France and Spain. The researchers sampled a total of 423 snail specimens from 36 sites distributed across Europe, with an emphasis on gathering large numbers of the white-lipped variety. When they sequenced genes from the mitochondrial DNA of each of these snails and used algorithms to analyze the genetic diversity between them, they found that a distinct lineage (the snails with the white-lipped shells) was indeed endemic to the two very specific and distant places in question.

Explaining this is tricky. Previously, some had speculated that the strange distributions of creatures such as the white-lipped grove snails could be explained by convergent evolution—in which two populations evolve the same trait by coincidence—but the underlying genetic similarities between the two groups rules that out. Alternately, some scientists had suggested that the white-lipped variety had simply spread over the whole continent, then been wiped out everywhere besides Ireland and the Pyrenees, but the researchers say their sampling and subsequent DNA analysis eliminate that possibility too.

“If the snails naturally colonized Ireland, you would expect to find some of the same genetic type in other areas of Europe, especially Britain. We just don’t find them,” Davidson, the lead author, said in a press statement.

Moreover, if they’d gradually spread across the continent, there would be some genetic variation within the white-lipped type, because evolution would introduce variety over the thousands of years it would have taken them to spread from the Pyrenees to Ireland. That variation doesn’t exist, at least in the genes sampled. This means that rather than the organism gradually expanding its range, large populations instead were somehow moved en mass to the other location within the space of a few dozen generations, ensuring a lack of genetic variety.

“There is a very clear pattern, which is difficult to explain except by involving humans,” Davidson said. Humans, after all, colonized Ireland roughly 9,000 years ago, and the oldest fossil evidence of grove snails in Ireland dates to roughly the same era. Additionally, there is archaeological evidence of early sea trade between the ancient peoples of Spain and Ireland via the Atlantic and even evidence that humans routinely ate these types of snails before the advent of agriculture, as their burnt shells have been found in Stone Age trash heaps.

The simplest explanation, then? Boats. These snails may have inadvertently traveled on the floor of the small, coast-hugging skiffs these early humans used for travel, or they may have been intentionally carried to Ireland by the seafarers as a food source. “The highways of the past were rivers and the ocean–as the river that flanks the Pyrenees was an ancient trade route to the Atlantic, what we’re actually seeing might be the long lasting legacy of snails that hitched a ride as humans travelled from the South of France to Ireland 8,000 years ago,” Davidson said.

  1. CAT 2018 Question Paper - Slot 2

    All of the following evidence supports the passage’s explanation of sea travel/trade EXCEPT:

    1. the coincidental existence of similar traits in the white-lipped grove snails of Ireland and the Pyrenees because of convergent evolution.
    2. the oldest fossil evidence of white-lipped grove snails in Ireland dates back to roughly 9,000 years ago, the time when humans colonised Ireland.
    3. absence of genetic variation within the white-lipped grove snails of Ireland and the Pyrenees, whose genes were sampled.
    4. archaeological evidence of early sea trade between the ancient peoples of Spain and Ireland via the Atlantic Ocean.
    Choice A
    the coincidental existence of similar traits in the white-lipped grove snails of Ireland and the Pyrenees because of convergent evolution.

  2. CAT 2018 Question Paper - Slot 2

    The passage outlines several hypotheses and evidence related to white-lipped grove snails to arrive at the most convincing explanation for:

    1. why the white-lipped variety of grove snails were wiped out everywhere except in Ireland and the Pyrenees.
    2. how the white-lipped variety of grove snails independently evolved in Ireland and the Pyrenees.
    3. why the white-lipped variety of grove snails are found only in Ireland and the Pyrenees.
    4. how the white-lipped variety of grove snails independently evolved in Ireland and the Pyrenees.
    Choice C
    why the white-lipped variety of grove snails are found only in Ireland and the Pyrenees.

  3. CAT 2018 Question Paper - Slot 2

    Which one of the following makes the author eliminate convergent evolution as a probable explanation for why white-lipped grove snails are found in Ireland and the Pyrenees?

    1. The absence of genetic variation between white-lipped grove snails of Ireland and the Pyrenees.
    2. The absence of genetic similarities between white-lipped grove snails of Ireland and snails from other parts of Europe, especially Britain.
    3. The coincidental evolution of similar traits (white-lipped shell) in the grove snails of Ireland and the Pyrenees.
    4. The distinct lineage of white-lipped grove snails found specifically in Ireland and the Pyrenees.
    Choice A
    The absence of genetic variation between white-lipped grove snails of Ireland and the Pyrenees.

  4. CAT 2018 Question Paper - Slot 2

    In paragraph 4, the evidence that “humans routinely ate these types of snails before the advent of agriculture” can be used to conclude that:

    1. white-lipped grove snails may have inadvertently traveled from the Pyrenees to Ireland on the floor of the small, coast-hugging skiffs that early seafarers used for travel.
    2. the seafarers who traveled from the Pyrenees to Ireland might have carried white-lipped grove snails with them as edibles.
    3. rivers and oceans in the Stone Age facilitated trade in white-lipped grove snails.
    4. 9,000 years ago, during the Stone Age, humans traveled from the South of France to Ireland via the Atlantic Ocean.
    Choice B
    the seafarers who traveled from the Pyrenees to Ireland might have carried white-lipped grove snails with them as edibles.

Meritocracy

The complexity of modern problems often precludes any one person from fully understanding them. Factors contributing to rising obesity levels, for example, include transportation systems and infrastructure, media, convenience foods, changing social norms, human biology and psychological factors. The multidimensional or layered character of complex problems also undermines the principle of meritocracy: the idea that the ‘best person’ should be hired. There is no best person. When putting together an oncological research team, a biotech company such as Gilead or Genentech would not construct a multiple-choice test and hire the top scorers, or hire people whose resumes score highest according to some performance criteria. Instead, they would seek diversity. They would build a team of people who bring diverse knowledge bases, tools and analytic skills.

Believers in a meritocracy might grant that teams ought to be diverse but then argue that meritocratic principles should apply within each category. Thus the team should consist of the ‘best’ mathematicians, the ‘best’ oncologists, and the ‘best’ biostatisticians from within the pool. That position suffers from a similar flaw.

Even with a knowledge domain, no test or criteria applied to individuals will produce the best team. Each of these domains possesses such depth and breadth, that no test can exist. Consider the field of neuroscience. Upwards of 50,000 papers were published last year covering various techniques, domains of enquiry and levels of analysis, ranging from molecules and synapses up through networks of neurons. Given that complexity, any attempt to rank a collection of neuroscientists from best to worst, as if they were competitors in the 50-metre butterfly, must fail. What could be true is that given a specific task and the composition of a particular team, one scientist would be more likely to contribute than another. Optimal hiring depends on context. Optimal teams will be diverse.

Evidence for this claim can be seen in the way that papers and patents that combine diverse ideas tend to rank as high-impact. It can also be found in the structure of the so-called random decision forest, a state-of-the-art machine-learning algorithm.

Random forests consist of ensembles of decision trees. If classifying pictures, each tree makes a vote: is that a picture of a fox or a dog? A weighted majority rules. Random forests can serve many ends. They can identify bank fraud and diseases, recommend ceiling fans and predict online dating behaviour. When building a forest, you do not select the best trees as they tend to make similar classifications. You want diversity. Programmers achieve that diversity by training each tree on different data, a technique known as bagging. They also boost the forest ‘cognitively’ by training trees on the hardest cases – those that the current forest gets wrong. This ensures even more diversity and accurate forests."

Yet the fallacy of meritocracy persists. Corporations, non-profits, governments, universities and even preschools test, score and hire the ‘best’. This all but guarantees not creating the best team. Ranking people by common criteria produces homogeneity. That’s not likely to lead to breakthroughs.

  1. CAT 2018 Question Paper - Slot 2

    The author critiques meritocracy for all the following reasons EXCEPT that:

    1. modern problems are multifaceted and require varied skill-sets to be solved.
    2. diversity and context-specificity are important for making major advances in any field.
    3. criteria designed to assess merit are insufficient to test expertise in any field of knowledge.
    4. an ideal team comprises of best individuals from diverse fields of knowledge.
    Choice D
    an ideal team comprises of best individuals from diverse fields of knowledge.

  2. CAT 2018 Question Paper - Slot 2

    Which of the following conditions would weaken the efficacy of a random decision forest?

    1. If a large number of decision trees in the ensemble were trained on data derived from easy cases.
    2. If a large number of decision trees in the ensemble were trained on data derived from easy and hard cases.
    3. If the types of ensembles of decision trees in the forest were doubled.
    4. If the types of decision trees in each ensemble of the forest were doubled.
    Choice A
    If a large number of decision trees in the ensemble were trained on data derived from easy cases.

  3. CAT 2018 Question Paper - Slot 2

    Which of the following conditions, if true, would invalidate the passage’s main argument?

    1. If assessment tests were made more extensive and rigorous.
    2. If top-scorers possessed multidisciplinary knowledge that enabled them to look at a problem from several perspectives.
    3. If it were proven that teams characterised by diversity end up being conflicted about problems and take a long time to arrive at a solution.
    4. If a new machine-learning algorithm were developed that proved to be more effective than the random decision forest.
    Choice B
    If top-scorers possessed multidisciplinary knowledge that enabled them to look at a problem from several perspectives.

  4. CAT 2018 Question Paper - Slot 2

    On the basis of the passage, which of the following teams is likely to be most effective in solving the problem of rising obesity levels?

    1. A team comprised of nutritionists, psychologists, urban planners and media personnel, who have each scored a distinction in their respective subject tests.
    2. A team comprised of nutritionists, psychologists, urban planners and media personnel, who have each performed well in their respective subject tests.
    3. A specialised team of nutritionists from various countries, who are also trained in the machine-learning algorithm of random decision forest.
    4. A specialised team of top nutritionists from various countries, who also possess some knowledge of psychology.
    Choice B
    A team comprised of nutritionists, psychologists, urban planners and media personnel, who have each performed well in their respective subject tests.

  5. CAT 2018 Question Paper - Slot 2

    Which of the following best describes the purpose of the example of neuroscience?

    1. In the modern age, every field of knowledge is so vast that a meaningful assessment of merit is impossible.
    2. Unlike other fields of knowledge, neuroscience is an exceptionally complex field, making a meaningful assessment of neuroscientists impossible.
    3. In narrow fields of knowledge, a meaningful assessment of expertise has always been possible.
    4. Neuroscience is an advanced field of science because of its connections with other branches of science like oncology and biostatistics.
    Choice A
    In the modern age, every field of knowledge is so vast that a meaningful assessment of merit is impossible.

Metric fixation

More and more companies, government agencies, educational institutions and philanthropic organisations are today in the grip of a new phenomenon: ‘metric fixation’. The key components of metric fixation are the belief that it is possible – and desirable – to replace professional judgment (acquired through personal experience and talent) with numerical indicators of comparative performance based upon standardised data (metrics); and that the best way to motivate people within these organisations is by attaching rewards and penalties to their measured performance.

The rewards can be monetary, in the form of pay for performance, say, or reputational, in the form of college rankings, hospital ratings, surgical report cards and so on. But the most dramatic negative effect of metric fixation is its propensity to incentivise gaming: that is, encouraging professionals to maximise the metrics in ways that are at odds with the larger purpose of the organisation. If the rate of major crimes in a district becomes the metric according to which police officers are promoted, then some officers will respond by simply not recording crimes or downgrading them from major offences to misdemeanours. Or take the case of surgeons. When the metrics of success and failure are made public – affecting their reputation and income – some surgeons will improve their metric scores by refusing to operate on patients with more complex problems, whose surgical outcomes are more likely to be negative. Who suffers? The patients who don’t get operated upon.

When reward is tied to measured performance, metric fixation invites just this sort of gaming. But metric fixation also leads to a variety of more subtle unintended negative consequences. These include goal displacement, which comes in many varieties: when performance is judged by a few measures, and the stakes are high (keeping one’s job, getting a pay rise or raising the stock price at the time that stock options are vested), people focus on satisfying those measures – often at the expense of other, more important organisational goals that are not measured. The best-known example is ‘teaching to the test’, a widespread phenomenon that has distorted primary and secondary education in the United States since the adoption of the No Child Left Behind Act of 2001.

Short-termism is another negative. Measured performance encourages what the US sociologist Robert K Merton in 1936 called ‘the imperious immediacy of interests where the actor’s paramount concern with the foreseen immediate consequences excludes consideration of further or other consequences’. In short, advancing short-term goals at the expense of long-range considerations. This problem is endemic to publicly traded corporations that sacrifice long-term research and development, and the development of their staff, to the perceived imperatives of the quarterly report.

  1. CAT 2018 Question Paper - Slot 2

    Of the following, which would have added the least depth to the author’s argument?

    1. An analysis of the reasons why metrics fixation is becoming popular despite its drawbacks.
    2. A comparative case study of metrics- and non-metrics-based evaluation, and its impact on the main goals of an organisation.
    3. More real-life illustrations of the consequences of employees and professionals gaming metrics-based performance measurement systems.
    4. Assessment of the pros and cons of a professional judgment-based evaluation system.
    Choice C
    More real-life illustrations of the consequences of employees and professionals gaming metrics-based performance measurement systems.

  2. CAT 2018 Question Paper - Slot 2

    Which of the following is NOT a consequence of the 'metric fixation' phenomenon mentioned in the passage?

    1. Short-term orientation induced by frequent measurement of performance.
    2. Finding a way to show better results without actually improving performance.
    3. Improving cooperation among employees leading to increased organisational effectiveness in the long run.
    4. Deviating from organisationally important objectives to measurable yet less important objectives.
    Choice C
    Improving cooperation among employees leading to increased organisational effectiveness in the long run.

  3. CAT 2018 Question Paper - Slot 2

    What main point does the author want to convey through the examples of the police officer and the surgeon?

    1. Some professionals are likely to be significantly influenced by the design of performance measurement systems.
    2. Metrics-linked rewards may encourage unethical behaviour among some professionals.
    3. The actions of police officers and surgeons have a significantly impact on society.
    4. Critical public roles should not be evaluated on metrics-based performance measures.
    Choice B
    Metrics-linked rewards may encourage unethical behaviour among some professionals.

  4. CAT 2018 Question Paper - Slot 2

    All of the following can be a possible feature of the No Child Left Behind Act of 2001, EXCEPT:

    1. school funding and sanctions are tied to yearly improvement shown on tests.
    2. standardised test scores can be critical in determining a student’s educational future.
    3. the focus is more on test-taking skills than on higher order thinking and problem-solving.
    4. assessment is dependent on the teacher's subjective evaluation of students' class participation.
    Choice D
    assessment is dependent on the teacher's subjective evaluation of students' class participation.

  5. CAT 2018 Question Paper - Slot 2

    What is the main idea that the author is trying to highlight in the passage?

    1. Long-term organisational goals should not be ignored for short-term measures of organisational success.
    2. All kinds of organisations are now relying on metrics to measure performance and to give rewards and punishments.
    3. Evaluating performance by using measurable performance metrics may misguide organisational goal achievement.
    4. Performance measurement needs to be precise and cost-effective to be useful for evaluating organisational performance.
    Choice C
    Evaluating performance by using measurable performance metrics may misguide organisational goal achievement.

Rings of Satrun

NOT everything looks lovelier the longer and closer its inspection. But Saturn does. It is gorgeous through Earthly telescopes. However, the 13 years of close observation provided by Cassini, an American spacecraft, showed the planet, its moons and its remarkable rings off better and better, revealing finer structures, striking novelties and greater drama.

By and large the big things in the solar system—planets and moons—are thought of as having been around since the beginning. The suggestion that rings and moons are new is, though, made even more interesting by the fact that one of those moons, Enceladus, is widely considered the most promising site in the solar system on which to look for alien life. If Enceladus is both young and bears life, that life must have come into being quickly. This is also believed to have been the case on Earth. Were it true on Enceladus, that would encourage the idea that life evolves easily when conditions are right.

One reason for thinking Saturn’s rings are young is that they are bright. The solar system is suffused with comet dust, and comet dust is dark. Leaving Saturn’s ring system (which Cassini has shown to be more than 90% water ice) out in such a mist is like leaving laundry hanging on a line downwind from a smokestack: it will get dirty. The lighter the rings are, the faster this will happen, for the less mass they contain, the less celestial pollution they can absorb before they start to discolour Jeff Cuzzi, a scientist at America’s space agency, NASA, who helped run Cassini, told the Lunar and Planetary Science Conference in Houston that combining the mass estimates with Cassini’s measurements of the density of comet-dust near Saturn suggests the rings are no older than the first dinosaurs, nor younger than the last of them—that is, they are somewhere between 200m and 70m years old.

That timing fits well with a theory put forward in 2016, by Matija Cuk of the SETI Institute, in California and his colleagues. They suggest that at around the same time as the rings came into being an old set of moons orbiting Saturn destroyed themselves, and from their remains emerged not only the rings but also the planet’s current suite of inner moons—Rhea, Dione, Tethys, Enceladus and Mimas.

Dr Cuk and his colleagues used computer simulations of Saturn’s moons’ orbits as a sort of time machine. Looking at the rate at which tidal friction is causing these orbits to lengthen they extrapolated backwards to find out what those orbits would have looked like in the past. They discovered that about 100m years ago the orbits of two of them, Tethys and Dione, would have interacted in a way that left the planes in which they orbit markedly tilted. But their orbits are untilted. The obvious, if unsettling, conclusion was that this interaction never happened—and thus that at the time when it should have happened, Dione and Tethys were simply not there. They must have come into being later.

  1. CAT 2018 Question Paper - Slot 2

    The phrase “leaving laundry hanging on a line downwind from a smokestack” is used to explain how the ringed planet’s:

    1. atmosphere absorbs comet dust.
    2. moons create a gap between the rings.
    3. rings discolour and darken over time.
    4. rings lose mass over time.
    Choice C
    rings discolour and darken over time.

  2. CAT 2018 Question Paper - Slot 2

    Data provided by Cassini challenged the assumption that:

    1. all big things in the solar system have been around since the beginning.
    2. new celestial bodies can form from the destruction of old celestial bodies
    3. Saturn’s ring system is composed mostly of water ice.
    4. there was life on earth when Saturn’s rings were being formed.
    Choice A
    all big things in the solar system have been around since the beginning.

  3. CAT 2018 Question Paper - Slot 2

    Based on information provided in the passage, we can infer that, in addition to water ice, Saturn’s rings might also have small amounts of:

    1. methane and rock particles.
    2. rock particles and comet dust.
    3. helium and methane.
    4. helium and comet dust.
    Choice B
    rock particles and comet dust.

  4. CAT 2018 Question Paper - Slot 2

    The main objective of the passage is to:

    1. provide evidence that Saturn’s rings and moons are recent creations.
    2. highlight the beauty, finer structures and celestial drama of Saturn’s rings and moons.
    3. demonstrate how the orbital patterns of Saturn’s rings and moons change over time.
    4. establish that Saturn’s rings and inner moons have been around since the beginning of time.
    Choice A
    provide evidence that Saturn’s rings and moons are recent creations.

  5. CAT 2018 Question Paper - Slot 2

    Based on information provided in the passage, we can conclude all of the following EXCEPT:

    1. Saturn’s lighter rings discolour faster than rings with greater mass.
    2. Thethys and Dione are less than 100 million years old.
    3. none of Saturn’s moons ever had suitable conditions for life to evolve.
    4. Saturn’s rings were created from the remains of older moons.
    Choice C
    none of Saturn’s moons ever had suitable conditions for life to evolve.

CAT VARC : CAT 2017 Question Paper Slot 1

Passage 1: Map Making

Understanding where you are in the world is a basic survival skill, which is why we, like most species come hard-wired with specialized brain areas to create cognitive maps of our surroundings. Where humans are unique, though, with the possible exception of honeybees, is that we try to communicate this understanding the world with others. We have along history of doing this by drawing maps – the earliest version yet discovered were scrawled on cave walls 14,000 years ago. Human cultures have been drawing them on stone tablets, papyrus, paper and now computer screens ever since.

Given such a long history of human map-making, it perhaps surprising that is only within the last few hundred years that north has been consistently considered to be at the top. In fact, for much of human history, north almost never appeared at the top, according to Jerry Brotton, a map historian... “North was rarely put at the top for the simple fact that north is where darkness comes from,” he says. “West is also very unlikely o be put at the top because west is where the sun disappears.”

Confusingly, early Chinese maps seem to buck this trend. But, Brotton, says, even though they did have compasses at the time, that isn’t the reason that they placed north at the top. Early Chinese compasses were actually oriented to point south, which was considered to be more desirable than deepest darkest north. But in Chinese maps, the emperor, who lived in the north of the country was always put at the top of the map, with everyone else, his loyal subjects, looking up towards him. “In Chinese culture the Emperor looks south because it’s where the winds come from, it’s a good direction. North is not very good but you are in a position of the subjection to the emperor, so you look up to him,” says Brotton.

Given that each culture has a very different idea of who, or what, they should look upto it’s perhaps not surprising that there is very little consistency in which way early maps pointed. In ancient Egyptian times the top of the world was east, the position of sunrise. Early Islamic maps favoured south at the top because most of the early Muslim cultures were north of Mecca, so they imagined looking up (south) towards it Christian maps from the same era (called Mappa Mundi) put east at the top, towards the Garden of Eden and with Jerusalem in the centre.

So when did everyone get together and decide that north was the top? It’s tempting to put it down to European explorers like Christopher Columbus and Ferdinand Megellan who were navigating by the North Star. But Brotton argues that these early explorers didn’t think of the world like that at all. “When Columbus describes the world it is in accordance with east being at the top,” he says “Columbus says he is going towards paradise, so his mentality is from a medieval mappa mundi.” We’ve got to remember, adds Brotton, that at the time, “no one knows what they are doing and where they are going.”

  1. CAT 2017 Question Paper - Slot 1

    Which one of the following best describes what the passage is trying to do?

    1. It questions on explanation about how maps are designed.
    2. It corrects a misconception about the way maps are designed.
    3. It critiques a methodology used to create maps.
    4. It explores some myths about maps.
    Choice B
    It corrects a misconception about the way maps are designed.

  2. CAT 2017 Question Paper - Slot 1

    Early maps did NOT put north at the top for all the following reasons EXCEPT

    1. North was the source of darkness
    2. South was favoured by some emperors
    3. East and south were more important for religious reasons for some civilisations
    4. East was considered by some civilisations to be a more positive direction
    Choice B
    South was favoured by some emperors

  3. CAT 2017 Question Paper - Slot 1

    According to the passage, early Chinese maps placed north at the top because

    1. the Chinese invented the compass and were aware of magnetic north.
    2. they wanted to show respect to the emperor.
    3. the Chinese emperor appreciated the winds from the south.
    4. north was considered the most desirable direction.
    Choice B
    they wanted to show respect to the emperor.

  4. CAT 2017 Question Paper - Slot 1

    It can be inferred from the passage that European explorers like Columbus and Megellan

    1. set the precedent for north-up maps.
    2. navigated by the compass.
    3. used an eastward orientation for religious reasons.
    4. navigated with the help of early maps.
    Choice C
    used an eastward orientation for religious reasons.

  5. CAT 2017 Question Paper - Slot 1

    Which one of the following about the northern orientation of modern maps is asserted in the passage

    1. The biggest contributory factor was the understanding of magnetic north
    2. The biggest contributory factor was the role of European explorers
    3. The biggest contributory factor was the influence of Christian maps
    4. The biggest contributory factor is not stated in the passage
    Choice D
    The biggest contributory factor is not stated in the passage

  6. CAT 2017 Question Paper - Slot 1

    The role of natural phenomena in influencing map-making conventions is seen most clearly in

    1. early Egyptian maps
    2. early Islamic maps
    3. early Chinese maps
    4. early Christian maps
    Choice A
    early Egyptian maps

Passage 2: Impact of Printed text and iPhone

I used a smartphone GPS to find my way through the cobblestoned maze of Geneva's Old Town, in search of a handmade machine that changed the world more than any other invention. Near a 13th-century cathedral in this Swiss city on the shores of a lovely lake, I found what I was looking for: a Gutenberg printing press. "This was the Internet of its day — at least as influential as the iPhone," said Gabriel de Montmollin, the director of the Museum of the Reformation, toying with the replica of Johann Gutenberg's great invention. [Before the invention of the printing press] it used to take four monks...up to a year to produce a single book. With the advance in movable type in 15th-century Europe, one press could crank out 3,000 pages a day.

Before long, average people could travel to places that used to be unknown to them — with maps! Medical information passed more freely and quickly, diminishing the sway of quacks...The printing press offered the prospect that tyrants would never be able to kill a book or suppress an idea. Gutenberg's brainchild broke the monopoly that clerics had on scripture. And later, stirred by pamphlets from a version of that same press, the American colonies rose up against a king and gave birth to a nation. So, a question in the summer of this 10th anniversary of the iPhone: has the device that is perhaps the most revolutionary of all time given us a single magnificent idea? Nearly every advancement of the written word through new technology has also advanced humankind. Sure, you can say the iPhone changed everything. By putting the world's recorded knowledge in the palm of a hand, it revolutionized work, dining, travel and socializing. It made us more narcissistic — here's more of me doing cool stuff! — and it unleashed an army of awful trolls. We no longer have the patience to sit through a baseball game without that reach to the pocket. And one more casualty of Apple selling more than a billion phones in a decade's time: daydreaming has become a lost art.

For all of that, I'm still waiting to see if the iPhone can do what the printing press did for religion and democracy...the Geneva museum makes a strong case that the printing press opened more minds than anything else...it's hard to imagine the French or American revolutions without those enlightened voices in print...

Not long after Steve Jobs introduced his iPhone, he said the bound book was probably headed for history's attic. Not so fast. After a period of rapid growth in e-books, something closer to the medium for Chaucer's volumes has made a great comeback.

The hope of the iPhone, and the Internet in general, was that it would free people in closed societies. But the failure of the Arab Spring, and the continued suppression of ideas in North Korea, China and Iran, has not borne that out... The iPhone is still young. It has certainly been "one of the most important, world-changing and successful products in history, “ as Apple CEO. Tim Cook said. But I'm not sure if the world changed for the better with the iPhone — as it did with the printing press — or merely, changed.

  1. CAT 2017 Question Paper - Slot 1

    The printing press has been likened to the Internet for which one of the following reasons?

    1. It enabled rapid access to new information and the sharing of new ideas
    2. It represented new and revolutionary technology compared to the past
    3. It encouraged reading among people by giving them access to thousands of books
    4. It gave people access to pamphlets and literature in several languages
    Choice A
    It enabled rapid access to new information and the sharing of new ideas

  2. CAT 2017 Question Paper - Slot 1

    According to the passage, the invention of the printing press did all of the following EXCEPT

    1. Promoted the spread of enlightened political views across countries.
    2. Gave people direct access to authentic medical information and religious texts.
    3. Shortened the time taken to produce books and pamphlets.
    4. Enabled people to perform various tasks simultaneously.
    Choice D
    Enabled people to perform various tasks simultaneously.

  3. CAT 2017 Question Paper - Slot 1

    Steve Jobs predicted which one'of the following with the introduction of the iPhone?

    1. People would switch from reading on the Internet to reading on their iPhones.
    2. People would lose interest in historical and traditional classics.
    3. Reading printed books would become a thing of the past.
    4. The production of e-books would eventually fall.
    Choice C
    Reading printed books would become a thing of the past.

  4. CAT 2017 Question Paper - Slot 1

    "I'm still waiting to see if the iPhone can do what the printing press did for religion and democracy." The author uses which one of the following to indicate his uncertainty?

    1. The rise of religious groups in many parts of the world.
    2. The expansion in trolling and narcissism among users of the Internet.
    3. The continued suppression of free speech in closed societies.
    4. The decline in reading habits among those who use the device.
    Choice C
    The continued suppression of free speech in closed societies.

  5. CAT 2017 Question Paper - Slot 1

    The author attributes the French and American revolutions to the invention of the printing press because

    1. maps enabled large numbers of Europeans to travel and settle in the American continent.
    2. the rapid spread of information exposed people to new ideas on freedom and democracy.
    3. it encouraged religious freedom among the people by destroying the monopoly of religious leaders on the scriptures.
    4. it made available revolutionary strategies and opinions to the people.
    Choice B
    the rapid spread of information exposed people to new ideas on freedom and democracy.

  6. CAT 2017 Question Paper - Slot 1

    The main conclusion of the passage is that the new technology has

    1. some advantages, but these are outweighed by its disadvantages.
    2. so far not proved as successful as the printing press in opening people's minds.
    3. been disappointing because it has changed society too rapidly.
    4. been more wasteful than the printing press because people spend more time daydreaming or surfing.
    Choice B
    so far not proved as successful as the printing press in opening people's minds.

Passage 3: American Malls

This year alone, more than 8,600 stores could close, according to industry estimates, many of them the brand -name anchor outlets that real estate developers once stumbled over themselves to court. Already there have been 5,300 retail closings this year... Sears Holdings—which owns Kmart—said in March that there's "substantial doubt" it can stay in business altogether, and will close 300 stores this year. So far this year, nine national retail chains have filed for bankruptcy.

Local jobs are a major casualty of what analysts are calling, with only a hint of hyperbole, the retail apocalypse. Since 2002, department stores have lost 448,000 jobs, a 25% decline, while the number of store closures this year is on pace to surpass the worst depths of the Great Recession. The growth of online retailers, meanwhile, has failed to offset those losses, with the ecommerce sector adding just 178,000 jobs over the past 15 years. Some of those jobs can be found in the massive distribution centers Amazon has opened across the country, often not too far from malls the company helped shutter.

But those are workplaces, not gathering places. The mall is both. And in the 61 years since the first enclosed one opened in suburban Minneapolis, the shopping mall has been where a huge swath of middle-class America went for far more than shopping. It was the home of first jobs and blind dates, the place for family photos and ear piercings, where goths and grandmothers could somehow walk through the same doors and find something they all liked. Sure, the food was lousy for you and the oceans of parking lots encouraged car-heavy development, something now scorned by contemporary planners. But for better or worse, the mall has been America's public square for the last 60 years.

So what happens when it disappears?

Think of your mall. Or think of the one you went to as a kid. Think of the perfume clouds in the department stores. The fountains splashing below the skylights. The cinnamon wafting from the food court. As far back as ancient Greece, societies have congregated around a central marketplace. In medieval Europe, they were outside cathedrals. For half of the 20th century and almost 20 years into the new one, much of America has found their agora on the terrazzo between Orange Julius and Sbarro, Waldenbooks and the Gap, Sunglass Hut and Hot Topic.

That mall was an ecosystem unto itself, a combination of community and commercialism peddling everything you needed and everything you didn't: Magic Eye posters, wind catchers. Air Jordans....

A growing number of Americans, however, don't see the need to go to any Macy's at all. Our digital lives are frictionless and ruthlessly efficient, with retail and romance available at a click. Malls were designed for leisure, abundance, ambling. You parked and planned to spend some time. Today, much of that time has been given over to busier lives and second jobs and apps that let you swipe right instead of haunt the food court. ' Malls, says Harvard business professor Leonard Schlesinger, "were built for patterns of social interaction that increasingly don't exist."

  1. CAT 2017 Question Paper - Slot 1

    The central idea of this passage is that:

    1. the closure of mails has affected the economic and social life of middle-class America.
    2. the advantages of malls outweigh their disadvantages.
    3. malls used to perform a social function that has been lost.
    4. malls are closing down because people have found alternate ways to shop.
    Choice C
    malls used to perform a social function that has been lost.

  2. CAT 2017 Question Paper - Slot 1

    Why does the author say in paragraph 2, 'the massive distribution centers Amazon has opened across the country, often not too far from malls the company helped shutter'?

    1. To highlight the irony of the situation.
    2. To indicate that mails and distribution centres are located in the same area.
    3. To show that Amazon is helping certain brands go online.
    4. To indicate that the shopping habits of the American middle class have changed.
    Choice A
    To highlight the irony of the situation.

  3. CAT 2017 Question Paper - Slot 1

    In paragraph 1, the phrase "real estate developers once stumbled over themselves to court" suggests that they

    1. took brand-name anchor outlets to court.
    2. no longer pursue brand-name hanger outlets.
    3. malls are closing down because people have found alternate ways to shop.
    4. collaborated with one another to get brand-name anchor outlets.
    Choice B
    no longer pursue brand-name hanger outlets.

  4. CAT 2017 Question Paper - Slot 1

    The author calls the mall an ecosystem unto itself because

    1. people of all ages and from all walks of life went there.
    2. people could shop as well as eat in one place.
    3. it was a commercial space as well as a gathering place.
    4. it sold things that were needed as well as those that were not.
    Choice C
    it was a commercial space as well as a gathering place.

  5. CAT 2017 Question Paper - Slot 1

    Why does the author say that the mall has been America's public square?

    1. Malls did not bar anybody from entering the space.
    2. Malls were a great place to shop for a huge section of the middle class.
    3. Malls were a hangout place where families grew close to each other.
    4. Malls were a great place for everyone to gather and interact.
    Choice D
    Malls were a great place to shop for a huge section of the middle class.

  6. CAT 2017 Question Paper - Slot 1

    The author describes 'Perfume clouds in the department stores' in order to

    1. evoke memories by painting a picture of malls
    2. describe the smells and sights of mails
    3. emphasise that all brands were available under one roof.
    4. show that malls smelt good because of the various stores and food court.
    Choice A
    evoke memories by painting a. picture of mails

Passage 4: Evolutionary diversity

Scientists have long recognised the incredible diversity within a species. But they thought it reflected evolutionary changes that unfolded imperceptibly, over millions of years. That divergence between populations within a species was enforced, according to Ernst Mayr, the great evolutionary biologist of the 1940s, when a population was separated from the rest of the species by a mountain range or a desert, preventing breeding across the divide over geologic scales of time. Without the separation, gene flow was relentless. But as the separation persisted, the isolated population grew apart and speciation occurred.

In the mid-1960s, the biologist Paul Ehrlich - author of The Population Bomb (1968) - and his Stanford University colleague Peter Raven challenged Mayr's ideas about speciation. They had studied checkerspot butterflies living in the Jasper Ridge Biological Preserve in California, and it soon became clear that they were not examining a single population. Through years of capturing, marking and then recapturing the butterflies, they were able to prove that within the population, spread over just 50 acres of suitable checkerspot habitat, there were three groups that rarely interacted despite their very close proximity.

Among other ideas, Ehrlich and Raven argued in a now classic paper from 1969 that gene flow was not as predictable and ubiquitous as Mayr and his cohort maintained, and thus evolutionary divergence between neighbouring groups in a population was probably common. They also asserted that isolation and gene flow were less important to evolutionary divergence than natural selection (when factors such as mate choice, weather, disease or predation cause better-adapted individuals to survive and pass on their successful genetic traits). For example, Ehrlich and Raven suggested that, without the force of natural selection, an isolated population would remain unchanged and that, in other scenarios, natural selection could be strong enough to overpower gene flow...

  1. CAT 2017 Question Paper - Slot 1

    Which of the following best sums up Ehrlich and Raven's argument in their classic 1969 paper?

    1. Ernst Mayr was wrong in identifying physical separation as the cause of species diversity
    2. Checkerspot butterflies in the 50-acre Jasper Ridge Preserve formed three groups that rarely interacted with each other
    3. While a factor, isolation was not as important to speciation as natural selection
    4. Gene flow is less common and more erratic than Mayr and his colleagues claimed.
    Choice C
    While a factor, isolation was not as important to speciation as natural selection

  2. CAT 2017 Question Paper - Slot 1

    All of the following statements are true according to the passage EXCEPT

    1. Gene flow contributes to evolutionary divergence.
    2. The Population Bomb questioned dominant ideas about species diversity.
    3. Evolutionary changes unfold imperceptibly over time.
    4. Checkerspot butterflies are known to exhibit speciation while living in close proximity.
    Choice B
    The Population Bomb questioned dominant ideas about species diversity.

  3. CAT 2017 Question Paper - Slot 1

    The author discusses Mayr, Ehrlich and Raven to demonstrate that

    1. evolution is a sensitive and controversial topic.
    2. Ehrlich and Raven's ideas about evolutionary divergence are widely accepted by scientists.
    3. the causes of speciation are debated by scientists.
    4. checkerspot butterflies offer the best example of Ehrlich and Raven's ideas about speciation.
    Choice C
    the causes of speciation are debated by scientists.

Passage 5: Olympics

Do sports mega events like the summer Olympic Games benefit the host city economically? It depends, but the prospects are less than rosy. The trick is converting...several billion dollars in operating costs during the 17-day fiesta of the Games into a basis for long-term economic returns. These days, the summer Olympic Games themselves generate total revenue of $4 billion to $5 billion, but the lion's share of this goes to the International Olympics Committee, the National Olympics Committees and the International Sports Federations. Any economic benefit would have to flow from the value of the Games as an advertisement for the city, the new transportation and communications infrastructure that was created for the Games, or the ongoing use of the new facilities.

Evidence suggests that the advertising effect is far from certain. The infrastructure benefit depends on the initial condition of the city and the effectiveness of the planning. The facilities benefit is dubious at best for buildings such as velodromes or natatoriums and problematic for 100,000-seat Olympic stadiums. The latter require a conversion plan for future use, the former are usually doomed to near vacancy. Hosting the summer Games generally requires 30-plus sports venues and dozens of training centers. Today, the Bird's Nest in Beijing sits virtually empty, while the Olympic Stadium in Sydney costs some $30 million a year to operate.

Part of the problem is that Olympics planning takes place in a frenzied and time-pressured atmosphere of intense competition with the other prospective host cities — not optimal conditions for contemplating the future shape of an urban landscape. Another part of the problem is that urban land is generally scarce and growing scarcer. The new facilities often stand for decades or longer. Even if they have future use, are they the best use of precious urban real estate?

Further, cities must consider the human cost. Residential areas often are razed and citizens relocated (without adequate preparation or compensation). Life is made more hectic and congested. There are, after all, other productive uses that can be made of vanishing fiscal resources.

  1. CAT 2017 Question Paper - Slot 1

    The central point in the first paragraph is that the economic benefits of the Olympic Games

    1. are shared equally among the three organising committees.
    2. accrue mostly through revenue from advertisements and ticket sales.
    3. accrue to host cities, if at all, only in the long term.
    4. are usually eroded by expenditure incurred by the host city.
    Choice C
    accrue to host cities, if at all, only in the long term.

  2. CAT 2017 Question Paper - Slot 1

    Sports facilities built for the Olympics are not fully utilised after the Games are over because

    1. their scale and the costs of operating them are large.
    2. their location away from the city centre usually limits easy access.
    3. the authorities do not adapt them to local conditions.
    4. they become outdated having being built with little planning and under time pressure.
    Choice A
    their scale and the costs of operating them are large.

  3. CAT 2017 Question Paper - Slot 1

    The author feels that the Games place a burden on the host city for all of the following reasons EXCEPT that

    1. they divert scarce urban land from more productive uses.
    2. they involve the demolition of residential structures to accommodate sports facilities and infrastructure.
    3. the finances used to fund the Games could be better used for other purposes.
    4. the influx of visitors during the Games places a huge strain on the urban infrastructure.
    Choice D
    the influx of visitors during the Games places a huge strain on the urban infrastructure.

CAT VARC : CAT 2017 Question Paper Slot 2

Passage 1: Creativity

Creativity is at once our most precious resource and our most inexhaustible one. As anyone who has ever spent any time with children knows, every single human being is born creative; every human being is innately endowed with the ability to combine and recombine data, perceptions, materials and ideas, and devise new ways of thinking and doing.What fosters creativity? More than anything else: the presence of other creative people. The big myth is that creativity is the province of great individual gen.iuses. In. fact creativity is a social process. Our biggest creative breakthroughs come when people learn from, compete with, and collaborate with other people.

Cities are the true fonts of creativity... With their diverse populations, dense social networks, and public spaces where people can meet spontaneously and serendipitously, they spark and catalyze new ideas. With their infrastructure for finance, organization and trade, they allow those ideas to be swiftly actualized.

As for what staunches creativity, that's easy, if ironic. It's the very institutions that we build to manage, exploit and perpetuate the fruits of creativity — our big bureaucracies, and sad to say, too many of our schools. Creativity is disruptive; schools and organizations are regimented, standardized and stultifying.

The education expert Sir Ken Robinson points to a 1968 study reporting on a group of 1,600 children who were tested over time for their ability to think in out-of-the-box ways. When the children were between 3 and 5 years old, 98 percent achieved positive scores. When they were 8 to 10, only 32 percent passed the same test, and only 10 percent at 13 to 15. When 280,000 25-year-olds took the test, just 2 percent passed. By the time we are adults, our creativity has been wrung out of us.

I once asked the great urbanist Jane Jacobs what makes some places more creative than others. She said, essentially, that the question was an easy one. All cities, she said, were filled with creative people; that's our default state as people. But some cities had more than their shares of leaders, people and institutions that blocked out that creativity. She called them "squelchers."

Creativity (or the lack of it) follows the same general contours of the great socio-economic divide — our rising inequality — that plagues us. According to my own estimates, roughly a third of us across the United States, and perhaps as much as half of us in our most creative cities — are able to do work which engages our creative faculties to some extent, whether as artists, musicians, writers, techies, innovators, entrepreneurs, doctors, lawyers, journalists or educators — those of us who work with our minds. That leaves a group that I term "the other 66 percent," who toil in low-wage rote and rotten jobs — if they have jobs at all — in which their creativity is subjugated, ignored or wasted.

Creativity itself is not in danger. It's flourishing is all around us — in science and technology, arts and culture, in our rapidly revitalizing cities. But we still have a long way to go if we want to build a truly creative society that supports and rewards the creativity of each and every one of us.

  1. CAT 2017 Question Paper - Slot 2

    In the author's view, cities promote human creativity for all the following reasons EXCEPT that they

    1. contain spaces that enable people to meet and share new ideas.
    2. expose people to different and novel ideas, because they are home to varied groups of people.
    3. provide the financial and institutional networks that enable ideas to become reality.
    4. provide access to cultural activities that promote new and creative ways of thinking.
    Choice D
    provide access to cultural activities that promote new and creative ways of thinking.

  2. CAT 2017 Question Paper - Slot 2

    The author uses 'ironic' in the third paragraph to point out that

    1. people need social contact rather than isolation to nurture their creativity.
    2. institutions created to promote creativity eventually stifle it.
    3. the larger the creative population in a city, the more likely it is to be stifled.
    4. large bureaucracies and institutions are the inevitable outcome of successful cities.
    Choice B
    institutions created to promote creativity eventually stifle it.

  3. CAT 2017 Question Paper - Slot 2

    The central idea of this passage is that

    1. social interaction is necessary to nurture creativity.
    2. creativity and ideas are gradually declining in all societies.
    3. the creativity divide is widening in societies in line with socio-economic trends.
    4. more people should work in jobs that engage their creative faculties.
    Choice A
    social interaction is necessary to nurture creativity.

  4. CAT 2017 Question Paper - Slot 2

    Jane Jacobs believed that cities that are more creative

    1. have to struggle to retain their creativity.
    2. have to 'squelch' unproductive people and promote creative ones.
    3. have leaders and institutions that do not block creativity.
    4. typically do not start off as creative hubs.
    Choice C
    have leaders and institutions that do not block creativity.

  5. CAT 2017 Question Paper - Slot 2

    The 1968 study is used here to show that

    1. as they get older, children usually learn to be more creative.
    2. schooling today does not encourage creative thinking in children.
    3. the more children learn, the less creative they become.
    4. technology today prevents children from being creative.
    Choice B
    schooling today does not encourage creative thinking in children.

  6. CAT 2017 Question Paper - Slot 2

    The author's conclusions about the most 'creative cities' in the US (paragraph 6) are based on his assumption that

    1. people who work with their hands are not doing creative work
    2. more than half the population works in non-creative jobs.
    3. only artists, musicians, writers, and so on should be valued in a society.
    4. most cities ignore or waste the creativity of low-wage workers.
    Choice A
    people who work with their hands are not doing creative work

CAT VARC : CAT 2017 Question Paper Slot 2

Passage 2: Subnivium

During the frigid season... it's often necessary to nestle under a blanket to try to stay warm. The temperature difference between the blanket and the air outside is so palpable that we often have trouble leaving our warm refuge. Many plants and animals similarly hunker down, relying on snow cover for safety from winter's harsh conditions. The small area between the snowpack and the ground, called the subnivium... might be the most important ecosystem that you have never heard of.

The subnivium is so well-insulated and stable that its temperature holds steady at around 32 degree Fahrenheit (0 degree Celsius). Although that might still sound cold, a constant temperature of 32 degree Fahrenheit can often be 30 to 40 degrees warmer than the air temperature during the peak of winter. Because of this large temperature difference, a wide variety of species...depend on the subnivium for winter protection.

For many organisms living in temperate and Arctic regions, the difference between being under the snow or outside it is a matter of life and death. Consequently, disruptions to the subnivium brought about by climate change will affect everything from population dynamics to nutrient cycling through the ecosystem.

The formation and stability of the subnivium requires more than a few flurries. Winter ecologists have suggested that eight inches of snow is necessary to develop a stable layer of insulation. Depth is not the only factor, however. More accurately, the stability of the subnivium depends on the interaction between snow depth and snow density. Imagine being under a stack of blankets that are all flattened and pressed together. When compressed, the blankets essentially form one compacted layer. In contrast, when they are lightly placed on top of one another, their insulative capacity increases because the air pockets between them trap heat. Greater depths of low-density snow are therefore better at insulating the ground.

Both depth and density of snow are sensitive to temperature. Scientists are now beginning to explore how climate change will affect the subnivium, as well as the species that depend on it. At first glance, warmer winters seem beneficial for species that have difficulty surviving subzero temperatures; however, as with most ecological phenomena, the consequences are not so straightforward. Research has shown that the snow season (the period when snow is more likely than rain) has become shorter since l970. When rain falls on snow, it increases the density of the snow and reduces its insulative capacity. Therefore, even though winters are expected to become warmer overall from future climate change, the subnivium will tend to become colder and more variable with less protection from the above-ground temperatures.

The effects of a colder subnivium are complex... For example, shrubs such as crowberry and alpine azalea that grow along the forest floor tend to block the wind and so retain higher depths of snow around them. This captured snow helps to keep soils insulated and in turn increases plant decomposition and nutrient release. In field experiments, researchers removed a portion. of the snow cover to investigate the importance of the subnivium's insulation. They found that soil frost in the snow-free area resulted in damage to plant roots and sometimes even the death of the plant.

  1. CAT 2017 Question Paper - Slot 2

    The purpose of this passage is to

    1. introduce readers to a relatively unknown ecosystem: the subnivium.
    2. explain how the subnivium works to provide shelter and food to several species.
    3. outline the effects of climate change on the subnivium.
    4. draw an analogy between the effect of blankets on humans and of snow cover on species living in the subnivium.
    Choice C
    outline the effects of climate change on the subnivium.

  2. CAT 2017 Question Paper - Slot 2

    All of the following statements are true EXCEPT

    1. Snow depth and Snow density both influence the stability of the subnivium.
    2. Climate change has some positive effects on the subnivium.
    3. The subnivium maintains a steady temperature that can be 30 to 40 degrees warmer than the winter air temperature.
    4. Researchers have established the adverse effects of dwindling snow cover on the subnivium.
    Choice B
    Climate change has some positive effects on the subnivium.

  3. CAT 2017 Question Paper - Slot 2

    Based on this extract, the author would support which one of the following actions?

    1. The use of snow machines in winter to ensure snow cover of at least eight inches.
    2. Government action to curb climate change.
    3. Adding nutrients to the soil in winter.
    4. Planting more shrubs in areas of short snow season.
    Choice B
    Government action to curb climate change.

  4. CAT 2017 Question Paper - Slot 2

    In paragraph 6, the author provides the examples of crowberry and alpine azalea to demonstrate that

    1. Despite frigid temperatures, several species survive in temperate and Arctic regions.
    2. Due to frigid temperatures in the temperate and Arctic regions, plant species that survive tend to be shrubs rather than trees.
    3. The crowberry and alpine azalea are abundant in temperate and Arctic regions.
    4. The stability of the subnivium depends on several interrelated factors, including shrubs on the forest floor.
    Choice D
    The stability of the subnivium depends on several interrelated factors, including shrubs on the forest floor.

  5. CAT 2017 Question Paper - Slot 2

    Which one of the following statements can be inferred from the passage?

    1. In an ecosystem, altering any one element has a ripple effect on all others.
    2. Climate change affects temperate and Artie regions more than equatorial or arid ones.
    3. A compact layer of wool is warmer than a similarly compact layer of goose down.
    4. The loss of the subnivium, while tragic, will affect only temperate and Artic regions.
    Choice A
    In an ecosystem, altering any one element has a ripple effect on all others.

  6. CAT 2017 Question Paper - Slot 2

    In paragraph 1, the author uses blankets as a device to

    1. evoke the bitter cold of winter in the minds of readers.
    2. explain how blankets work to keep us warm.
    3. draw an analogy between blankets and the snow pack.
    4. alert readers to the fatal effects of excessive exposure to the cold
    Choice C
    draw an analogy between blankets and the snow pack.

CAT VARC : CAT 2017 Question Paper Slot 2

Passage 3: Electric Vehicles

The end of the age of the internal combustion engine is in sight. There are small signs everywhere: the shift to hybrid vehicles is already under way among manufacturers. Volvo has announced it will make no purely petrol-engined cars after 2019...and Tesla has just started selling its first electric car aimed squarely at the middle classes: the Tesla 3 sells for $35,000 in the US, and 400,000 people have put down a small, refundable deposit towards one. Several thousand have already taken delivery, and the company hopes to sell half a million more next year. This is a remarkable figure for a machine with a fairly short range and a very limited number of specialised charging stations.

Some of it reflects the remarkable abilities of Elon Musk, the company's founder, as a salesman, engineer, and a man able to get the most out his factory workers and the governments he deals with...Mr Musk is selling a dream that the world wants to believe in. This last may be the most important factor in the story. The private car is...a device of immense practical help and economic significance, but at the same time a theatre for myths of unattainable selffulfilment. The one thing you will never see in a car advertisement is traffic, even though that is the element in which drivers spend their lives. Every single driver in a traffic jam is trying to escape from it, yet it is the inevitable consequence of mass car ownership.

The sleek and swift electric car is at one level merely the most contemporary fantasy of autonomy and power. But it might also disrupt our exterior landscapes nearly as much as the fossil fuel-engined car did in the last century. Electrical cars would of course pollute far less than fossil fuel-driven ones; instead of oil reserves, the rarest materials for batteries would make undeserving despots and their dynasties fantastically rich. Petrol stations would disappear. The air in cities would once more be breathable and their streets as quiet as those of Venice. This isn't an unmixed good. Cars that were as silent as bicycles would still be as dangerous as they are now to anyone they hit without audible warning.

The dream goes further than that. The electric cars of the future will be so thoroughly equipped with sensors and reaction mechanisms that they will never hit anyone. Just as brakes don't let you skid today, the steering wheel of tomorrow will swerve you away from danger before you have even noticed it...

This is where the fantasy of autonomy comes full circle. The logical outcome of cars which need no driver is that they will become cars which need no owner either. Instead, they will work as taxis do, summoned at will but only for the journeys we actually need. This the future towards which Uber...is working. The ultimate development of the private car will be to reinvent public transport. Traffic jams will be abolished only when the private car becomes a public utility. What then will happen to our fantasies of independence? We' ll all have to take to electrically powered bicycles.

  1. CAT 2017 Question Paper - Slot 2

    Which of the following statements best reflects the author's argument?

    1. Hybrid and electric vehicles signal the end of the age of internal combustion engines.
    2. Elon Musk is a remarkably gifted salesman.
    3. The private car represents an unattainable myth of independence.
    4. The future Uber car will be environmentally friendlier than even the Tesla.
    Choice C
    The private car represents an unattainable myth of independence.

  2. CAT 2017 Question Paper - Slot 2

    The author points out all of the following about electric cars EXCEPT

    1. Their reliance on rare materials for batteries will support despotic rule.
    2. They will reduce air and noise pollution.
    3. They will not decrease the number of traffic jams.
    4. They will ultimately undermine rather than further driver autonomy.
    Choice D
    They will ultimately undermine rather than further driver autonomy

  3. CAT 2017 Question Paper - Slot 2

    According to the author, the main reason for Tesla's remarkable sales is that

    1. in the long run, the Tesla is more cost effective than fossil fuel-driven cars.
    2. the US government has announced a tax subsidy for Tesla buyers.
    3. the company is rapidly upscaling the number of specialised charging stations for customer convenience.
    4. people believe in the autonomy represented by private cars.
    Choice D
    people believe in the autonomy represented by private cars.

  4. CAT 2017 Question Paper - Slot 2

    The author comes to the conclusion that

    1. car drivers will no longer own cars but will have to use public transport.
    2. cars will be controlled by technology that is more efficient than car drivers.
    3. car drivers dream of autonomy but the future may be public transport.
    4. electrically powered bicycles are the only way to achieve autonomy in transportation.
    Choice C
    car drivers dream of autonomy but the future may be public transport.

  5. CAT 2017 Question Paper - Slot 2

    In paragraphs 5 and 6, the author provides the example of Uber to argue that

    1. in the future, electric cars will be equipped with mechanisms that prevent collisions.
    2. in the future, traffic jams will not exist.
    3. in the future, the private car will be transformed into a form of public transport.
    4. in the future, Uber rides will outstrip Tesla sales.
    Choice C
    in the future, the private car will be transformed into a form of public transport.

  6. CAT 2017 Question Paper - Slot 2

    In paragraph 6, the author mentions electrically powered bicycles to argue that

    1. if Elon Musk were a true visionary, he would invest funds in developing electric bicycles.
    2. our fantasies of autonomy might unexpectedly require us to consider electric bicycles.
    3. in terms of environmental friendliness and safety, electric bicycles rather than electric cars are the future.
    4. electric buses are the best form of public transport.
    Choice B
    our fantasies of autonomy might unexpectedly require us to consider electric bicycles.

Passage 4: Typewriters

Typewriters are the epitome of a technology that has been comprehensively rendered obsolete by the digital age. The ink comes off the ribbon, they weigh a ton, and second thoughts are a disaster. But they are also personal, portable and, above all, private. Type a document and lock it away and more or less the only way anyone else can get it is if you give it to them. That is why the Russians have decided to go back to typewriters in some government offices, and why in the US, some departments have never abandoned them. Yet it is not just their resistance to algorithms and secret surveillance that keeps typewriter production lines — well one, at least — in business (the last British one closed a year ago). Nor is it only the nostalgic appeal of the metal body and the stout well-defined keys that make them popular on eBay. A typewriter demands something particular: attentiveness. By the time the paper is loaded, the ribbon tightened, the carriage returned, the spacing and the margins set, there's a big premium on hitting the right key. That means sorting out ideas, pulling together a kind of order and organising details before actually striking off. There can be no thinking on screen with a typewriter. Nor are there any easy distractions. No online shopping. No urgent emails. No Twitter. No need even for electricity — perfect for writing in a remote hideaway. The thinking process is accompanied by the encouraging clack of keys, and the ratchet of the carriage return. Ping!

  1. CAT 2017 Question Paper - Slot 2

    Which one of the following best describes what the passage is trying to do?

    1. It describes why people continue to use typewriters even in the digital age.
    2. It argues that typewriters will continue to be used even though they are an obsolete technology.
    3. It highlights the personal benefits of using typewriters.
    4. It shows that computers offer fewer options than typewriters.
    Choice A
    It describes why people continue to use typewriters even in the digital age.

  2. CAT 2017 Question Paper - Slot 2

    According to the passage, some governments still use typewriters because:

    1. they do not want to abandon old technologies that may be useful in the future.
    2. they want to ensure that typewriter production lines remain in business.
    3. they like the nostalgic appeal of typewriter.
    4. they can control who reads the document.
    Choice D
    they can control who reads the document.

  3. CAT 2017 Question Paper - Slot 2

    The writer praises typewriters for all the following reasons EXCEPT

    1. Unlike computers, they can only be used for typing.
    2. You cannot revise what you have typed on a typewriter.
    3. Typewriters are noisier than computers.
    4. Typewriters are messier to use than computers.
    Choice D
    Typewriters are messier to use than computers.

Passage 5: Viking age

Despite their fierce reputation, Vikings may not have always been the plunderers and pillagers popular culture imagines them to be. In fact, they got their start trading in northern European markets, researchers suggest.

Combs carved from animal antlers, as well as comb manufacturing waste and raw antler material has turned up at three archaeological sites in Denmark, including a medieval marketplace in the city of Ribe. A team of researchers from Denmark and the U.K. hoped to identify the species of animal to which the antlers once belonged by analyzing collagen proteins in the samples and comparing them across the animal kingdom, Laura Geggel reports for LiveScience. Somewhat surprisingly, molecular analysis of the artifacts revealed that some combs and other material had been carved from reindeer antlers.... Given that reindeer (Rangifer tarandus) don't live in Denmark, the researchers posit that it arrived on Viking ships from Norway. Antler craftsmanship, in the form of decorative combs, was part of Viking culture. Such combs served as symbols of good health, Geggel writes. The fact that the animals shed their antlers also made them easy to collect from the large herds that inhabited Norway.

Since the artifacts were found in marketplace areas at each site it's more likely that the Norsemen came to trade rather than pillage. Most of the artifacts also date to the 780s, but some are as old as 725. That predates the beginning of Viking raids on Great Britain by about 70 years. (Traditionally, the so-called "Viking Age" began with these raids in 793 and ended with the Norman conquest of Great Britain in l066.) Archaeologists had suspected that the Vikings had experience with long maritime voyages [that] might have preceded their raiding days. Beyond Norway, these combs would have been a popular industry in Scandinavia as wela: It' s possible that the antler combs represent a larger trade network, where the Norsemen supplied raw material to craftsmen in Denmark and elsewhere.

  1. CAT 2017 Question Paper - Slot 2

    The primary purpose of the passage is:

    1. to explain the presence of reindeer antler combs in Denmark.
    2. to contradict the widely-accepted beginning date for the Viking Age in Britain, and propose an alternate one.
    3. to challenge the popular perception of Vikings as raiders by using evidence that suggests their early trade relations with Europe.
    4. to argue that besides being violent pillagers„Vikings were also skilled craftsmen and efficient traders.
    Choice C
    to challenge the popular perception of Vikings as raiders by using evidence that suggests their early trade relations with Europe.

  2. CAT 2017 Question Paper - Slot 2

    The evidence - "Most of the artifacts also date to the 780s, but some are as old as 725" — has been used in the passage to argue that:

    1. the beginning date of the Viking Age should be changed from 793 to 725.
    2. the Viking raids started as early as 725.
    3. some of the antler artifacts found in Denmark and Great Britain could have come from Scandinavia.
    4. the Vikings' trade relations with Europe pre-dates the Viking raids.
    Choice D
    the Vikings' trade relations with Europe pre-dates the Viking raids.

  3. CAT 2017 Question Paper - Slot 2

    All of the following hold true for Vikings EXCEPT

    1. Vikings brought reindeer from Norway to Denmark for trade purposes.
    2. Before becoming the raiders of northern Europe, Vikings had trade relations with European nations.
    3. Antler combs, regarded by the Vikings as a symbol of good health, were part of the Viking culture.
    4. Vikings, once upon a time, had trade relations with Denmark and Scandinavia.
    Choice A
    Vikings brought reindeer from Norway to Denmark for trade purposes.


CAT Questions | CAT Quantitative Aptitude

CAT Questions | CAT DILR

CAT Questions | Verbal Ability for CAT


Where is 2IIM located?

2IIM Online CAT Coaching
A Fermat Education Initiative,
58/16, Indira Gandhi Street,
Kaveri Rangan Nagar, Saligramam, Chennai 600 093

How to reach 2IIM?

Phone: (91) 44 4505 8484
Mobile: (91) 99626 48484
WhatsApp: WhatsApp Now
Email: prep@2iim.com